Environmental Health
Environmental Health
Environmental Health
David Ball
PUBLIC HEALTH
UNDERSTANDING PUBLIC HEALTH
SERIES EDITORS: NICK BLACK & ROSALIND RAINE
Health Policy
will deal effectively with a seemingly Understanding Public Health
never-ending supply of hazards which is an innovative series of
impinge on health and wellbeing. This twenty books, published by
book provides a multidisciplinary window Open University Press in
onto environmental policy and its collaboration with the
formulation. From this you will observe London School of Hygiene
both order which exists at the centre and & Tropical Medicine.
controversies around the borders. It provides self-directed
This book considers: learning covering the major
◗ Key threats to human health from issues in public health
the physical environment affecting low, middle and
◗ Policies that might be pursued to high income countries.
minimise those risks The series is aimed at those
◗ How risks can be identified and studying public health,
quantified either by distance learning
◗ How such information can be or more traditional
communicated to the public methods, as well as public
◗ How health impact assessments can health practitioners and David Ball
be carried out policy makers.
◗ How risks can be managed and
regulated
Throughout the world, recognition of the importance of public health to sustainable, safe and
healthy societies is growing. The achievements of public health in nineteenth-century Europe were
for much of the twentieth century overshadowed by advances in personal care, in particular in hos-
pital care. Now, with the dawning of a new century, there is increasing understanding of the inevit-
able limits of individual health care and of the need to complement such services with effective public
health strategies. Major improvements in people’s health will come from controlling communicable
diseases, eradicating environmental hazards, improving people’s diets and enhancing the availability
and quality of effective health care. To achieve this, every country needs a cadre of knowledgeable
public health practitioners with social, political and organizational skills to lead and bring about
changes at international, national and local levels.
This is one of a series of 20 books that provides a foundation for those wishing to join in and
contribute to the twenty-first-century regeneration of public health, helping to put the concerns and
perspectives of public health at the heart of policy-making and service provision. While each book
stands alone, together they provide a comprehensive account of the three main aims of public health:
protecting the public from environmental hazards, improving the health of the public and ensuring
high quality health services are available to all. Some of the books focus on methods, others on key
topics. They have been written by staff at the London School of Hygiene & Tropical Medicine with
considerable experience of teaching public health to students from low, middle and high income
countries. Much of the material has been developed and tested with postgraduate students both in
face-to-face teaching and through distance learning.
The books are designed for self-directed learning. Each chapter has explicit learning objectives, key
terms are highlighted and the text contains many activities to enable the reader to test their own
understanding of the ideas and material covered. Written in a clear and accessible style, the series will
be essential reading for students taking postgraduate courses in public health and will also be of
interest to public health practitioners and policy-makers.
Titles in the series
Analytical models for decision making: Colin Sanderson and Reinhold Gruen
Controlling communicable disease: Norman Noah
Economic analysis for management and policy: Stephen Jan, Lilani Kumaranayake, Jenny
Roberts, Kara Hanson and Kate Archibald
Economic evaluation: Julia Fox-Rushby and John Cairns (eds)
Environmental epidemiology: Paul Wilkinson (ed)
Environment, health and sustainable development: Megan Landon
Environmental health policy: David Ball
Financial management in health services: Reinhold Gruen and Anne Howarth
Global change and health: Kelley Lee and Jeff Collin (eds)
Health care evaluation: Sarah Smith, Don Sinclair, Rosalind Raine and Barnaby Reeves
Health promotion practice: Wendy Macdowall, Chris Bonell and Maggie Davies (eds)
Health promotion theory: Maggie Davies and Wendy Macdowall (eds)
Introduction to epidemiology: Lucianne Bailey, Katerina Vardulaki, Julia Langham and
Daniel Chandramohan
Introduction to health economics: David Wonderling, Reinhold Gruen and Nick Black
Issues in public health: Joceline Pomerleau and Martin McKee (eds)
Making health policy: Kent Buse, Nicholas Mays and Gill Walt
Managing health services: Nick Goodwin, Reinhold Gruen and Valerie Iles
Medical anthropology: Robert Pool and Wenzel Geissler
Principles of social research: Judith Green and John Browne (eds)
Understanding health services: Nick Black and Reinhold Gruen
Environmental
Health Policy
David Ball
Open University Press
McGraw-Hill Education
McGraw-Hill House
Shoppenhangers Road
Maidenhead
Berkshire
England
SL6 2QL
email:enquiries@openup.co.uk
world wide web: www.openup.co.uk
and Two Penn Plaza, New York, NY 10121-2289, USA
Section 1 Introduction 5
1 Introduction to environmental health policy 7
2 The emergence of risk assessment 14
Glossary 271
Index 277
Acknowledgements
Open University Press and the London School of Hygiene & Tropical Medicine have
made every effort to obtain permission from copyright holders to reproduce material in
this book and to acknowledge these sources correctly. Any omissions brought to our
attention will be remedied in future editions.
We would like to express our grateful thanks to the following copyright holders for
granting permission to reproduce material in this book.
Chapter 1
Table 1.1 from IRGC (2005).
Chapter 2
Extracts from Covello and Mumpower (1985: 4, 8, 9).
Chapter 3
Figure 3.1 from Ball (2000); Figure 3.2 from NRC Red Book (1983); Table 3.1 from BMA
(1987); extract from NRC Red Book (1983: 4–5).
Chapter 4
Extract from Ottoboni (1991: 26, 27); extract from Commonwealth of Australia (2002:
52–3).
Chapter 5
Extracts from Paustenbach (1989: 82–3, 86–7, 87–9); extract from Commonwealth of
Australia (2002: 78–80).
Chapter 6
Extracts from Covello and Merkhofer (1993: 92–4, 117); extract from Rodricks (1992:
16–17); Figure 6.5 from Rodricks (1992: 28).
Chapter 7
Extract from Paustenbach (1989: 90–1); Table 7.1 from Rodricks (1992: 188–9); Figure 7.1
from HMSO (1995); Figures 7.2 and 7.3 from HMSO (1989).
viii Acknowledgements
Chapter 8
Table 8.1 from Environment International 28 (2002); Figure 8.1 from The Green Book
(2003); Figure 8.3 from Ball DJ and Soby B (1995) Valuing consumer safety. Inter-
national Journal for Consumer Safety 2(3): 171–131. Taylor & Francis Ltd, http://
www.tandf.co.uk/journals; Figure 8.4 from Accident Analysis and Prevention 32 (2000);
extract from Risk Analysis 25(4) (2005).
Chapter 9
Figures 9.1, 9.2 and 9.3 from UNEP (1985); extract from UNEP (1985); extract and Figure
9.5 from Stokell et al. (1991); Figure 9.4 from NRPB slide pack; extract and Figure 9.6
from Ball DJ and Goats GC (1996) Risk management and consumer safety. International
Journal for Consumer Safety 3(3): 111–124. Taylor & Francis Ltd, http://www.tandf.co.uk/
journals; Figure 9.7 from ISOE.
Chapter 10
Extract from WHO (1987); Figure 10.2 from HMSO (1988); extract from Ball DJ and
Goats GC (1996) Risk management and consumer safety. International Journal for Con-
sumer Safety 3(3): 111–124. Taylor & Francis Ltd, http://www.tandf.co.uk/journals;
extracts from Kocher (1996); Figure 10.3 from MCA (2006); extract and Figures 10.4 and
10.5 from Travis et al. (1987).
Chapter 11
Extracts from Schwartz and Thompson (1990: 17), Human and Ecological Risk Assessment.
6(4) (2000), and Arrow et al. (1996).
Chapter 12
Chapter 13
Extract from Adams (1995: 34); extract from Schwartz and Thompson (1990: 6); extract
from Adams and Thompson (2002: 18–22); Figures 13.1, 13.2, 13.3 and 13.4 from Adams
(1995: 34, 35, 36, 202).
Chapter 14
Figures 14.1, 14.2 and 14.3 from BMA (1998: 36, 51 and 82); extracts from Thérivel and
Partidário (1996: 4–5, 104–8); extracts from Joyce (2001: 8, 9–10).
Acknowledgements ix
Chapter 15
Table 15.1 from Ames et al. (1987); extract from Strategic Advisory Board (1990); Tables
15.3 and 15.4 from US Strategic Advisory Board (1990); extract from Clarence-Davies
(1996: 78, 79).
Chapter 16
Extracts from O’Brien (2000: 120–1, xiii–xiv, 131–2, 133–4, 143–5); extract from Sun-
stein (2002: 102–4).
Chapter 17
Table 17.1 from Risk Analysis 15(2) (1995); Figures 17.2 and 17.3 and extract from IRGC
(2005: 53, 13, 15); Figure 17.1 from NRC (1996: 28); extract from NRC (1996: 22–3);
Figure 17.4 from Lundgren (1994: 3); Figure 17.5 from Risk Analysis 23(5) (2003); extracts
from Löfstedt (2005; 19, 21).
Chapter 18
Extracts from Seedhouse (2000: 53–6, 77–8); Figure 18.1 © Ortwin Renn (2005); extract
from Graham (2001); extract from Lichtenstein et al. (1990).
Chapter 19
Extracts from UNDP (2006); extract from Kiyn (2006); Figure 19.1 and extract from
Coggan et al. (2000); extract from Nilsen (2004); Activity 19.4 is based on Seedhouse
(1997).
Chapter 20
Extracts and Table 20.1 from Graham and Wiener (1995); extract from Bial et al. (2000);
extract from O’Brien (2000).
Overview of the book
Introduction
For this reason, no single discipline could possibly provide a full description of the
roots and mechanisms of environmental health policy. This book, therefore, sets
out to provide a multidisciplinary window onto environmental policy and its for-
mulation. From this you will observe both the order that exists at the centre, and
the controversies around the borders. Since there is no such thing as a recipe for
making policy, every circumstance being unique, policy makers are best equipped
for the task if they fully understand the nature of the task, the tools at their disposal
and the likely complexities of the situations to be encountered.
Many of the themes raised in this book have been developed in companion vol-
umes in the series, often from a different perspective. To view things from different
perspectives is useful for a deeper understanding even if the accounts are not
entirely consistent. This book explores the roots of many controversies without
necessarily providing answers. What it will do is increase your insight and hence
ability to make those difficult choices demanded of anyone contributing to
environmental health policy.
It is accepted that you already will have your own views on environmental health
and related policies based on your own professional background and experience.
This will provide an interesting backcloth against which to work through the book.
However, because of the multidisciplinary approach, some of the material may be
very new, but no specialist knowledge is required. At the end, you should be in a
stronger position to interpret the complex technical and social issues surrounding
environmental health policy and be better equipped to form your own perspective
on them.
2 Overview of the book
Before beginning, let us ask why would you, or anyone, want to study environ-
mental health policy and what might be the benefits if you did? Listed here are
some reasons, though you might well be able to think of others, either now or later:
1 It might not always be obvious, but environmental health policies, even their
absence, affect us all.
2 Health and the environment are of great importance to most people.
3 The world faces major environmental health challenges at the local, regional
and global level.
4 Environmental health policies require and consume vast resources. There are
never enough resources to go around, so it is important to try to make the best
use of them.
5 Where environmental health policy is affecting people’s health and welfare,
those people will be more enfranchised if they understand the different ways in
which policy is formed.
6 If you aspire to work in the policy field, it is essential that you understand the
bases of policy, including the strengths and limitations of different approaches
to policy formulation, in order to do the best job.
7 There are many forces affecting the evolution of policy, some intentionally and
some inadvertently. If good policy decisions are to be made, these forces need to
be recognized and understood.
8 The indirect implications of environmental health policies on lifestyles, com-
munities and businesses are also significant. This needs to be observed and
considered.
This book is based on the conceptual outline of the ‘Environmental Health Policy’
unit taught at the London School of Hygiene & Tropical Medicine. It was reviewed,
and updated in 2005–6, and adapted for distance learning.
The book traces an evolving story, focusing on the past 20 years of development
right up to the present. The policy-making process continues to adjust, so being
aware of changes of emphasis, and the reasons for them, strengthens understand-
ing. Understanding is paramount if you are to implement effective policy. The
approach draws on the natural and biological sciences, economics, sociology,
psychology, communication, philosophy and politics. Do not be put off by this
– all is explained.
The book is comprised of seven sections and 20 chapters. Each chapter is set out in
this way:
• an overview
• a list of learning objectives
• a list of key terms
• a range of activities
• feedback on the activities
• a summary.
Overview of the book 3
All chapters have references, some of which you might want to follow up, though
the text is self-contained. A great deal of useful material can also be found on the
internet.
Although examples and case studies in this book are drawn from low, middle
and high income countries, you should be aware that most of the theory on
environmental health policy which is used internationally, has originated in
English-speaking, mainly higher income countries, and has spread around the
world through its use by international agencies, both public and private. Policy
making of the type described here also tends to be practised more at the level
of central and regional government, though this should not deter others from
seeking an understanding of the procedure, as everyone is a part of the system and
knowledge assists enfranchisement.
The description of the section and chapter contents will give you an idea of what
will follow.
Section 1, Introduction. Chapter 1 asks what is meant by ‘environmental health’ and
‘environmental health policy’. It points out choices which can be made regarding
the former, and, by way of an analysis of some general principles of environmental
health policy, identifies the key factors on the minds of most environmental policy
makers at the start of the twenty-first century. The chapter concludes by noting
that an impressive array of tools have been devised to help policy makers. Chapter 2
explores the background to one of these tools (others come later) which has
achieved notable prominence internationally, namely, health risk assessment.
How has it come about and why is it important now?
Section 2, Assessing environmental health risks. Chapters 3–7 constitute the main
scientific input to the book. Even for non-scientists with an interest in environ-
mental health policy, it is important to have an idea of where the scientific know-
ledge comes from and on what basis. Without that, you would not be able to assess
its merits and ask questions. Chapter 3 gives an overview of the now dominant
model of health risk assessment, breaking it down into its component stages.
Chapters 4–7 take you through each of these stages, giving you solid grounding
from which to move on to other topics.
Section 3, Rational action and environmental health policy. In this third section you
will move on from assessing risks to the various models that can help you decide,
once you have assessed them, what should be done. Should a risk be tolerated, and
if not, by how much should it be reduced? As a first step to answering this question,
Chapter 8 considers the contribution of economics to environmental decision
making. Building upon this, Chapters 9 and 10 then examine national and
international frameworks for decision making involving hazards of three kinds:
environmental, radiological, and safety. From this emerges a ‘rational’ model of
policy making, grounded in science and economics.
Section 4, Beyond the rational action approach. There is no avoiding the fact that
environmental health policy decisions are frequently controversial, and it would
be pointless to pretend otherwise. Indeed, the ‘rational’ model described in the
previous section does not find universal acceptance. To this end, Section 4 intro-
duces some very different ideas about how decisions are and should be made so
that the reasons for these disputes over policy can be understood. Chapter 11
describes arguments (and counter-arguments) that are lodged against the scientific
4 Overview of the book
Overview
In this introductory chapter you will first consider several definitions of ‘environ-
mental health’. It will help to clarify which definition is relevant to your own
interests, though it is possible that this may change as you progress. Whatever your
personal interest, however, it is important that you are aware of the different inter-
pretations that others might legitimately place on this term, since you will cer-
tainly encounter them. Second, you will examine the array of principles that
impact upon environmental health policy as currently practised. Finally, you will
receive a brief and preliminary introduction to the key tools recommended by
international agencies, governments and other organizations as aids for decision
makers in determining their policies.
Learning objectives
Key terms
Environmental health Those aspects of human health and disease that are determined by
factors in the environment.
Policy Broad statement of goals, objectives and means that create the framework for activity.
Often taking the form of explicit written documents but may also be implicit or unwritten.
Before studying environmental health policy, you need to consider what is meant
by the term ‘environmental health’ and what it means to you or your organization.
One definition can be found in the European Charter, Environment and Health
8 Introduction
ActivityExamine
1.1
Table 1.1 and, thinking about where you live and your own interests, answer
the following questions:
1 What aspects of environmental health do you consider should be included in your
working definition, and why?
2 Are there any persons or groups in the place in which you live or work who might
favour a different definition and, if so, why?
Feedback
Basically, it is a matter of choice as to which definition of environmental health you
select. There is no ‘correct’ answer. Table 1.1 shows how wide the possibilities can be.
Whatever you decide, it is important to be aware of where you draw the boundaries
and also why other people might draw them differently.
Introduction to environmental health policy 9
fairly common explanation might be: ‘A policy may . . . be considered as the inspir-
ation and guidance for action, a plan as a set of coordinated and timed objectives
for the implementation of the policy, and a programme as a set of projects in a
particular area’ (Wood and Djeddour 1992).
ActivityThinking
1.2
about your own situation, write down some underlying principles that might
help you develop a strategic policy on environmental health. This is quite a difficult
question at this stage if you are new to this field, so do not be concerned if you find it
hard. Later, it will become much easier.
Feedback
Many organizations list their guiding principles on the world-wide web. For example,
on its website: www.cieh.org, the London-based Chartered Institute of Environmental
Health (CIEH) lists its principles as:
• concerns for human health and well-being, and the desire to improve this through
appropriate environmental change and intervention;
• equity, and the wish to promote the benefits of the environmental health approach to
all members and sections of society including the most disadvantaged;
• precaution, because it is believed that prevention of harm is better than cure;
• an holistic approach, because of the linkage between environmental, social and
economic factors and their collective impact upon health and well-being;
• sustainability, in recognition of the need to create and maintain future conditions
which will secure health and well-being;
• civic participation, because of the requirement that environmental health policy
recognize the role of public concerns and the need to maintain openness and
transparency in decision-making processes;
• recognition of the forces of globalization, and the fact that environmental health
problems do not respect man-made borders.
The above principles adopted by the CIEH are of special interest because they refer
to many considerations which are on the minds of present-day environmental
health policy makers.
ActivityRead
1.3
through the CIEH principles carefully and make a note of those words and
phrases which you think might be important to policy makers.
Introduction to environmental health policy 11
Feedback
You might have noted the following features of the principles. Do not be concerned if
you did not identify all or even many of them, as you now are only at the point where
the stage is being set. Nonetheless, it will help you, as you progress, to be able to link
sections of this book to sets of principles such as these. What might at first seem like
disparate topics will more easily be seen to belong to a whole.
The first principle refers to both health and welfare, a theme also picked up by the
World Health Organization (WHO) in its writings. The implication is that the interest
is not only in the absence of disease, illness and injury, but also wider issues of social
welfare and fulfilment. More subtly, the first principle also uses the word ‘appropriate’
when talking about environmental change and intervention. The matter of what is an
‘appropriate’ change or intervention is of deep significance to policy makers, as you
will discover as you progress through this book.
The second principle raises the issue of ‘equity’. Equity considerations are now higher
on the agenda of policy makers than previously. As you read on, you will understand
how this situation arose and the arguments around it. Likewise, the third principle raises
an entirely different principle of ‘precaution’. It is probable that you have heard of the
‘precautionary principle’, and this alludes to it. Policy makers need to understand what
this principle is, and when and how to apply it.
The fourth principle introduces the idea of an ‘holistic approach’, one which is said to
recognize the link between environmental, social and economic factors in affecting
health. This, as you will discover, marks another important shift which has occurred in
thinking in environmental health policy making, and has resulted in the development of
many new tools to aid policy makers. The fifth principle brings in yet another concept
of great influence in modern policy making, that of ‘sustainability’.
The sixth principle ventures onto more new ground, that of civic participation in
environmental health policy and the need to recognize public concerns and be
accountable and transparent. This is why environmental health policy makers need to
be familiar with the psychology of risk perception, and methods of risk communication
and deliberation. The final principle points out that environmental health issues are
not confined to the local, but have regional and even global dimensions.
By the time you have finished this book, you will have studied every one of these
items and have much to say about them. Their commonality, through sets of
principles like these, is worth keeping in mind.
Policy tools
The above principles of the CIEH contain all the essential elements with which
present-day policy makers are likely to be concerned. Other agencies will use
different words, but the essence will be basically the same. To help policy makers
address these elements, many tools have been devised. You may already be aware,
or if not, will soon observe, that great emphasis is seen to have been placed upon
the use of risk assessment in particular, in documents published by agencies such
as the World Health Organization and the United Nations (UN). For example,
12 Introduction
Agenda 21, which was designed by the UN as a comprehensive plan of global action
aimed at integrating environmental and development issues, including health,
into decision making (see Chapter 19), makes extensive reference to the use of
risk assessment in determining action plans. Risk-based thinking, of which risk
assessment is a part, has, in fact, become an almost standard approach to decision
making and can be found in all countries of the world, particularly in national and
regional agencies, but is gradually spreading downwards to the local level. Because
of its importance, this book continues with a detailed description of the techniques
associated with this approach, before moving on to examine other techniques
including environmental impact assessment, health impact assessment, and social
impact assessment, which have been devised, sometimes in response to perceived
weaknesses in risk-based approaches, and at other times to complement them.
As a further indication of what lies ahead, Table 1.2 sets out a fairly typical
view of the necessary components of an environmental management framework.
Although based on a manufacturing company’s view of its responsibilities, general
environmental emphases are present. As you can see, under the human and
environmental safety element, the key tools identified are human health risk
assessment, which is the main concern here, and ecological risk assessment. How-
ever, other entries are pertinent to our interest, including the efficient use of
resources, and hence economic analysis, and addressing what are described as
‘societal concerns’. This brings in tools related to ‘understanding’ and ‘responding’,
all of which ties in with the guiding principles of the CIEH and other agencies
working in the field of environmental health policy.
Summary
In this introductory chapter you thought about the scope of your interests regard-
ing environmental health and noted that others might legitimately define their
interests in another way. You examined a typical set of principles of an agency
whose interest is environmental health policy, and noted how these principles
are concerned with a host of issues ranging from health (of course), to welfare,
appropriateness of interventions, equity, precaution, holism, sustainability, par-
ticipation and globalization. The promise has been made that all these will be
covered in this book. Finally, you are aware of the existence of an array of tools
which have been devised to help environmental health policy makers perform
their duties, and which will be described in later chapters.
References
International Risk Governance Council (2005) Risk Governance: Towards an Integrative Approach.
Geneva: IRGC.
White PR, De Smet B, Owens JW and Hindel P (1995) Environmental management in an
international consumer goods company. Resources, Conservation and Recycling 14(1):
171–84.
Wood C and Djeddour M (1992) Strategic environmental assessment: environmental
assessment of policies, plans and programmes. The Impact Assessment Bulletin 10(1): 3–22.
World Health Organization Regional Office for Europe (1990) Environment and Health: The
European Charter and Commentary. WHO Regional Publications, European Series No. 35.
Copenhagen: WHO Regional Office.
The emergence of risk
2 assessment
Overview
In the introductory chapter you learned that a range of tools have been devised to
help environmental health policy makers. Paramount among these is risk assess-
ment which is now a major environmental health policy tool used at the local,
national and international level as an aid for setting priorities. It is in operation in
many other sectors besides environmental health, and this has assisted its rise to
prominence. Although risk-based decision making has existed in some form for
millennia, the extent of its present-day application is unique. This chapter
describes influences affecting the development of risk assessment as an approach to
decision making and why its current status is so high. It concludes by examining
how the circumstances of risk assessment today differ from those of the past, and
by considering the range of applications within environmental health.
Learning objectives
Key terms
Risk assessment A method for estimating the probability or likelihood of a specified type of
harm occurring to an individual or a population.
Risk-based thinking An approach to decision making that relies in part upon risk assessment.
The origins of risk assessment can be traced back thousands of years, but it is only
in modern times that it has become so highly formalized and the dominant tool in
The emergence of risk assessment 15
ActivityThinking
2.1
about the country in which you live:
1 Which risks are of current concern?
2 Are these risks the same as those reported in the media?
Feedback
The answer to these questions will depend very much on the level of industrialization
and the culture in which you live.
1 In some high income countries, current preoccupations include risks associated with
the use of mobile phones and their associated transmitting stations, genetically modi-
fied crops, air pollution, global warming, food additives and lack of exercise. In low and
middle income countries concerns may well focus on the risk of natural disasters, the
supply and safety of water, the presence of disease, and the occurrence of injuries.
2 These issues attract a great deal of media attention, but this is not necessarily
because they pose the greatest threat. While the media often reflect public concern,
they have their own interests which may lead to a different perspective being presented
from that which is of concern within the population.
idea was firmly established that the probability or likelihood of future events could
be calculated, a logic which in fact had previously been denied or even discouraged.
For example, in 1792, LaPlace made an analysis of the probability of death with and
without smallpox vaccination.
However, the question as to why it took so long for probability theory to emerge is
not easy to answer. Covello and Mumpower provide several plausible explanations.
These include the requirements of the then developing commercial organizations,
particularly those dealing with shipping and life insurance, and limitations in
mathematical techniques, but they conclude that none of these alone is con-
vincing and point instead to an intriguing argument offered by David (1962) and
others, revolving around the attitude of the Christian Church. According to David
‘[there] seems to have been a taboo on speculations with regard to health, philo-
sophers implying that to count the sick or even the number of boys born was
impious in that it probed the inscrutable purpose of God’. Likewise, Grier (1980)
argues that much of the intellectual thinking about probability in the seventeenth
and eighteenth centuries was rooted in intense discussions in church debates over
the morality of charging interest on loans. Risk as a topic was caught up in these
ecclesiastical debates because one argument in favour of permitting interest to be
charged on loans was that the lender was taking a risk that his money might be lost.
This clerical matter, it seemed, needed to be resolved before risk could become a
legitimate subject for public discussion. It should be recalled that the penalties of
going against the doctrines of the Church could be, in those days, severe.
The development of risk-based thinking was also delayed by another factor. This
was the quest for causal links between adverse health effects and different types of
hazard (Covello and Mumpower 1985). The main methods of establishing causal
links had, throughout history, been based on observation, either involving per-
sonal trial and error as in tasting a new fruit, or in observing the effect upon
others, including animals, of different diets. On a more sophisticated level, the
precursors of modern epidemiological methods which seek to establish associ-
ations or cause–effect relationships by the observation of adverse health effects in
groups, can be traced back thousands of years to the eras of the Greeks and the
Romans, but it was only in the sixteenth to the eighteenth centuries that exten-
sive progress was made. This delay has been attributed to a lack of models of
scientific, biological, chemical and physical processes. However, at that time, there
were major intellectual developments underway in Europe during the period
known as the Enlightenment. Exponents of the Enlightenment believed that
human reason could be used to combat ignorance and superstition to build a
better world. Their principal targets were the beliefs of the established Church and
the domination of society by a hereditary aristocracy. For example, prior to this
period, it was believed that most illnesses, injuries, misfortunes and disasters
could best be explained in social, religious or magical terms. These views were
entrenched and resistant to change.
ActivityHaving
2.2
read this summary of the historical analysis by Vince Covello and Jeryl Mumpower,
write down a list of factors that slowed the emergence of risk-based methods.
The emergence of risk assessment 17
Feedback
The potential barriers include limitations of mathematical techniques especially prob-
ability theory, lack of demand prior to the emergence of the insurance industry, the
limited scientific understanding of the chemical, biological and physical processes which
could lead to ill health, and the belief that ill health and injuries were ‘punishments from
God’ and that ‘the work of God’ should not be a topic of investigation or speculation.
Recent developments
You can trace the origins of risk-based thinking back through time in many differ-
ent areas of human activity besides insurance and heavy engineering. Judith
Green (1997), for example, gives an outstanding analysis of our interpretation of
‘accidents’ and their transformation from acts of fate to predictable and hence
preventable occurrences.
Nonetheless, as the UK Cabinet Office’s Strategy Unit has observed (Cabinet Office
2002), in the context of the last century:
ActivityWrite
2.3
down an example of where you personally make a compromise between expos-
ing yourself to some risk and receiving a benefit. How do you justify this compromise?
Do the same for a risk that is the responsibility of an organization such as your
employer or your municipality.
Feedback
There are countless examples to choose from. Many sports that people play present high
injury risks which would not normally be tolerated, but people still play them because
they get enjoyment out of it and because it keeps them fit. Many people smoke cigarettes
while fully aware of the health risks, but apparently accept these in exchange for the
sensation it provides. Organizations also take risks when they invest effort and resources
in projects, since future success can never be guaranteed. In fact, you could say that every
action, not excluding the implementation of a new environmental health policy, intro-
duces risk, but without taking such chances, there would be no opportunity for progress.
The emergence of risk assessment 19
ActivityTable
2.4
2.1 shows a sample of the results from the work of Tengs and colleagues. What
inferences can you draw from this?
Feedback
Table 2.1 shows the net resource cost per year of life saved by the named intervention.
For example, the figure of $500 against influenza vaccination means that for every $500
spent on that measure, one extra year of life is gained by all those who received the
treatment (it does not mean that each vaccination cost $500). The implications of the
data in Table 2.1 are that there are very large differences in the cost-effectiveness of
interventions, and therefore that more lives could be saved if resources were shifted
away from those with a high cost per life-year figure.
This need to use the resources that we have efficiently and to maximum public
benefit can be recognized throughout society. It is frequently, though with some
notable exceptions, encountered in legislation, even legislation aimed at protect-
ing health and the environment, as there is usually some sort of requirement to
allocate resources efficiently. In Activity 1.3 you came across the word ‘appropriate’
in the context of environmental health policy principles. Here we are touching on
one aspect of the word ‘appropriate’ – it refers to interventions which inter alia are
cost-effective. Where there is not a requirement to allocate resources efficiently,
space is usually provided for policy makers to introduce such considerations. The
World Health Organization (WHO), for example, when it recommended air quality
guidelines as a basis for protecting human health from adverse effects of air pollu-
tion, stated that their purpose was to provide guidance to governments in making
risk management decisions. The WHO went on to emphasize that the guidelines
should not be regarded as mandatory, but should be considered in the context of
prevailing exposure levels, and social, economic and cultural conditions (WHO
1987). In other words, the priority to be given to the achievement of the guidelines
was seen as being subject to other local needs, which might be more important,
including efficiency.
The need to use risk-based approaches in public policy decision making can be
observed implicitly or explicitly in a wide variety of legislation, and in the pro-
cedures of international agencies, such as the International Atomic Energy Agency
(IAEA) and conventions such as SOLAS (Safety of Life at Sea).
Risk-based requirements are now mandatory in many countries. In the UK, the
statute governing health and safety in the workplace (the 1974 Health and Safety at
Work Act) adopts a risk-based approach, as does the more recent Management of
Health and Safety at Work Regulations 1999. Likewise, in Europe, the Workplace
The emergence of risk assessment 21
Safety Directive, introduced in 1993, has regulations covering health and safety
management which specifically require a risk-based approach. Requirements of
this kind can be found in many countries.
The health sector itself is also a prime candidate for such methodologies. As Arnold
Relman, former editor of the New England Journal of Medicine, argued in 1988, in
order to properly organize health care in the future, it will be necessary
to know much more about the relative costs, safety, and effectiveness of all the things
physicians do or employ in the diagnosis, treatment, and prevention of disease. Armed with
these facts, physicians will be in a much stronger position to advise their patients and
determine the use of medical resources, payers will be better able to decide what to pay
for, and the public will have a better understanding of what is available and what they want.
ActivityCan
2.5
you think of any statutes, requirements or advice applying in your region that
incorporate the need for a risk-based approach? What are they?
Feedback
The requirement for a risk-based approach may not be explicit. Nonetheless, if you
look at legislation, codes of practice or advice in any of the fields of occupational health
and safety, consumer protection, food safety, transport safety or environmental health,
you might well find the use of words that imply that while controls are expected, they
are only required if it is ‘reasonable’, ‘practicable’, ‘cost-effective’, etc. to apply them.
Implicit in the use of such words is that someone has assessed the risk reduction
benefit of an intervention and compared it with the cost to see if it is reasonable.
Sometimes there are absolute requirements which say that an intervention must be
made irrespective of cost, but this is unusual.
Given the history outlined above, and the rapidly changing times in which we live, it
should not be surprising that contemporary ways of thinking about and coping with
risks are different from earlier times. In the last century alone, major changes have
occurred in the nature and perception of risks. The following edited extract from
Vince Covello and Jeryl Mumpower (1985) lists nine changes which they observed.
Though based on US experience, you may find some parallels with your own coun-
try. You may well find some differences too, which should also be of interest.
ActivityAs2.6you read the following edited version of the changes in thinking about risk identified
by Covello and Mumpower (1985), make notes on whether the changes identified are
also apparent, or not, within your own country. If some factors do not apply, suggest
reasons why that might be so.
22 Introduction
Feedback
These changes cannot be expected to have been felt with the same force in all coun-
tries. The changes are typical of a high income, technological society. For example,
life expectancy and the displacement of infectious diseases by degenerative diseases can
be expected to be partially correlated with income level and state of development.
Likewise, the number of regulations to control health risks and the techniques used can
be expected to vary from country to country, though, as noted earlier, the use and
promotion of these techniques by international agencies will mean that to some extent
their presence can be expected, though it may not as yet have diffused far beyond the
central authorities. Regarding the degree to which the public is interested and expects
to participate in environmental decisions, this is more complicated still. One of the
problems of English-speaking countries has been an apparent loss of trust in govern-
ments and expertise. Loss of trust drives the level of public concern and the demand for
participation in decision making. The extent of this is country-specific.
Summary
Risk assessment is now seen as an important tool for bringing about effective and
efficient environmental health policies. In this chapter you looked at the history of
risk assessment and noted a number of factors which slowed its initial develop-
ment, and also at influences which are bringing about present-day change in the
way society deals with risk. The strength of these influences, and hence the need to
accommodate them, will vary from place to place and this should be kept in mind
when thinking about policy development.
References
Cabinet Office (2002) Risk: improving government’s capability to handle risk and uncertainty.
http://www.strategy.gov.uk/work_areas/risk/index.asp
Commonwealth of Australia (2002) Environmental Health Risk Assessment: Guidelines for Assess-
ing Human Health Risks from Environmental Hazards. Canberra: HMSO.
Covello VT and Mumpower J (1985) Risk analysis and risk management: an historical perspec-
tive. Risk Analysis 5(2): 103–20.
David FN (1962) Games, Gods and Gambling. London: Griffin & Co.
Green J (1997) Risk and Misfortune: The Social Construction of Accidents. London: UCL Press.
Grier B (1980) One Thousand Years of Mathematical Psychology. Madison, WI: Society for
Mathematical Psychology.
Paustenbach DJ (1989) The Risk Assessment of Environmental and Human Health Hazards. New
York: John Wiley and Sons, Ltd.
Relman AS (1988) Assessment and accountability: the third revolution in medical care. New
England Journal of Medicine 319: 1220–2.
Tengs T et al. (1995) Five hundred life saving interventions and their cost effectiveness, Risk
Analysis 15(3): 369–90.
World Health Organization (1987) Air Quality Guidelines for Europe. Copenhagen: WHO
European Office.
SECTION 2
Assessing
environmental
health risks
A model for human health
3 risk assessment
Overview
In the previous chapter you saw how risk assessment had moved centre stage by the
twenty-first century as a tool for the management of environmental health risks
and the formulation of health policy. This chapter will introduce the classic quanti-
tative risk assessment model in the field of environmental health, and give an
outline description of each component.
Learning objectives
Key terms
Risk assessment The activity of estimating the potential impact of a chemical, biological,
physical or other hazard upon a target individual, group or population.
Risk management The use of risk assessment in combination with socio-economic and
political inputs to evaluate and select measures to manage risk.
In 1983, the US National Research Council (NRC 1983) published an early and
influential book (referred to as ‘the Red Book’) on the nature of risk assessment as
applied to public health. It opened by making an important distinction between
risk assessment and risk management. Risk assessment was seen as referring to the use
of factual information to define the health effects resulting from the exposure of
individual persons or populations to hazardous agents or situations. Risk manage-
ment was seen as the process of weighing policy alternatives and selecting the most
appropriate action. That is, risk management required the integration of risk
assessment data with social, economic, and political concerns prior to any decision
being made. You will notice some similarity to the World Health Organization’s
position on air quality guidelines as described in Chapter 2. The WHO also saw
the need for risk management decisions to reflect local priorities in the form of
economic, social and political circumstances.
There is good reason for this. Suppose, for example, a quantitative risk assessment
of discharges from a factory indicated that they might result in some health detri-
ment to the local population. While in the ideal world this hazard would surely be
eliminated, the reality might be that the factory provides employment for the
community, and to control its emissions would undermine its viability. Thus there
is a trade-off to be made between environmental health and other outcomes related
to economic welfare, which in turn have implications for employment, housing,
diet and health itself. Thus, risk management decisions often have to take account
of other issues, some of which may fall outside the technical arena.
You may conclude that risk assessment alone, while it can well be a prerequisite for
policy formulation, is not sufficient on its own. In recent times this separation of
risk assessment and risk management, as you will see in Chapter 17, has been
somewhat eroded, but for the time being we will use it as a working model. This
separation is shown in Figure 3.1.
ActivityWhat
3.1
do you think are the advantages of separating risk assessment from risk
management?
Figure 3.1 The relationship between risk assessment and risk management
Source: Ball (2000)
30 Assessing environmental health risks
Feedback
The risk assessment stage is primarily about technical information and analysis and is
the area of people skilled in those matters, for example, epidemiologists, toxicologists,
physicians and risk analysts. The information used is usually perceived as quantitative
and objective. In contrast, policy formulation and decision making require many other
inputs, some of which are far more qualitative and subjective. By separating the two
functions, the technical experts are able to fulfil their role, which is often difficult
enough, without the added complexity of the value judgements that are present in
decision making. Furthermore, potentially important socio-economic issues do not get
subsumed into the technical process where, conceivably, they might be lost.
Note that according to this model, human health risk assessment is a predominantly
technical matter. You will return to this later.
The term ‘risk’ has been used by numerous different professions as well as by the
public and has come to mean many different things (Althaus 2005). Although
attempts have been made to encourage standardized definitions, it is unlikely that
a common usage will be achieved within the near future. Consequently, users of
this term will need to be clear in their own minds and to make clear to others the
particular definition they are using. You will also find that the meaning which you
yourself attach to this term will change depending upon the circumstances.
According to common usage, ‘risk’ is more or less synonymous with ‘danger’,
though danger itself is an ill-defined term, like ‘safety’, and best avoided if possible.
In environmental health protection, the definition of risk most often used follows
from that used in engineering and allied fields, where it refers to the probability of
some specified form of harm being realized. For example, the probability of an
earthquake of specified magnitude occurring at a particular location during, say,
the next year, or of a chemical release of a certain size from a manufacturing plant
during a specified time period. It could also refer to the probability that cancer will
occur in a given population in a specified time interval following exposure to a
chemical.
Probability itself is a mathematical expression of the likelihood of a chance event
and can be expressed in a number of different ways, for example, as an annual or
lifetime risk of dying, of contracting some disease, or of suffering an injury as a
result, say, of an accident. It could also refer to the risk of a one-off event, for
example, a tsunami. Table 3.1 lists some health risks, expressed as annual risks,
which pertained a few years ago in the United Kingdom (BMA 1987). Alter-
natively, as an example of a lifetime risk, the WHO says that the lifetime risk of
cancer from exposure to hexavalent chromium is estimated to be 4 × 10−2 (i.e. 4 per
cent) for continuous exposure to an air concentration of 1 µg per m3 (WHO
1987).
A model for human health risk assessment 31
Table 3.1 The risks of an individual dying from specified causes in any one year, UK data 1980s
Cause Risk
Smoking 10 cigarettes a day one in 200
All natural causes, age 40 one in 850
Any kind of violence or poisoning one in 3300
Influenza one in 5000
Accident on the road one in 8000
Leukaemia one in 12 500
Playing soccer one in 25 000
Accident at home one in 26 000
Accident at work one in 43 500
Working with radiation in nuclear industry one in 57 000
Homicide one in 100 000
Accident on railway one in 500 000
Hit by lightning one in 10 000 000
Release of radiation from nuclear facility one in 10 000 000
Source: British Medical Association (1987)
Other terms that you will encounter include hazard. Hazard is usually defined as
the property of a substance, or a situation, that could lead to harm. Arsenic in
drinking water could constitute a hazard, as could an unprotected well shaft.
It is worthwhile being aware that the term risk could have strikingly different
meanings for other users. In the world of banking, for example, ‘risk’ is differenti-
ated from ‘uncertainty’ and is sometimes used to indicate that it is possible to make
a precise estimate of the probability of an outcome. In the field of economic
appraisal, ‘risk’ could mean the possibility of more than one outcome occurring
(RCEP 1998).
It is necessary to be familiar with other uses of the word ‘risk’. Here we will consider
‘individual risk’ and ‘population risk’, though other kinds will be examined later.
Individual risks, as the name implies, are those risks which apply to an hypothetical
individual with either the characteristics of a typical person, or of a highly exposed
or particularly susceptible person, depending on the purpose. Individual risk is
most often described in terms of the risk per year or per lifetime of some specified
event, such as contracting a particular disease.
Population risk usually relates to the number of adverse health effects (e.g. fatal-
ities, cancers, injuries, etc.) expected in a population over a specified time period, or
the rate of adverse effects for a given location or group of people (Covello and
Merkhofer 1993).
32 Assessing environmental health risks
As described in the Red Book, and adopted in similar form by other agencies
worldwide, health risk assessment can be divided into four major stages (Figure
3.2):
• hazard identification
• dose–response assessment
• exposure assessment
• risk characterization.
It is appropriate to consider these stages in some detail.
Hazard identification
Dose–response assessment
This is the process of characterizing the relationship between the dose of an agent
administered or received and the incidence of an adverse health effect in exposed
populations. The process considers important factors such as intensity of exposure,
the age distribution of those exposed, and possibly other variables that might affect
response, such as gender and lifestyle. Dose–response relationships mainly are
based on toxicological studies involving animal testing and so extrapolation from
high to low doses is often required as well as between species, say, from animals to
humans.
Exposure assessment
Risk characterization
Summary
References
Althaus CE (2005) A disciplinary perspective on the epistemological status of risk. Risk Analysis
25(3): 567–88.
Ball DJ (2000) Risks of injury: an overview, in Harries M, McLatchie G, Williams C and King J
(eds) ABC of Sports Medicine. London: BMJ Books.
British Medical Association (1987) Living with Risk. Chichester: John Wiley & Sons Ltd.
Covello VT and Merkhofer MW (1993) Risk Assessment Methods: Approaches for Assessing Health
and Environmental Risks. New York: Plenum Press.
NRC (1983) Risk Assessment in the Federal Government: Managing the Process. Washington, DC:
National Academy Press.
Royal Commission on Environmental Pollution (1998) Setting Environmental Standards.
London: The Stationery Office.
World Health Organization (1987) Air Quality Guidelines for Europe. Copenhagen: WHO.
Hazard identification
4
Overview
In Chapter 3 you saw how the four-stage model of human health risk assessment
is constructed. This chapter will describe in greater detail the first stage of that
process – hazard identification. You will be introduced to some relevant termin-
ology, and proceed to consider the two most important techniques of hazard
identification for environmental health purposes. These are toxicological studies
and epidemiology. The approaches are very different, but they complement each
other remarkably well.
Learning objectives
Key terms
Acute exposure A short exposure, typically of hours or minutes, possibly at a high level.
Carcinogen Any chemical or physical agent able to induce cancer in living organisms.
Case-control study An observational study starting with the identification of a group of cases
and controls. The level of exposure to the risk factor of interest can then be measured
(retrospectively) and compared.
Chronic exposure An exposure of long duration, months or years, and usually to a low
concentration.
Cohort study A follow-up observational study where groups of individuals are defined on the
basis of their exposure to a certain suspected risk factor for a disease.
Confounding variable (confounder) A variable that is associated with the exposure under
study and is also a risk factor for the outcome in its own right.
Cross-sectional study A study design where exposure and outcome are measured at the same
time.
36 Assessing environmental health risks
Ecological fallacy The effects measured in groups may not be applicable at the level of
individuals.
Epidemiology The study of the distribution and determinants of health states or events in
specified populations, and the application of this study to the control of health problems.
Incidence The frequency of new cases in a defined population during a specified period of time.
in vitro A laboratory test using living cells taken from an organism.
in vivo A laboratory test carried out on living organisms such as whole animals or human
volunteers.
LD50 The dose that when administered to animals in a test is lethal to 50 per cent.
Mutagen An agent that can cause genetic damage to individual cells.
Prevalence The frequency of existing cases in a defined population at a particular point in time
(point prevalence), or over a given period of time (period prevalence), as a proportion of the
total population.
Teratogen An agent which can induce congenital anomalies in a developing foetus.
Toxicology The study of the adverse effects of chemicals on living organisms.
The first stage in the classic model of risk assessment is the identification of haz-
ards. As in Table 1.1, hazards come in many forms but this model is primarily
concerned with human health hazards arising from exposure to chemical or other
agents. You will be familiar with terms like ‘hazard’ and ‘poison’ but may not be
aware of their particular meanings in specialist circles.
ActivityAs4.1you read the following edited passage by Alice Ottoboni (1991), former toxicologist
for the California State Department of Public Health, note how key terms such as
‘poison’ and ‘hazard’ are defined. You will also encounter the technical term, LD50,
widely used in the study of toxicology.
When you have read the article, you should have an understanding of the meaning of
these terms. You should also be able to reflect upon the following quote attributed to
the famous sixteenth-century Swiss physician, Paracelsus: ‘What is it that is not a
poison? All things are poison and nothing is without poison. It is the dose only that
makes a thing not a poison.’ Is Paracelsus’ quote consistent with modern terminology?
of the toxic action of chemicals. Poisons are chemicals that produce illness or death when
taken in very small quantities. Legally, a poison is defined as a chemical that has an LD50 of
50 milligrams, or less, per kilogram of body weight. An LD50 is the quantity of chemical
administered in one dose that is lethal for 50 percent of test animals within a 14-day
period. LD means lethal dose and the subscript 50 refers to the percentage of the animals
for which the dose was lethal.
Fifty mg/kg is equal to approximately three-fourths of a teaspoon for an average adult and
about one-eighth of a teaspoon for an average 2-year-old child. There are very few chemi-
cals that are lethal in such small quantities. Thus, there are not many chemicals that can be
classed as poisons, yet there are many, many chemicals that are capable of causing illness or
death. Even among the dreaded pesticides, the majority do not fall within the category of
poison. Therefore, to consider only poisons are harmful or that harmful chemicals are, of
necessity, poisons is a misleading and dangerous assumption.
LD50s are almost always related to the toxicity of chemicals; however, chemicals that are
corrosive can also kill, especially when they are ingested. Thus, a corrosive chemical, like a
toxic one, will be classed as a poison if it is lethal in doses of 50 mg/kg or less. A good
example of chemicals that are labelled as poisons because of their corrosiveness are the
well-known drain-opener products that are found in many kitchens. One might consider it
splitting hairs to distinguish between corrosiveness and toxicity as a cause of death.
Although the distinction has little significance in human terms, it is one that must be made
in a proper study of the harmful effects of chemicals.
Another distinction that should be made is the difference between toxicity and hazard. The
latter has come into common use as a synonym for the former. Actually hazard is a much
more complex concept than toxicity because it includes conditions of use. The hazard
presented by a chemical has two components: (1) the inherent ability of a chemical to do
harm by virtue of its explosiveness, flammability, corrosiveness, toxicity, and so forth; and
(2) the ease with which contact can be established between the chemical and the object of
concern . . . For example, an extremely toxic chemical, such as strychnine, when sealed in
an unopenable vial, can be handled freely by people with no chance that a poisoning will
occur. Its toxicity has not changed, but it presents no hazard because no contact can be
established between the chemical and people. Conversely, a chemical that is not highly
toxic can be very hazardous when used in a manner that makes it available for accidental
ingestion.
Feedback
Paracelsus was clearly using a different definition of ‘poison’ from that now favoured.
Nonetheless, he was a man far ahead of his time. His point here, which is valid, is that
the effects of an agent are dependent upon the dose received, something which you will
explore further in Chapter 5. The passage by Alice Ottoboni shows that commonly
used terms, like ‘poison’ and ‘hazard’, have special significance for professionals.
Hazard identification, at least in the area of environmental health, uses three basic
approaches. These are:
38 Assessing environmental health risks
Toxicological studies
Toxicology is the science that investigates the adverse systemic effects of chemicals,
that is, the effects upon the whole body. Hazard identification by toxicity testing
is mainly reliant upon in vivo testing on animals conducted according to standard
protocols. A range of tests are used, the main ones being (Commonwealth of
Australia 2002):
• acute toxicity tests. These investigate the effects of single doses of a substance
and commonly are used to identify the medium lethal dose, or LD50 over a
14-day post-dosing period.
• sub-chronic toxicity tests. These investigate the effects in the short term of
repeated doses.
• chronic toxicity tests. These are studies of the effects of long-term exposure to an
agent.
• reproductive toxicity tests. These are designed to identify the effects of test
substances on the reproductive performance of males and females.
• mutagenicity tests. These are designed to determine if test chemicals can cause
gene or chromosomal mutations that can be passed to another generation.
• carcinogenicity tests. These seek to determine the cancer-causing potential of
an agent.
Ethical considerations rule out the deliberate exposure of people to hazardous
agents except in the case of clinical trials with volunteers. As a result, toxicity data
for humans are often unavailable, other than in the aftermath of accidents such as
those at Bhopal, Seveso, and Minamata.
ActivityToxicological
4.2
testing is performed on animals (itself now raising ethical questions in
some countries). However, this has its own drawbacks. Note some of the factors that
you think might limit its usefulness.
Feedback
The main problem is uncertainties in extrapolating from animals to humans, and in
extrapolating from the typically high doses given to animals over short periods of time
in laboratory tests, and the generally much lower doses experienced by humans over
long periods in the environment. Exposure pathways may also differ.
Hazard identification 39
Human equivalent doses have traditionally been derived from animal data by the
relatively crude method of scaling the dose in proportion to body weight, although
in some cases more sophisticated methods are available to provide biologically
equivalent doses, though these methods are also prone to a range of uncertainties.
Even in situations where human data are available, as, for example, in the
unintended exposure of workers to toxic materials, extrapolation to the general
public may be problematic. This is because occupational data often apply to a
relatively healthy, mainly male workforce within a specific age-band, whereas the
general public are not restricted in this way.
Epidemiological studies
ActivityThe
4.3
following abridged passage (Commonwealth of Australia 2002) provides a brief
but valuable review of the four main categories of epidemiological study. It is important
to understand the strengths and limitations of the different methods and of epidemi-
ology in general. In particular, the ability to make links between cause and effect. As
you read it, make a note of the relative contributions to hazard identification made by
epidemiology and toxicology.
exposure. Such studies can be carried out by either reviewing past studies (retrospective)
or by tracking people into the future (prospective cohort). The essential feature of these
longitudinal studies is that for each individual prior exposure information can be related to
subsequent disease experience.
Ecological studies involve the investigation of a group of people such as those living within a
geographical area such as a region or state. For example, place and time of residence may be
used to create surrogate measures of the real exposure of interest. Rates of disease and
average exposure levels to a particular agent are determined independently, and on a group
basis. This may give rise to a spurious apparent correlation, called the ecological fallacy.
Because it is not ascertained whether individuals who have been exposed to the agent are
the same individuals who developed the disease, statements about causal relationships are
inappropriate. However, ecological studies are relatively inexpensive for linking available
health data sets and environmental information and are useful for hypothesis generation.
Examples of ecological studies are the assessments of the relationship between tobacco
sales in different countries and lung cancer rates, and fluoride in water and dental caries.
A subset of ecological studies, known as time-series studies, is regarded as very helpful in
understanding the influence of short-term fluctuations in, say, air pollutants on day-to-day
changes in population morbidity and mortality after controlling for factors such as sea-
son and air temperature. However, disentangling the effects of individual pollutants as
measured in a mixture such as urban air pollution can be difficult . . .
Epidemiological studies are rarely definitive and a single study cannot establish causality.
A ‘weight of evidence’ approach is generally required, involving the interpretation of all
available information.
Overall, epidemiological studies, depending upon their design, may serve two purposes:
hypothesis-generation or assessment of a causal relationship. Their ability to evaluate a
causal relationship may be limited by a lack of control of potential confounders (confound-
ers are factors which distort the effect of the agent of interest) or a lack of statistical
power (usually because of limited sample sizes).
Feedback
A clear advantage of epidemiological studies is that there is no need to extrapolate
between species. This would be particularly problematic if the interest were in, say,
mental functioning, or behavioural or subjective effects, but is always an issue. On
the other hand, epidemiological studies are more vulnerable to uncertainties in both
exposure and dose, which can be tightly controlled in toxicological work. Toxicology
is also necessary for confirming causal mechanisms. And when it comes to dose–
response relationships, the subject of the next chapter, epidemiological studies can
be seen to measure exposure–response rather than dose–response.
Summary
In the field of environmental health there are two principal methods of hazard
identification: toxicology and epidemiology. Toxicology is mainly reliant upon
animal testing, whereas epidemiology acquires knowledge by the study of the
occurrence of symptoms and disease in human populations. In this chapter you
Hazard identification 41
have learned about the basics of the approaches and some of their strengths
and limitations. In moving on to policy formulation, it is important to have an
awareness of the strengths and limitations of the various information inputs.
References
Overview
In Chapter 4 you saw how chemical hazards are identified. This chapter introduces
you to the next stage of the classical environmental health risk assessment model,
namely, the determination of the dose–response relationship. As before, there is a
fair amount of terminology involved but this is inevitable in dealing with a multi-
disciplinary subject such as environmental health policy. Here you will be examin-
ing how toxic agents are assessed, and in particular, the different approaches used
in assessing agents that pose a finite risk at any level of exposure, however small,
and those which are harmful only above a certain dose. As you will find out, this
distinction is of fundamental importance for the development of risk management
as applied to health policy.
Learning objectives
Key terms
Acceptable daily intake (ADI) The daily intake of a chemical which, during a lifetime, appears
to be without appreciable risk. Usually measured in mg/kg/day.
Deoxyribonucleic acid (DNA) The molecule in which the genetic blueprint for living cells is
encoded.
Genotoxic A chemical that can cause damage in genetic material leading to heritable changes.
Linear no threshold hypothesis The presumption that risk is linearly related to dose at low
doses for non-threshold agents.
Dose–response assessment 43
Lowest observed adverse effect level The lowest experimental or observed concentration or
amount of a substance that causes adverse alterations in target organisms.
Multi-hit model Dose–response models that assume that more than one interaction with a
toxic material at the molecular level is necessary to cause an effect.
No observed adverse effect level (NOAEL) The highest dose at which no significant adverse
effects are noticed in a population.
One-hit model Dose–response models that assume that a response occurs after a target has
been impacted once at the molecular level by a biologically effective dose.
Reference dose An estimate of the highest daily dosage of a risk agent that is unlikely to
produce an appreciable harmful effect in humans.
Safety factor A single factor or several factors used to derive an acceptable intake of a chemical.
Stochastic A random statistical phenomenon.
Threshold The lowest dose or exposure level which will produce a toxic effect.
Figure 5.1 Hypothetical dose–response curves for a single chemical agent. If an agent causes
more than one effect (three in this example) there will be multiple curves.
44 Assessing environmental health risks
Figure 5.2 looks at an individual dose–response curve in more detail. Note that here
the ordinate measures the magnitude of a specified effect in a typical individual.
For all doses above a certain dose (denoted X in Figure 5.2), the maximum effect is
exhibited. Below this there is a range where the magnitude of the effect depends
upon the dose received. Importantly, there is also a threshold below which there is
no effect. Thresholds are believed to exist for some agents because of the ability of
the body to metabolize or excrete a toxin, or to repair damage, up to a certain dose.
The threshold approach has been used for over half a century to identify doses at
which humans would not be expected to exhibit adverse effects. Traditional toxico-
logical procedures define a safe level of exposure for humans as some fraction of the
maximum dose level at which there is no observed effect. There are two terms that
you need to be familiar with. The lowest dose which leads to an observable effect is
known as the ‘lowest observed effect level’ (LOEL). The maximum dose without
effect is referred to as the ‘no observed adverse effect level’ (NOAEL). Note the
inclusion of the word ‘adverse’. This is because some exposures create observable
effects but these are not thought to be harmful, for example, one glass of beer.
The question now arises as to the fraction by which the NOAEL should be multi-
plied in order to define the level of dose which is ‘safe’. Bearing in mind that most
dose–response curves are based upon animal tests, it is customary to use a fraction
1
of to allow for the fact that humans might be ten times more sensitive than the
10
test animals. This interspecies factor is clearly somewhat arbitrary – it is a rule of
thumb. Normally, however, the fraction applied (often known as the ‘safety factor’)
1 1
is smaller than this; typically is used, i.e. a further factor of is introduced. This
100 10
is to allow for further uncertainty associated with the variable sensitivity within the
human population, that is, intraspecies variability, to specific agents. For example,
Dose–response assessment 45
children or people with pre-existing health problems may be more vulnerable than
the average member of the population to a certain hazardous agent. Further allow-
ances might also be made depending on the severity of the consequence of exposure,
and the quantity and quality of the available scientific data on which the assess-
ment is based. So if an agent has particularly insidious or uncertain consequences, a
further judgemental factor might be incorporated.
ActivityOne
5.1
shortcoming of the threshold and safety factor approach is that the potentially
useful information provided by the slope of the dose–response curve in the region of
the threshold is not used. In what way could this information be useful?
Feedback
A moderate safety factor may provide an adequate safety margin if the dose–response
curve is steep in the region of the threshold dose, but not if the gradient is shallow. The
choice of safety factor could have economic implications, so it is important to try and
gauge it appropriately.
1
Having decided upon the safety factor, say , the NOAEL is then multiplied by
100
this factor to yield various other related parameters which are used in specifying
‘safe’ levels:
• Acceptable daily intake (ADI). The daily intake of a chemical which, during a
lifetime, appears to be without appreciable risk. ADIs are usually expressed in
mg/kg-body weight/day but may have different units depending on the
exposure route.
• Tolerable daily intake (TDI). An estimate of the daily intake of a substance which
can occur over a lifetime without appreciable health risk. TDI is used in relation
to contaminants which should not be present, whereas ADI is used in the con-
text of contaminants that are present because they were deliberately used, such
as pesticides on food crops.
• Tolerable weekly intake (TWI).
• Reference dose (RfD). An estimate of the daily exposure (mg/kg/day) to the gen-
eral population that is likely to be without an appreciable risk of deleterious
effects during a lifetime of exposure.
Non-threshold agents
Note that the use of a safety factor to determine acceptable human exposure levels
relies upon the existence of a threshold dose below which adverse effects do not
occur. However, the existence of a threshold below which there are no effects is
not accepted for some compounds, notably, carcinogens. In the case of these
materials, it is widely assumed that however small the dose, some risk of cancer
remains.
46 Assessing environmental health risks
Although biological arguments exist, for example, that DNA, while it may be sus-
ceptible to damage by a carcinogen, may have an effective self-repair mechanism at
low doses, the threshold at which this eliminates risk is still likely to vary from one
person to another, and its value is therefore very uncertain.
Another problem is that for agents that have either a very low or no threshold,
there may be no animal test data available. Typically, dose–response curves are
based on small numbers (for cost reasons) of test animals receiving high doses in
short periods of time, but environmental health policy makers are mainly con-
cerned with the quite different situation of large numbers of people being exposed
to low doses over long periods.
ActivityFigure
5.2
5.3 shows a situation which might be faced by an environmental health policy
maker. The example is of exposure to ionizing radiation. Unusually, in this example,
human dose–response data are available, based on studies of survivors of Hiroshima
and Nagasaki, and from occupational exposures to high levels of radiation in uranium
mines. This information provides the cluster of data in the top-right corner of Figure
5.3. However, these are clearly abnormal exposures, and if your task were to assess the
risk of harm to a population exposed to much lower, background levels of radiation, as
in the lower-left corner of Figure 5.3, how would you do this?
Figure 5.3 The problem of evidence that is not available. Health effect data may exist for
high exposures but mainly the interest is in low exposures.
Feedback
In dealing with ionizing radiation and some genotoxic carcinogens, it is assumed that
there is no threshold. A non-threshold model has therefore to be adopted. Tradition-
ally, non-threshold models assume a linear relationship between the lowest
experimentally-observed dose and zero dose. This is known as the LNT (linear no
threshold) hypothesis. Note, it is an hypothesis and is not scientifically proven, although
there are scientific arguments that support this assumption. In general, scientists are
discouraged from extrapolating outside of the range of measured data, though the
environmental policy maker unfortunately has little choice in this matter.
Dose–response assessment 47
ActivityWhat
5.3
is the implication of the LNT hypothesis for the safety of humans exposed to
ionizing radiation and chemical carcinogens, whether exposure occurs in the health
service, industry, or from the natural background?
Feedback
The implication is far-reaching. It is that there is no such thing as a safe dose if ‘safe’ is
taken as meaning zero risk. This means that policy makers have somehow to define
levels of risk for non-threshold agents which are non-zero but somehow ‘tolerable’.
This is a major issue that you will encounter as you progress through this book.
Another important and debated issue involving the LNT hypothesis is the assumed
linear relationship between dose and effect at doses outside the range for which
data exist, either from animal tests or epidemiological studies. Figure 5.4 summar-
izes the situation. Data points are shown, but what does the curve really look like at
low doses? Is it straight (b); supra-linear (a); sub-linear (c); or threshold (d)?
ActivityThe
5.4
implications for policy makers of the above uncertainties of low-dose extrapola-
tion models are discussed in the following abridged extracts which have been collated
by Paustenbach (1989). As you read them, familiarize yourself with the dilemmas faced
by scientists who are trying to provide useful information on which environmental
policy makers may base their decisions.
48 Assessing environmental health risks
Feedback
The shape of the dose–response curve at low doses is uncertain. There is no universally
accepted model for the shape at low doses and scientific judgement is needed in each
case. Where there is little information to go on, the LNT hypothesis is normally
preferred, but may over-state risk in some cases.
ActivityIn5.5reading the following guidance from the WHO (1987) regarding exposures to nitro-
gen dioxide and to nickel in the atmosphere, think about the forms of the two pieces of
guidance and consider why they are different.
Exposure guidelines
3 3
Nitrogen dioxide levels of 400 g/m and 150 g/m are recommended as 1-hour and 24-hour
guidelines respectively. The 1-hour guideline is based on the judgement that the lowest-
observed-effect-level (LOEL) in asthmatics (560 g/m3) is not necessarily adverse and a
guideline somewhat lower provides a further margin of protection. The 24-hour guideline
is based on the judgement that repeated exposures approaching the minimum repetitively
observed effect level are to be avoided, so as to create a margin of protection against
chronic effects.
At an air nickel dust concentration of 1 g/m3, a conservative estimate of lifetime risk is
4 × 10−4.
Feedback
The WHO has taken nitrogen dioxide to be a compound without carcinogenic effects,
in which case the guideline was derived from knowledge of the LOAL, LOAEL or
NOAEL and application of a protection (safety) factor. For nickel, this has been put
through the WHO’s two-step procedure which, first, assesses whether the compound
is a proven human carcinogen (a group 1 compound); a probable human carcinogen
(a group 2a compound has limited evidence of human carcinogenicity and sufficient
evidence of animal carcinogenicity; group 2b has inadequate evidence of human
carcinogenicity and sufficient evidence of animal carcinogenicity); or unclassified
(group 3). Second, having identified it as a potential human carcinogen, epidemiological
and toxicological data have been used to identify a risk factor since no absolutely safe
level can be defined. The risk factor, which is expressed as the lifetime risk of developing
cancer as a result of continuous exposure at the specified level, should be seen as a
rough estimate and not the true cancer risk. It serves as a basis for policy makers in
balancing risks and benefits and establishing the degree of urgency of the associated
public health problem.
50 Assessing environmental health risks
ActivityFor5.6the policy maker there are important issues embedded in the choice of a threshold
or non-threshold approach for controlling exposures. In reading the following extract
from the Commonwealth of Australia (2002), make a note of the answers to these
questions:
1 What are the advantages of the threshold approach?
2 What are the advantages of the non-threshold approach?
3 Why should both approaches be treated with caution?
Non-threshold models currently in use are inflexible and generally do not take account of
the complexities of the events between exposure to an agent and the induction of a
neoplasm. Risks estimated at doses below the range of experimental data can vary con-
siderably depending on the model used, even though the various mathematical models used
generally fit the experimental data equally well. The numerical estimate of the level of risk
falsely gives the impression that it represents an exact measure of actual risk. The numer-
ical expression provides little or no information on the uncertainties related to the esti-
mated level of risk, nor does it allow comparison with values for non-cancer health effects.
Feedback
1 Advantages are the simplicity of the approach and the gradually increasing evidence
of validity. The approach also allows a consistent approach to the specification of ADIs.
2 Risks at low doses can be estimated and this may help policy makers. There is also
some evidence of validity of estimates, and some that they are conservative, i.e. more
protective than the model suggests.
3 In toxicity tests on animals, the choice of experimental design parameters influences
the NOAEL and hence the ADI, PTWI or RfD, and safety factors remain rather arbi-
trary. For both threshold and non-threshold models, the uncertainties associated with
the predictions are potentially quite large and the numbers quoted should not be taken
as strict safety levels or as accurate risk factors.
Summary
References
California Department of Health Services (1986) Guidelines for the Assessment of Carcinogenic
Substances. Sacramento, CA: CDHS.
Commonwealth of Australia (2002) Environmental Health Risk Assessment: Guidelines for Assess-
ing Human Health Risks from Environmental Hazards. Canberra: Commonwealth of
Australia.
Munro IC and Krewski DR (1981) Risk assessment and regulatory decision-making. Food
Cosmet. Toxicol. 19: 549–60.
Office of Science and Technology Policy (1985) Chemical carcinogens: a review of the science
and its associated principles. Federal Register 50(50): 10372–442.
Paustenbach DJ (1989) The Risk Assessment of Environmental Hazards. New York: John Wiley
& Sons, Ltd.
World Health Organization (1987) Air Quality Guidelines for Europe. Copenhagen: WHO.
Exposure assessment
6
Overview
The next stage in the process of quantitative risk assessment is exposure assessment.
Exposure assessment is just as important as hazard identification and dose–response
assessment. This is logical because if there is no exposure to a hazard, then its
existence is unimportant. Here you will learn about the different routes through
which exposure may occur and how to quantify them.
Learning objectives
Key terms
Environmental medium A specific part of the environment such as air, water or soil,
sometimes referred to as an environmental compartment.
Exposure route How an agent enters the body (such as by inhalation, ingestion, contact).
ingestion (e.g. drinking water contaminated with arsenic, or consuming food with
pesticide residues), inhalation (e.g. inhaling fumes from cooking stoves), and
dermal contact (e.g. handling chemicals or contaminated materials).
The most accurate way of assessing exposure is by measuring the concentration of
the agent of concern in relation to the presence and activities of affected persons.
Measurement, however, is often expensive and may be impractical. It is therefore
common to rely upon mathematical models that estimate concentrations to which
persons are exposed. Whichever method is used, it is still necessary to approximate
the dose.
ActivityIn6.1their book, Risk Assessment Methods: Approaches for Assessing Health and Environmental
Risks (1993) Vincent Covello and Miley Merkhofer describe difficulties posed in the
assessment of exposures. As you read the following abridged extract, consider the
following questions:
1 Why is human exposure assessment difficult?
2 Why is it important to find out who is exposed?
Exposure assessment
For many risk assessments, exposure assessment is the most difficult task. The reason for
this is that exposure assessment often depends on factors that are hard to estimate and
for which there are few data. Critical information on the conditions of exposure is often
lacking; for example, although industries are generally required to keep records on work-
ers’ exposures to toxic chemicals, the levels of exposure and the particular chemicals
involved are often not known or not recorded. Exposures to the general population are
even less well documented due to the limited availability of systems capable of carrying
out such measurements.
A major source of complexity in exposure assessment is the strong influence that indi-
vidual personal habits can have on human exposure. In the case of food contaminated
with risk agents, food storage practices, food preparation, and dietary habits have a major
influence on the amount of the risk agent actually consumed. For example, if raw meats
have been contaminated with pathogenic micro-organisms, a critical factor in predicting
health consequences is whether and to what extent the meat is cooked. As another
example, to estimate consumer exposure to a pesticide requires data on pesticide resi-
dues, data on how many pesticide-treated food products are consumed by an average
person, and data on how many people consume unusual amounts of the pesticide-treated
food products.
If the risk agent is encountered through the use of a consumer product, the patterns of use
will affect exposure. A solvent whose vapour is potentially toxic, for example, may be used
outdoors or in a confined, poorly ventilated living space. Thus, for consumer products, risk
assessments need to consider how products will be used.
If exposure occurs through air or water, exposure assessment must consider how the risk
agent moves from its source and if it is altered over time. Chemical agents are generally
54 Assessing environmental health risks
diluted in the environment and may degrade after release. The aim of exposure assessment
in this case is to determine the concentration of toxic materials where they interface with
target populations.
Another important aspect of exposure assessment is determining which groups in the
population may be exposed to a risk agent; some subgroups may be especially susceptible
to adverse health effects. These include pregnant women, very young and very old people,
and people with impaired health.
Exposure to multiple risk agents often results in portions of the population becoming
more sensitive to single agents. Exposure to risk agents that act synergistically greatly
complicates risk assessment. Exposure to both cigarette smoke and asbestos results in a
rate of cancer incidence much greater than that indicated by the dose–response data for
the individual substances. Synergisms also often necessitate that exposure assessment
consider the activities in which the exposed individuals engage. Strenuous activity often
increases the uptake of risk agents. Multiple sources also complicate exposure assess-
ments. An individual can be exposed to a single risk agent from several distinct sources.
Exposure to lead, for example, can come from breathing air, eating food, and drinking water.
Feedback
1 There is a general lack of exposure data and it is also hard to gather. People are
constantly moving around and have widely differing habits. When it comes to exposure
assessment, it is frequently the case that ‘the devil is in the detail’.
2 For a specific agent, different subgroups may be more or less vulnerable depending
upon their personal habits, needs, lifestyle, gender and state of development.
The primary routes of exposure to environmental risk agents for human beings are
through the inhalation of gases, vapours and dusts; through ingestion of foods,
water, or unintentionally of other materials including dust and soil; and via skin
contact.
ActivityPolychlorinated
6.2
biphenyls (PCBs) constitute a class of 209 man-made, closely related
organic chemicals. Previously manufactured in many industrial countries for use in elec-
trical equipment, PCBs are now strictly controlled. A small number have been found to
have toxicological effects, but the main concern is that PCBs have been found to have
spread throughout the environment where, due to their slow rate of degradation, they
are very persistent.
Think about the pathways by which PCBs could spread through the environment to
human beings. Then compare them with the feedback below.
Exposure assessment 55
Feedback
Figure 6.1 illustrates how chemical agents may spread through the environment from a
source and come into contact with people. PCBs in particular can escape from manu-
facturing plant in the liquid or vapour phase. They also leak from electrical devices
during use and following disposal, unless incinerated under carefully controlled condi-
tions. They are soluble in lipid-rich (fatty) tissues and secretions including mothers’
milk. PCBs wash into water courses and the oceans, bioaccumulating, because of their
fat solubility, in fish, thus entering the food chain. Other routes, affecting land animals,
are also active, such as deposition on grass and contamination of animal feed. Today
the main source for humans is the consumption of fatty foods such as meat, fish, eggs,
milk and milk products, including human milk.
Figure 6.1 Chemical agents may spread from a factory via atmospheric emissions, spillages
which contaminate runoff and ground water, tracking out on vehicle tyres, or via shoes and
personal clothing. Resulting exposures can be through ingestion, inhalation, and dermal
contact with contaminated dusts deposited on surfaces or soil.
ActivityAt6.3one time tetraethyl lead was widely used as a petrol additive for the purpose of
improving the fuel economy of cars. This, however, resulted in contamination of the
atmosphere by lead, and especially for people living in towns and cities, their blood–lead
concentrations were increased following inhalation of lead particles. This resulted in
health concerns, because lead exposure is linked with effects on the central nervous
system, especially in children. Consequently, many countries have now taken measures
to reduce or eliminate the use of tetraethyl lead.
Draw up a list of routes by which people may be exposed to lead emissions from car
exhausts.
56 Assessing environmental health risks
Feedback
Figure 6.2 shows how lead not only enters the body through the lungs but via other
media. Pathways include deposition on leafy foodstuffs, deposition on soil and uptake by
root crops, and ingestion of trace amounts of contaminated soil or deposited dust via
the hand-to-mouth route.
Figure 6.2 Pathways by which lead emissions from vehicle exhausts may contribute to the
concentration of lead in the blood of exposed persons
Figure 6.3 provides an overview of the methods available for assessing exposure,
both direct and indirect.
Direct monitoring
In this approach individuals carry personal devices which monitor the environ-
ment around them. Doctors and nurses who work in a radiodiagnostic or radio-
therapeutic capacity in hospitals often wear film badges that record their exposure
to ionizing radiation. These badges can be checked periodically to assess exposure.
The approach has been extended in recent years through the invention of small,
portable devices, known as personal samplers. These can measure a range of
atmospheric pollutants.
Figure 6.3 Approaches to exposure assessment
Source: Adapted from National Academy of Sciences (1991)
58 Assessing environmental health risks
Indirect monitoring
In this approach it is customary to rely upon information from any national, urban
or rural monitoring which has been carried out. A good number of countries now
have permanent monitoring systems in place which measure air and water quality
on a continuous basis at a network of fixed monitoring sites.
Use of biomarkers
By inference
In many situations, monitoring data will not exist. In these cases published data
from situations which reflect as closely as possible the circumstances of concern
may have to suffice.
By modelling
Table 6.1 Exposure to some substances can be detected by biological monitoring of exposed
persons
Substance Where monitored Comments
Lead Blood, tooth, hair Substantial data are available on the
level of risk associated with blood lead
ranges. Tooth and hair concentrations
measure longer-term, chronic,
exposure
Arsenic Urine The retention time is short and
measurements must be made close to
the time of exposure
Mercury Blood, urine At equilibrium the concentration of
mercury in blood reflects daily intake
and is probably the best measure
Cadmium Urine, blood Urinary levels tend to reflect body
burden whereas blood levels reflect
recent exposure
PCBs Blood and adipose (fat) Because of the long retention time in
tissue fat, long-term exposures are
measured
Organochlorine pesticides Blood and adipose tissue Because of the long retention time in
(aldrin, dieldrin, chlordane, fat, long-term exposures are
heptachlor, etc.) measured
Organophosphorus Blood By measuring cholinesterase levels
compounds (malathion,
chlorpyrifos, etc.)
Source: Based on Langley (1991)
Figure 6.4 Atmospheric dispersion models are simple in principle. They estimate how much a
plume of contaminant spreads out and dilutes before it reaches the population.
60 Assessing environmental health risks
Table 6.2 Types of model, of varying complexity, available for exposure assessment
Type of model Typical inputs Typical outputs
Atmospheric dispersion Emission rate of specified Atmospheric concentrations
pollutants; emission characteristics at locations of interest
(e.g. height above ground);
meteorological data; land
topography
Surface-water models Emission rates from local or Average concentration as a
dispersed sources; flow function of distance from
characteristics of receiving waters; source
properties of the contaminant (e.g.
solubility)
Ground-water models Infiltration rates; local geology; flow Concentration of pollutants
pattern; chemistry in water used for irrigation
and consumption
Food chain models Input rates; empirically derived Concentration in food
bioaccumulation rates for relevant products for human
organisms consumption
Table 6.3 Examples of typical measurement units for describing pollutant concentrations
Medium Measure Typical units of measurement
Air Weight of pollutant per unit volume or g/m3 (micrograms per cubic metre)
mass of air ppm or ppb (parts per million or parts
per billion by weight)
Air Volume of pollutant per unit volume of ppm or ppb (parts per million or billion
air by volume)
Water Weight of pollutant per unit volume or mg/l (milligrams per litre)
mass of water ppm (parts per million by weight)
Soil Weight of pollutant per unit weight of mg/kg (milligrams per kilogram)
soil ppm (parts per million by weight)
Food Weight of contaminant per unit weight mg/kg (milligrams per kilogram)
of food ppm (parts per million by weight)
Feedback
You would need to consider all or some of the following: the medium through which
the pollutant is being transmitted (air, surface water, ground water, etc.); what the
characteristics of the source of the pollutant are (continuous or intermittent releases;
well-defined point location or dispersed source); what the geographic scale of the
problem is (local, regional, national or global); the nature of the vulnerable receptors
Exposure assessment 61
Dose estimation
Feedback
Respiratory rate is affected by age, gender and level of activity. Table 6.4 gives some
typical values, measured in litres of air per minute.
In the case of exposure resulting from ingestion of food, liquids, dust or soil, it is
necessary to have average consumption data. Food consumption rates have been
measured in many countries and standard values have been adopted for some, but
clearly these will vary substantially depending upon cultural preferences, avail-
ability and income.
In the case of water consumption, typical values are often taken to be 2L/day for
adults (70 kg) and 1L/day for children (10 kg). In areas where the ground is con-
taminated, the primary intake mechanism is often the direct ingestion of
contaminated soil or, if atmospheric deposition is the mechanism giving rise to
contamination, dust. The view is that pre-school children are most vulnerable to
this exposure route. Paustenbach (1989) has reviewed soil uptake studies and has
concluded that the amount of soil uptake by a typical child (aged 2 to 6 years) is in
the region of 50 or 100 mg/day.
Dermal absorption through contact with contaminated media is harder to esti-
mate. This is influenced by contaminant concentration, the nature of the con-
taminated medium (solid, liquid, vapour or gas), area of skin exposed, exposure
duration, and the dermal absorption coefficient. Specific details of exposure cir-
cumstances would be required before intake by this mechanism could be
estimated.
ActivityEstimate
6.6
the total uptake in g/day of lead for adults and children (1–5 years) from
simultaneous exposure to lead in air, food and water. Make the following assumptions:
the mean lead in air concentration is 2 g/m3; adults inhale 20 m3/day and children 5 m3/
Exposure assessment 63
day of air; the respiratory absorption rate for adults and children is 40 per cent; adults
have an intake of 100 g/day of lead from food with an absorption rate of 10 per cent;
children have a lead intake from food of 50 /day with an absorption rate of 50 per cent;
the water concentration is 25 g/litre; adults consume 1 litre/day of water with an
absorption rate of 10 per cent; children consume 0.5 litre/day with an absorption rate
of 50 per cent.
Feedback
For adults, uptake is given by:
(2 g/day × 20 m3/day × 0.4) + (100 × 0.10) + (25 g/l × 1 l/day × 0.10) = 28.5 g/day
For children, uptake is given by:
(2 g/day × 5 m3/day × 0.4) + (50 × 0.50) + (25 g/l × 0.5 l/day × 0.50) = 35.25 g/day
ActivityRead
6.7
the following passage from Rodricks (1994) and then, using the adult and child
uptake rates for lead calculated above, and assuming adults weigh 70 kg and the children
weigh 15 kg, calculate and comment upon the doses of lead received by adults and
children.
Calculating dose
Everyone is generally familiar with the term dose, or dosage, as it is used to describe the
use of medicines. A single tablet of regular strength aspirin typically contains 325 milligrams
of the drug. An adult takes four tablets in one day, by mouth. The total weight of aspirin
ingested on that day is 1300 mg, or 1.3 grams. But weight is not dose. For reasons relating
to how aspirin affects the body biologically, i.e., how it relieves pain, the critical measure
is the amount taken into the body divided by the weight of the person, expressed in
kilograms. The aspirin dose for a 65 kg adult is thus 1300 mg/65 kg = 20 mg/kg body weight
(b.w.). The time over which the drug was taken is also important in judging its effectiveness.
Our adult took four tablets in one day, and the day is the usual time of interest. So a more
complete description of the aspirin dose in this case is given by 20 mg/kg b.w./day. The
typical dose units are thus milligrams of chemical per kilogram of body weight per day.
Note – and this is quite important – that if a 20 kg child were to take the same four tablets
on one day, his dose would be more than three times that of the adult, as follows: 1300 mg
aspirin/20 kg b.w. = 65 mg/kg b.w./day. For the same intake of aspirin, the lighter person
receives the greater dose.
Feedback
The dose of lead received by the adults is (28.5 g/day)/70 kg = 0.41 g/kg b.w./day, and by
the children is (35.25 g/day)/15 kg = 2.4 g/kg b.w./day. The implication is that the
64 Assessing environmental health risks
children receive a dose six times higher than the adults. However, one needs to be
careful about the nature of the effects being considered. In the case of sub-acute lead
exposure, the primary concern in children is cumulative effects on the central nervous
system. In this case, the important parameter may be the long-term average daily
uptake.
Feedback
You need to know that typically adults consume 2 L/day and children 1 L/day of
water unless you have more specific information. Also assume, in the absence of
specific information, that the adults weigh 70 kg and the children 15 kg. For adults,
the daily dose will be: ((1 g/L) × 2)/70 = 0.00003 mg/kg b.w./day, and for children it
will be: ((1 g/L) × 1)/15 = 0.000067 mg/kg b.w./day. In this case, the children are
getting roughly twice the dose of the adults despite consuming half the quantity of
water.
Environmental health advisers and risk assessors may wish to compare an indi-
vidual’s uptake of environmental chemicals with doses that have been linked to
adverse effects in humans. This is often achieved by calculating the maximum daily
dose (MDD) or the lifetime average daily dose (LADD) resulting from the exposure,
usually in units of mg/kg b.w./day. For chemicals with acute effects, the MDD is
usually compared with the NOAEL from short-term animal studies, and for chem-
icals with carcinogenic potential in long-term bioassays, the LADD is generally
compared with the NOAEL identified in long-term tests (Paustenbach 1989). The
ratio of the NOAEL to MDD or LADD is then referred to as the margin of safety.
To complete this chapter you should learn something of the fate of chemicals
entering the human body. As you have already seen, chemicals may be inhaled
through the nose and mouth into the lungs; they may be ingested and swallowed,
thus entering the gastrointestinal tract; and they may contact the skin and be
adsorbed or absorbed.
Exposure assessment 65
As soon as chemicals contact the body, they may damage tissues and cells. They
may also enter the bloodstream where they are rapidly circulated around the body,
possibly to be transformed (metabolized) by reactions with enzymes, large protein
molecules in the cells. The chemicals, or their metabolites, are then excreted in
urine, faeces, sweat, and exhaled air. Figure 6.5 illustrates how chemicals may be
absorbed, distributed and excreted from the body.
Figure 6.5 Schematic diagram showing how chemicals enter and pass through the body, or are
stored
Source: Rodricks (1992)
66 Assessing environmental health risks
Summary
References
Covello VT and Merkhofer MW (1993) Risk Assessment Methods: Approaches for Assessing Health
and Environmental Risks. New York: Plenum Press.
Langley AJ (1991) Protecting the public: biological monitoring, in El Saadi O and Langley
AJ (eds) The Health Risk Assessment and Management of Contaminated Sites. Adelaide: South
Australia Health Commission.
National Academy of Sciences (1991) Human Exposure Assessment for Airborne Pollutants.
Washington, DC: National Academy Press.
Paustenbach DJ (1989) The Risk Assessment of Environmental Hazards. New York: John Wiley
& Sons, Ltd.
Rodricks JV (1994) Calculated Risks: The Toxicity and Human Health Risks of Chemicals in Our
Environment. Cambridge: Cambridge University Press.
Snyder WS (1975) Report of the Task Group on Reference Man. International Commission on
Radiological Protection Publication No. 23. New York: Pergamon.
World Health Organization (1987) Air Quality Guidelines for Europe. Copenhagen: WHO.
Risk characterization
7
Overview
Learning objectives
Key terms
Conservatism Assuming the worst case in order to err on the side of safety.
Hazard index (HI) The quotient of the maximum daily dose and the acceptable daily intake of
a hazardous material.
Unit cancer risk (UCR) An estimate of the excess probability of cancer per unit dose.
Characterizing risks
Describing risk
Even when describing risks quantitatively, it must be recognized that doing this
with accuracy is seldom feasible because of variability in the hazardous agent and
population, and limitations in the toxicological and exposure data. For example, it
may be necessary to extrapolate over several orders of magnitude when establish-
ing human health risk factors that are based on animal experiments. A common
criticism of quantitative environmental health risk assessments has been that the
numbers give a false sense of accuracy and, unless policy makers are warned, may
be inadvertently misused.
ActivityRead
7.1
the following excerpt from Paustenbach (1989) and consider why it is important
to make assumptions and uncertainties clear when characterizing risks.
Feedback
The news cutting described by Paustenbach, although intended to inform, could clearly
be misinterpreted. A difficulty is that news editors generally feel it necessary to present
information to their readers in a simple form. The same problem can arise when risk
assessors communicate with policy makers. Indeed, policy makers and especially politi-
cians sometimes refuse to accept that uncertainty exists and demand that risk assessors
should ‘provide unambiguous answers’. However, this is an error – the nature of scientific
investigation is being misunderstood and an important policy issue is being sidelined.
ActivityWhat
7.2
creates uncertainty in risk assessments? From what you have already read about
the various stages of risk assessment, make a list of some of the sources that should be
taken into account.
Feedback
One can identify a surprising range of uncertainties associated with each stage of the
environmental risk assessment process. The following account lists some, though not all.
Dose–response stage
• Which dose–response model should be used to extrapolate from high to low doses?
• For what health effects should dose–response relationships be sought?
• What factors should be used for inter-species conversion of dose from animals to
humans?
• How should particular physiological characteristics of an exposed population be
accounted for?
• How should information on comparative metabolic rates in animals and humans be
used?
Risk characterization 73
• Which models should be used to predict the dispersion of pollutants through the
environment?
• How should dietary habits and other variations in lifestyle be accounted for?
• How could exposure measurements for a small group of people be extrapolated to a
population?
• How should exposures of special risk groups, e.g. pregnant women, children, etc., be
estimated?
• What is the proper unit of dose?
• What are the statistical uncertainties in estimating the extent of health effects?
• What are the biological uncertainties in estimating the extent of health effects?
• Which dose–response assessments and exposure assessments should be used?
• Which population sub-groups should be the focus of attention?
In general, when estimating risks, there are three broad types of uncertainty to
consider. These are:
• uncertainty to do with missing or incomplete information describing the situ-
ation under investigation;
• uncertainty affecting a particular parameter, e.g. measurement or sampling
errors, variability in the parameter;
• uncertainties in scientific models, e.g. of the dispersion of pollution through an
environment, or of the true form of the dose–response function.
While some of these uncertainties could be reduced by the collection of more data,
this is not necessarily true and may be prohibitively expensive. Therefore, it may
well be necessary to live with the uncertainty. However, various regulatory agencies
have produced sets of principles for risk assessment which provide general guid-
ance for alleviating some of the problems of inter-agency inconsistency. Table 7.1 is
based on the American experience (Rodricks 1992), but is similar to those in use by
other agencies around the world.
ActivityIn7.3looking at the principles in Table 7.1, where do you think the emphasis lies in terms of
the protection of health? In particular, note principles 3, 4, 7 and 8.
74 Assessing environmental health risks
Table 7.1 Guiding principles for the conduct of environmental health risk assessment
Principle Guidance
Principle 1 In general, data from studies in humans are preferred to animal data
for hazard and dose–response evaluation.
Principle 2 In the absence of human data, or when the available human data are
insufficiently quantitative or are insufficiently sensitive to rule out
risks, animal data will be used.
Principle 3 In the absence of information to demonstrate that such a decision is
incorrect, data from animal species, strain, and sex, showing the
greatest sensitivity to a chemical’s toxic properties will be selected as
the basis for human risk assessment.
Principle 4 Animal toxicity data collected by the same exposure route as that
experienced by humans are preferred for risk assessment.
Principle 5 For all toxic effects other than carcinogenicity, a threshold in the
dose–response curve is assumed. The lowest NOEL from all available
studies is assumed to be the threshold for the groups of subjects
(humans or animals) in which toxicity data were collected.
Principle 6 The threshold for the human population is estimated by dividing the
NOEL by a safety factor, the size of which depends on the nature and
quality of the toxicity data and the characteristics of the human
population.
Principle 7 For carcinogens a linear, no-threshold dose–response model is
assumed to apply at low doses.
Principle 8 Generally, human exposures and resulting doses and risks are
estimated for those members of the population experiencing the
highest intensity and rate of contact with the chemical, although
other, less exposed sub-groups and people experiencing average
exposures will frequently be included.
Source: Rodricks (1992)
Feedback
These principles are all ‘conservative’, that is, they all err on the side of caution by
assuming the worst case. This can lead to a criticism of risk assessments – that they can
result in overly restrictive policies through the application of multiple layers of caution.
The US Environmental Protection Agency (1995) refers to the need for ‘an
appropriate level of conservatism’. This is an issue you will learn much more about
later. Meanwhile, the following principles have been proposed:
Conservative principles
• actions should adequately protect public health and the environment;
• risk assessments should be transparent;
• conclusions drawn from the evidence should be separate from policy judgements
(which are the responsibility of policy makers);
Risk characterization 75
• the nature and likelihood of adverse health effects should be described along with
strengths and limitations of the assessment;
• health risk assessment should in many instances be undertaken with a recognition that
it is part of a larger environmental assessment;
• to protect public health and the environment an appropriate level of conservatism
should be incorporated to protect against uncertainties;
• if comparisons are made against environmental health criteria the criteria used should
be ones approved by the municipality, region or state;
• when using environmental health criteria to assess a situation, these should preferably
be based upon data (toxicological, exposure, etc.) from the region in question.
Summary
You have now examined all stages of the classical environmental health risk
assessment model. This may have been quite demanding technically, but it is
important, even if you are not technically inclined, to have some understanding of
science’s contribution to policy. As you have seen, the final stage of risk charac-
terization presents many difficulties for science, involving, as it does, the interface
between science and policy. You have seen the importance placed upon the proper
presentation of risk assessments as inputs to policy formulation and some of the
requirements for dealing with the issues arising. However, environmental health
risk assessment is a comparatively new and evolving field and you need to be aware
of how it is changing. You will reflect on this in later chapters.
References
Covello VT and Merkhofer MW (1993) Risk Assessment Methods: Approaches for Assessing Health
and Environmental Risks. New York: Plenum Press.
Health and Safety Executive (1989) Risk Criteria for Land-Use Planning in the Vicinity of Major
Industrial Hazards. London: HMSO.
Paustenbach DJ (1989) The Risk Assessment of Environmental Hazards. New York: John Wiley &
Sons, Ltd.
Rodricks JV (1992) Calculated Risk: The Toxicity and Human Health Risks of Chemicals in Our
Environment. Cambridge: Cambridge University Press.
UK Department of the Environment (1995) A Guide to Risk Assessment and Risk Management for
Environmental Protection. London: HMSO.
US Environmental Protection Agency (1995) Guidance for Risk Characterisation. Washington,
DC: US EPA Science Policy Council.
World Health Organization (1987) Air Quality Guidelines for Europe. Copenhagen: WHO.
SECTION 3
Rational action
and environmental
health policy
Economic appraisal
8
Overview
In Section 2 of this book you learnt how environmental health risk assessment is
carried out. Section 3 is concerned with risk management, or decision making.
Public policy decisions, including those dealing with environmental health, have
to reflect economic, social and political realities. You will recall, for example, that
the WHO has said that its air quality guidelines ‘must be considered in the context
of prevailing exposure levels and environmental, social, economic and cultural
conditions’ (WHO 1987). In this first chapter of Section 3 you will consider the role
of economics.
Learning objectives
Key terms
Contingent valuation Survey approach to asking individuals to imagine markets exist and to
give their willingness to pay for (accept) benefits (losses).
Ex-ante decisions Decisions that are made in advance of some potential outcome.
Expressed preference studies Studies that elicit consumers’ preferences by asking them
questions about real or hypothetical scenarios.
Externality Cost or benefit arising from an individual’s production or consumption decision
which indirectly affects the well-being of others.
Opportunity (economic) cost The value of the next best alternative foregone as a result of the
decision made.
Pareto rule An intervention satisfying this rule would improve the well-being of some people
without harming anyone else.
Potential Pareto improvement (Kaldor–Hicks principle) The basis for cost–benefit analysis. It
80 Rational action and environmental health policy
stipulates that a reallocation of resources which makes someone better off and someone worse
off represents an improvement only as long as those who gain could potentially compensate
those who lose.
Quality-Adjusted Life Year (QALY) The value of a year of life adjusted for its quality. A year in
perfect health is considered equal to 1.0 QALY.
Revealed preference studies Studies of actual consumer behaviour that indirectly reveal their
preferences.
Value of a Statistical Life (VSL) A monetary value used for assessing the efficiency of
interventions to improve health or safety of unknown individuals.
Willingness-to-pay (WTP) A method of measuring the value an individual places on reducing
the risk of developing a health problem or gaining an improvement in health.
The following case study has been described by Stevens et al. (2005). The Mexico
City urban area has an estimated population of 18 million residents. The city is
located in a shallow basin and also contains some 3 million vehicles, emitting
exhaust which is associated with a number of harmful effects upon the health of its
inhabitants. Toxicological and epidemiological studies indicate that the worst of
the pollutants emitted by the vehicles is fine particulate matter (PM), similar to
soot, which is associated with increased mortality, chronic bronchitis, and other
cardio-vascular and respiratory health outcomes. These emissions come mainly
from diesel vehicles. Environmental health policy makers have a number of
options open to them to deal with this hazard. These include requiring older,
smokier, diesel vehicles to be replaced, requiring retrofitting of existing diesel
vehicles with either particle filters or oxidation catalysts, or introducing more
stringent emission standards for new diesel vehicles.
ActivityGiven
8.1
that economics is concerned with the allocation of scarce resources among
many needs, what kind of information would you, as a policy maker, wish to have as an
aid to choosing which, if any, of the above options to recommend?
Feedback
It is essential to know how effective each of the measures would be in reducing emis-
sions, exposures, and hence harmful health outcomes. Also important are the costs of
the various programmes for the inhabitants of Mexico City, including those of retiring
old vehicles, or of purchasing, installing and maintaining filters or catalysts. The policy
maker would also need to think about other factors, either costs or benefits, which
might result from implementation of these strategies. For example, if it were decided
Economic appraisal 81
to opt for stringent (and expensive) emission controls on new vehicles, this might
encourage owners of older vehicles to keep them on the road for longer. Since older
vehicles are generally smokier, this could make things worse. Likewise, if oxidation
catalysts were required, would there be any health problems associated with their
disposal after they had been used? The policy maker would also want to have some
means of comparing the health benefits of each policy option against its costs, and
preferably this would be done in the same units of measurement (dollars, pesos, etc.).
It is useful to step back from the above example for a moment and think about the
broad categories of approach available to environmental health policy makers.
There are three such categories which require some form of regulation, and add-
itionally there are various approaches which rely upon voluntary agreements.
The first and traditional approach can be described as ‘design-based command and
control’, and is one which requires certain technologies or measures to be adopted.
For example, these could take the form of a requirement for domestic water sup-
pliers to use a specified filtration system, or to adopt certain management pro-
cedures to maintain the system. The second approach is ‘performance-based’. This
would specify the maximum allowable level of pollution in the medium (air, sur-
face water, drinking water, soil, food, etc.) of concern, as, for example, in WHO air
or water quality criteria. A third approach is to use market-oriented techniques.
These make use of incentives of some kind or another, for example, taxes or
subsidies, to encourage markets to function more efficiently.
Feedback
Design-based command and control measures are usually enacted in the form of emis-
sion controls. Requiring all polluters to adopt the same controls is fair but potentially
wasteful of resources overall, because some polluters may be located far from vulner-
able people. Another way of expressing this is to say that the dose, which is what
counts, is only indirectly related to the emissions. Performance-based control, on the
other hand, which might typically require a polluter to reduce emissions to comply with
an environmental quality standard (e.g. the maximum permissible amount of arsenic in
drinking water), are fairer from the point of view of those exposed, since nobody
should then receive an unacceptable dose of the harmful agent. Another advantage is
that the polluter has some flexibility in choosing how to reduce emissions, and may be
able to do this at lower cost than if same-for-all, design-based control, were enforced. A
disadvantage is that ambient monitoring is required to check compliance and this in
82 Rational action and environmental health policy
itself can be costly. Market-based approaches are more subtle in that they attempt to
adjust imbalances in the operation of the market. For example, it may be the case that
automobile manufacturers or operators do not feel that ambient air quality is their
concern (in economic jargon, it is an ‘externality’) and so vehicles are designed and
operated with little or no attempt to control emissions. However, from the point of
view of society as a whole, air is a common ‘good’ and its quality should be protected.
Therefore, a plausible solution is for policy makers to intervene in the market, with
subsidies, incentives, or some other measure, so that the externality is internalized in
vehicle manufacturers’ decision processes. The attractiveness of these types of meas-
ures lies mainly in their potential economic efficiency; however, what actually happens
remains under the control of the market and not the policy maker.
Table 8.1 includes these three approaches and a number of others to be discussed.
Zero or minimum risk is a special case which is applied in circumstances where a
hazard is deemed to be intolerable at any level of risk, and examples, based on the
actions of some countries, include DDT, PCBs, and the use of lead additives in
paint. Risk-based controls, on the other hand, work on the basis that there is a
residual level of risk which is acceptable to society, and we will investigate this
carefully in Chapters 9 and 10. Utility-based controls, however, are closer to what
the remainder of this chapter is about. It should be noted that the word utility has a
number of meanings. The common or dictionary definition refers to ‘a condition of
being useful or profitable’. In Table 8.1, it refers to an approach which weighs both
the benefits and the costs of an intervention. In economics, it usually refers to the
subjective value, as opposed to the actual value, of something.
Economic appraisal
The example of Mexico City introduced the notion of what is known as economic
appraisal. In general, international bodies, governments, municipalities, public
and private sector agencies of all sizes, and even individuals, are concerned in one
way or another with the economic appraisal of investments of resources including
environmental health interventions. That is, all new policies should be subject to
comprehensive but proportionate assessment, so as to best promote the public
interest.
Management experts in a wide variety of arenas use the ‘ROAMEF’ cycle to help
them make decisions: Rationale → Objectives → Appraisal → Monitoring →
Evaluation → Feedback (Figure 8.1).
The aims of economic appraisal are to ensure that all policies, programmes and
projects face the following questions before being given the go-ahead:
• Are there better ways to achieve this objective?
• Are there better uses for these resources?
This can be achieved through:
• identifying other possible approaches;
• wherever possible, attributing monetary values to all impacts;
• assessing the costs and benefits of options.
Table 8.1 Summary of alternative environmental control philosophies, all of which can be witnessed in use in the management of environmental hazards
Risk management Zero or Ambient Emission Technology-based Risk-based Utility-based Hybrid
philosophy minimum risk standards standards control control control approaches
Basis Commitment Health Level playing field Use of Scientific Utility theory Various
protection for all for polluters recognized simulation
technology
Strengths Clarity of goal Compliance Compliance Standardization; Analytical tool; Considers both May
relatively easily relatively easily encourages ability to forecast costs and accommodate
checked; equity checked; equity technological the unknown benefits of equity, utility and
for receptors for emitters innovation control other attributes
Limitations Associated Exposure of Dose and hence Potentially Data hungry; Anchored in a Difficulty of
benefits receptors not harm is only expensive; often large particular striking a balance
foregone; control accounted for indirectly related inflexible; may uncertainties in philosophy; between multiple
costs disregarded to emissions lead to e.g. dose– methodological attributes;
‘ratcheting’ response problems, requires
functions and especially in considerable
exposure valuing benefits intellectual
assessments input; open to
argument
Examples DDT, PCBs, lead- Air quality Discharge limits Use of BAT – Control of BATNEEC – ALARP/ALARA –
in-paint criteria to aquatic best available ‘non-threshold’ best available as low as
environment technology pollutants technology not reasonably
especially entailing practicable/
carcinogens excessive costs achievable
Source: Ball (2002)
84 Rational action and environmental health policy
Figure 8.1 This is the ‘ROAMEF’ cycle, numerous versions of which can be found in
management guides
Source: HM Treasury (2003)
Ideally, appraisal is carried out prior to a decision to implement some measure, but
it may be used for retrospective analysis of existing policies or measures.
In 1992, the WHO and the Gambian government initiated the National Impreg-
nated Bednet Programme in an attempt to reduce the effect of malaria on the
population (Aikins et al. 1998). Local health care centres provided permathrin dip-
ping facilities, an insecticidal treatment, free of charge to local people who brought
their bed nets in. The cost–effectiveness of the measure was investigated so that
it could be compared with other malaria control measures and other health
programmes which were competing for resources.
The main costs were found in insecticide provision, with other costs including
administration, transport and facilities. The consequences were measured in terms
of reductions in morbidity and mortality, by the effect on school attendance, and
by resource savings in the health service and from the point of view of households.
Health sector savings were mainly in the form of reduced treatment costs for mal-
aria, but for households there were other benefits besides treatment costs, for
example, less time spent caring for sick children.
The results showed that the total programme cost was approximately $90,000, 70
per cent of which was for the insecticide. The estimated number of deaths avoided
by the programme was 41, the number of life-years gained was 1,600, and the
number of child-years of protection 20,000. Thus, the cost per death voided was
$470, per life-year gained $27, and per child-year protected less than $1.
In general, the relevant costs and benefits of each option (possible intervention)
should be assessed. As in the above example, you first have to decide from whose
perspective you are carrying out the assessment. The perspective of a national gov-
ernment may differ from that of a private company, a municipality, an NGO, or an
individual.
Having decided, you will need to consider all the costs and benefits associated with
the intervention. Costs and benefits are normally assessed in terms of market
prices. However, in the field of public health you will often be dealing with the
costs and benefits of goods that have no market value and if your interest is cost–
benefit rather than cost–effectiveness analysis of health interventions, you will
have to go a step further and find a means of placing a value on health. The
problem is that human health has no market value because it is not a traded
commodity, press gangs and slavery having been outlawed, if not eliminated.
Yet another example of a good, or commodity, which is not directly traded is
people’s personal time. From the point of view of a company you might well
include in your cost–benefit assessment the time of your own staff (and price it
86 Rational action and environmental health policy
according to their wages and overheads) but not of other parties, e.g. the public,
who might be affected. Thus, time looking after sick children might be excluded,
though from a societal perspective it should be considered.
These non-traded goods are sometimes referred to as externalities. The problem with
externalities is that if they are omitted from the balance sheet, perhaps because
they are not seen as capable of being valued, they may not be considered at all in
the policy maker’s decision process. Later on in this chapter we will consider how
to value such non-traded commodities.
Figure 8.2 Health effects and costs (y axis) versus risk exposure (x axis)
some people without harming others. Although this seems a logical rule, its
application could result in interventions which benefit the well-off while doing
nothing for the poor, thus increasing inequality. Another difficulty is that the strict
Pareto criterion does not permit interventions which cause a disbenefit to anyone
at all. Usually, in the practical world, it is necessary to consider interventions which
benefit some people but may harm at least some others. Hence decision makers
normally apply the Kaldor–Hicks rule, which says that there is a potential Pareto
improvement if an intervention creates a situation in which the beneficiaries gain
enough that they could, in theory, compensate the losers for their losses. Thus, in
order to determine if an intervention offers a potential Pareto improvement, it is
sufficient to sum the monetary benefits and losses across the affected population
and see if the difference is positive or negative.
ActivityGive
8.3
an example of an environmental health intervention consistent with the strict
Pareto criterion and an example of one satisfying the Kaldor–Hicks rule.
Feedback
If you think about the wider implications of environmental health interventions, includ-
ing the fact that they require resources to be consumed which constitutes a disbenefit
for people who pay for them but who may not receive the benefits, then it is very hard
to think of any interventions which satisfy a strict Pareto criterion. Even if you do not
consider resource implications, it is usually the case that environmental health interven-
tions will cause some form of disbenefit to some people. The banning of the use of
DDT in many countries, for example, would be seen as a benefit by those concerned
about the effect this chemical has on the environment, but as a disbenefit by those
trying to control malaria. Potentially, however, it might be possible to argue the case for
the banning of DDT under the Kaldor–Hicks rule if you could show that the environ-
mental benefits to those who gained were greater than the health losses to those
exposed to malarial infection. Whether or not a good case could be made is debatable
(Bate 1999a, 1999b).
The above discussion leads directly to the issue of placing monetary values on non-
traded goods, for if techniques such as cost–benefit analysis are to be used, both the
costs and the benefits must be valued in monetary terms. For the environmental
health policy professional, the non-traded goods are clearly human health, human
safety, human lives, and the environment. This is a difficult area on two counts.
First, it raises ethical questions about the very practice of putting monetary values
on things like human health. Second, if the practice is accepted, there is the ques-
tion of how you actually arrive at numerical values for the purpose of cost–benefit
calculations.
Economic appraisal 89
ActivityThe
8.4
following passage, adapted from Barbara Soby and David Ball (1991), describes
arguments against putting monetary values on human health. As you read the extract,
think about your own position. Do you agree with the arguments or not? Can you think
of counter-arguments?
Feedback
The first of the above arguments is outwardly the most problematic. Nobody wants to
put a monetary value on human life or someone’s health. In fact, however, this is not
what is being done when monetary values are assigned to health and life for policy
purposes because the values are for use in forward planning (that is, from an ex-ante,
position) and importantly involve small changes in health risk. They are in no way meant
to measure the intrinsic worth of individuals or their well-being. Soby and Ball (1991) go
on to say that since resources for increasing health or safety are limited, placing an
explicit value on health can help decision makers make more efficient decisions, thus
preventing more ill health and saving more lives. Indeed, every time a decision is made
to spend or not to spend on a health prevention measure, human health is implicitly
valued. Failure to be explicit about it does not mean that health, life or injury is not
being valued, merely that the valuation is hidden. Furthermore, not placing an explicit
value on health and life does not provide a neutral stance: instead it potentially under-
values health, or over-values it, leading in both cases to inefficient spending on health.
With respect to the second objection, there is no reason in principle why qualitative
90 Rational action and environmental health policy
weighting factors should not be used to accommodate these issues, although the
evidence is that the factors themselves are not greatly different from unity. The remain-
ing three objections address the decision making process, and not the valuation of
health per se. The objections are legitimate and deserve attention, but do not force one
to dismiss the role of health valuation. Instead, a policy maker using such values in policy
analysis should retain the option of including these other factors within her decision
making model.
ActivityThe
8.5
economist Michael Jones-Lee (1984) approaches the ethical dilemma by examining
the alternatives to valuing health and concludes that these pose their own ethical
problems.In reading his list of four alternatives,consider how these ethical problems arise:
1 Ignore estimates of the value of health on the grounds that there is no obviously
correct way to value health or potential loss of life and because it is morally repug-
nant to try to do so.
2 Use informal judgement and educated good sense to estimate the benefits of health
interventions.
3 Set environmental standards (e.g. environmental quality standards or emission
standards) to protect health regardless of cost and use those instead.
4 Use cost–effectiveness analysis to identify the option that provides the greatest
health gain at least cost.
Feedback
The first approach effectively places the value of health, and life, at zero or infinity.
Either way, this will lead to a loss of efficiency and less good will be done. The second
approach, which relies on informal judgement, is likely to be inconsistent and therefore
inefficient also. The setting of environmental standards begs the question of the criteria
by which the standards are set (as the WHO has said, standards, inter alia, have to take
account of economics). Finally, cost–effectiveness analysis provides no indication of
how much it is reasonable to allocate towards environmental health protection.
From the above discussion you will note that the application of cost–benefit analysis
in health-related decision making is not to be casually undertaken. It is rightly an
area of considerable sensitivity and the professionals who use it must be able to
justify their actions as well as understand the associated limitations of the approach.
We now move on to the second major issue, which is how to arrive at valuations of
health states, and life even, for use in ex-ante environmental decision making.
most difficult proposition – valuing a human life. Recall that this is attempted for
the purpose of ex-ante decision making about health interventions which will
reduce risk, in most cases by small amounts, and is not therefore about the intrinsic
value of health or life.
In the early days, by which is meant up to some 50 years ago, it was suggested that a
monetary value might be assigned to a human life by examining one or more of the
following:
• life insurance coverage
• legal compensation
• the Human Capital approach.
Although seeing how much life insurance people purchased is simple enough, this
method quickly fell out of use because it was realized that insurance, where it was
purchased, was usually for the benefit of a person’s heirs and so was not therefore
an indication of that person’s own valuation of his or her life. Likewise, although it
was suggested that court awards for cases involving death and injury might give a
guide to the value of avoiding death and injury, this also proved problematic. This
was because awards may contain a punitive element, are frequently to compensate
surviving family members rather than provide a sum equivalent to the value of the
deceased’s life, and represent ex-post (after an event has occurred) decisions rather
than ex-ante decisions (which is what policy choices are), which may not be the
same. Nonetheless, painstaking work by the health economist Ted Miller (1990) in
the USA has enabled him to value a human life, by careful use of this technique,
at $3m, and this value is widely used in the USA for safety and health decision
purposes. Until the mid-1970s the favoured approach in the UK and some other
countries was that known as Human Capital. This valuation is based on the loss
of expected future earnings of an individual experiencing a fatal or non-fatal acci-
dent. The calculation takes account of the average age of individuals at risk, the
probability of an accident occurring, and either gross earnings or earnings net of
consumption.
As an example, if it is believed that the average age of car drivers who die in
road traffic accidents is 40 years, that their future earnings would have averaged
$5,000 per year, and that they would have retired from work at 65 years, then what
has been lost in terms of earnings is (65 − 40) × $5,000, or $125,000 and this is
the value of their life according to the Human Capital approach. While relatively
simple, the approach is clearly open to the criticism that the valuation is solely in
terms of people’s economic output. Unsurprisingly, this came to be seen as too
cold-blooded, for people’s lives are surely worth more than their wages, and in any
case some people are not employed, yet their lives are still valued.
By and large, the preferred techniques for valuing life and non-fatal injuries, at least
in Europe, now centre upon a suite of techniques under the banner of willingness-
to-pay (WTP). There are many authoritative documents which describe these tech-
niques, but a useful reference which is regularly updated and can be found on the
internet, is The Green Book of the UK’s HM Treasury (2003).
The methodology is as follows: first decide whether the impacts of activities or
interventions can be measured and quantified, and if prices can be determined
from market data. If this cannot readily be done, use willingness-to-pay (WTP) to
value a benefit, where WTP is determined by inferring a price from observing
92 Rational action and environmental health policy
consumer behaviour. If this does not provide values, determine whether WTP can
be estimated by asking people what they would be willing to pay for a benefit, or
(in the case of a cost) how much compensation consumers would demand in order
to accept it.
The approach raises new issues. For example, how could you infer a price or value
of something like health by observing consumer behaviour?
ActivityIf8.6you are a householder, how many smoke sensors or other recommended safety
devices have you fitted in your home? As many as the manufacturers or the fire brigade
recommend? Or did you decide to install just a few (or perhaps none at all) in view of
the cost and trouble? Could you infer something about your valuation of your own
safety by studying your behaviour in relation to the purchase or non-purchase of safety
devices? Think about these questions in your own context and write down some
answers.
Feedback
Researchers have studied the purchasing behaviour of consumers in consumer societies
with regard to smoke sensors and other items and from this have been able to place
limits on the value of life to which consumers appear to be responding. Making these
calculations does require some heroic assumptions. One is that consumers understand
the risk associated with buying and not buying smoke sensors or other safety devices.
Another is that they perform some sort of intuitive mental calculation of the risk of
harm in deciding whether to purchase them or not.
Despite these difficulties, numbers have been generated by studies such as these. If
you can find the data, the crucial part of the calculation is this: if a consumer is
prepared to spend an equivalent of $5 a year on smoke detectors to reduce her risk
of dying in a fire by 1 in 50,000 per year, then that consumer’s value of her own life
for safety purposes is given by:
This is because the $5 in effect purchases a fraction of that consumer’s life equal to
冢 冣
1 1
. Note that we are talking about a tiny fraction of a person’s life, not
50 000 50 000
a whole life, when this calculation is done. These calculations are only valid in
such circumstances, not when whole lives, especially of known persons, are at
stake.
A second WTP (or, more accurately, willingness-to-accept (WTA)) approach is to
study how much workers in risky jobs, such as mining, railway maintenance, deep
sea fishing, or sea diving, are paid for their work. It is logical that they should
receive extra pay in order to be attracted into or compensated for more dangerous
Economic appraisal 93
work. Some research suggests that this is the case, and valuations of life based on
these ‘compensating wage differentials’, as they are known, have been derived.
Within the discipline of health economics, the above techniques are known as
‘revealed preference’ methods. This is because people’s valuations of health risks
have been revealed by their behaviours, either at home or in the jobs they choose.
A related technique is to study how much institutions spend on health and safety
programmes. This was touched on when we discussed the work by Tammy Tengs
and colleagues in Chapter 2. If you can find out how much institutions are spend-
ing on specific health or safety interventions and what that means in terms of
greater health or safety, it is relatively straightforward to calculate the implied
value of health and safety. For example, if an agency has spent $1 million on a
health measure which it is believed will save 50 lives during its period of applica-
tion, then that agency clearly values a life at $20,000 or more.
Figure 8.3 shows the range and type of WTP approaches, from which it can be seen
that there is a second type known as ‘expressed preference’. These techniques
attempt to overcome a key difficulty of revealed preference techniques, that is, that
consumers may have other things on their mind when they make their decisions.
For example, a homeowner may buy smoke sensors or some other safety measure to
protect her children rather than herself, or may not understand the risk and be
persuaded more by advertising than any personal calculation of risk and benefit.
The solution to these problems is to pose a hypothetical situation to respondents in
the form of a question and ask them to respond. In this way the question can be
tailored precisely to address the point in which the researcher is interested. This
method is sometimes referred to as ‘contingent valuation’ because it is contingent
upon the existence of the hypothetical situation described.
ActivityHere
8.7
is an example of a fairly typical expressed preference WTP question. Answer it
and see if you can calculate your own value of your life.
94 Rational action and environmental health policy
Feedback
Logically, you should be prepared to pay something extra for your greater safety.
Suppose you are prepared to pay $2 more to go with company B. In effect, this would
buy you a risk reduction of:
1 1 2
− =
50 000 150 000 150 000
2
That means that for $2 you have avoided a risk of to your life, so the implied
150 000
value of your life is:
Researchers typically obtain answers to these types of WTP questions from ran-
domly selected samples of 1,000 or so people in the population of interest. Zeros,
which are not uncommon, and other ‘strange’ answers would be discarded, and the
remaining results essentially averaged (or the median taken) to come up with a
representative societal value of an individual’s life. Sometimes the values arrived at
are supplemented by an estimate of gross lost output (that is, wages not earned as a
result of dying prematurely, as in the earlier Human Capital calculation), and any
medical and ambulance costs. Table 8.2, for example, shows values from the UK
Department of Environment, Transport and the Regions in £(1997) per road casu-
alty, with the second column of data being based on WTP techniques derived from
a WTP survey using questions similar in style to that in Activity 8.7. The resulting
valuation is often referred to as the VSL (Value of a Statistical Life), since it applies
not to a particular individual but to an unknown ‘statistical person’.
Currently, the UK Department for Transport (DfT) uses a value of £1.145 million
per fatal casualty prevented (in year 2000 prices). This is used in the context of road
safety investment decisions. DfT also attributes monetary values to the prevention
of non-fatal casualties, also based on WTP techniques, as shown in Table 8.2 (http:/
/www.dft.gov.uk). The figure currently used for a serious road injury is £128,650,
and for a slight road injury £9,920. Valuations generally increase each year due to
changing prosperity, but sometimes they may change because of revisions to the
methodology (as when the UK switched from pure Human Capital to WTP in about
1985, causing the valuation of a life to jump from £185,000 to £465,900). Similarly,
the UK Health and Safety Executive (2001), whose primary responsibility is occu-
pational health and safety, uses a value of £1 million (2001 prices) for the value of
preventing a fatality, and most other UK agencies use similar values.
Around the world many safety valuation studies have been carried out. Figure 8.4
shows how widely the results are scattered. There are different ways of dealing
with results such as these. One is to select a middle value by some means and
another is to use a range of values. The former is more convenient and is the
conventional approach, but the latter is perhaps more realistic – it also forces the
user to acknowledge uncertainties.
Valuations of this kind are not restricted to matters of safety in relation to acci-
dents. They are also applied in occupational health and safety, for example. Similar
approaches are also found in health care services where it is necessary to consider
the cost–benefit ratio of alternative means of investing resources. The main differ-
ence in the health sector is that the health impacts, or health conditions being
considered, are not simply about saving lives, but about improving quality of life.
As a consequence, in the health sector, the term QALY, standing for quality-
adjusted life year, is encountered. The QALY scale allows both life years and quality
of life to be expressed in a single measure by weighting life years (saved or lost) by
the quality of life experienced in those years. QALYs need to be valued too. This can
either be done directly by expressed preference WTP studies using different health
states as the numerator, or it could be done by linking the train of one’s remaining
life years, measured in QALY units, to the WTP-derived value of life (Ball and
Soby 1995). In the latter case, consideration needs to be given to the issue of
discounting, described below.
WTP techniques have also been applied to environmental assets as an aid to
environmental decision makers. The Green Book (HMT 2003) contains useful
examples, which you can follow up if you wish, ranging from the valuation of air
quality to landscape, water quality, noise, the cleanliness of bathing beaches and
sea water, and the recreational value of forests. Other techniques are available
too, for valuing environmental resources. If this attracts your interest, a useful
introduction has been written by RK Turner et al. (1994).
96 Rational action and environmental health policy
Figure 8.4 The range of values of a statistical life from numerous studies
Source: Ball (2000)
Economic appraisal 97
Discounting
Discounting is a sometimes complicated and contentious issue and you can find
much about it on the internet. It encompasses the human sentiment that ‘A dollar
today is worth more to me than a dollar next year’, or, with some extension, that
‘Happiness is worth more to society now than next year.’
Discounting has come to mean any process of revaluing a future benefit to yield a
present value (or value today). The process of discounting reflects the complexities
of financial markets, politics, and human interests, but tends to be expressed in
fairly simple mathematical terms. Thus, the Present Value of some quantity (PV) is
given by:
PV = Xt/(1 + r)t
Where Xt is the expected benefit or cost of some item t years in the future and r is
the discount rate. The formula implies that having an amount PV in cash now is as
good as having a greater amount Xt in cash in t years’ time. So if you were told you
would receive $1,000 in ten years’ time, and if the discount rate were 5 per cent
(i.e. r = 0.05), then the present value would be $613.91.
Using a similar approach, the present value of a future stream of costs and benefits
associated with a project or safety intervention can be estimated and compared.
The recommended discount rate, sometimes referred to as the Social Time Prefer-
ence Rate (STPR), is 3.5 per cent in The Green Book, although the effect of different
rates is tabulated (Annex 6 of The Green Book). Generally, the higher a discount
rate, the less important appears the future. For this reason lower discount rates are
used for projects with lifetimes in excess of about 30 years. For example, for projects
extending to more than 300 years a discount rate of 1 per cent is recommended.
Even this is contested in some circles, however, as being too high and devaluing the
future.
If you need to evaluate projects into the future, then consideration of discounting
will be necessary, so follow it up in more detail (e.g. Annex 6 in The Green Book)
and look at a number of sources for advice. There are standardized procedures to
follow but be aware of the arguments for and against. The choice ultimately is
political.
This chapter began with a case study of air pollution in Mexico City. The authors
of the study (Stevens et al. 2005) were able to estimate the costs of the various
programmes to reduce fine particulate emissions from diesel vehicles, and their
effectiveness in reducing the concentrations of soot in the air. In turn, by using
models, they estimated population exposure under the different programmes and
finally, with the aid of suitable dose–response curves, the health benefits of reduced
emissions in terms of annual lives saved.
98 Rational action and environmental health policy
ActivityOne
8.8
particular problem faced by the authors was that of valuing, in dollars (or some
other currency), the value of the health benefits of the resident population. As you read
the following edited passage, note the difficulties and how they were approached.
Deriving a VSL
To compare the health benefits of a retrofit program to its costs, the benefits and costs
were converted to a common metric. We assigned monetary values to the reduction in
mortality risk using the value of a statistical life (VSL) methodology. VSL is a measure
of individuals’ willingness to trade wealth for small changes in mortality risk; it is not a
measure of the intrinsic worth of an individual. Because individuals in developing countries
earn less, it follows that they are usually willing to pay less (or accept less compensation)
for reductions (or increases) in mortality risk. Therefore, we expect the average Mexican
VSL to be lower than the average US VSL.
Ideally, we would derive a VSL from studies in Mexico. However, only one study of VSL
is available for Mexico. Given that VSL estimates often vary significantly among studies,
we estimated a range by adjusting US estimates of VSL for the difference in income,
using an estimate of the income elasticity. The income elasticity of VSL is defined as
the proportional difference in VSL associated with a proportional difference in income.
Income elasticity observed in developed country analyses is typically less than 1, which
indicates that as wealth decreases, the proportion of income an individual is willing to
pay to eliminate a specified risk increases. Two studies found an income elasticity of
approximately 0.5 when comparing a number of VSL studies in developed countries. In
contrast, comparing VSL estimates from middle-income countries (including Mexico)
to estimates for high-income countries indicates that income elasticity is much higher,
around 2.
A plausible range for a Mexican VSL was obtained by extrapolating from US estimates
using income elasticities of 0.5 and 2. Given that the US EPA’s central estimate of US VSL of
$6.2 million (adjusted to year 2000) and the Mexico/US gross national income per capita
ratio of 0.17, the range for Mexican VSL is $170,000 to $2.5 million.
Feedback
To make a direct comparison of the costs of the interventions against their health
benefits it was necessary to value the health benefits in monetary terms. A difficulty is
that studies of the value of health and safety have mainly been conducted in higher
income countries, so what data could be used? The authors tackle the problem by using
the US value of a statistical life (VSL) and scaling it according to the ratio of the gross
national per capita income for Mexico and the USA. The calculation takes note of the
fact that preferences for spending on health as a function of income in middle income
countries may differ from that in high income countries. This is captured by stating the
VSL as a range. Note that this is an area of some sensitivity as it may impinge upon
national pride if interpreted in a certain way.
Economic appraisal 99
ActivityRead
8.9
the following abridged passage by Stevens et al. (2005) which summarizes the
conclusions of their study on Mexico City, and note how their recommendations are
expressed.
Feedback
It is important to be aware that cost–benefit studies are generally regarded as an aid to
decision makers and not as an alternative. This is because cost–benefit studies, even
though they can internalize some quantities which were formerly omitted, do not
internalize all conceivable externalities. Depending on how widely the study is framed,
there may also be other ways of achieving the desired goal, for example, by encouraging
greater reliance upon public transport. In short, CBA studies do not tell policy makers
what to do, but they should assist them in their task.
Summary
References
Aikins MK et al. (1998) The Gambian national impregnated bednet programme: costs, con-
sequences and net cost-effectiveness. Social Science and Medicine 46(2): 181–91.
100 Rational action and environmental health policy
Ball DJ (2000) Consumer affairs and the valuation of safety. Accident Analysis and Prevention
32: 337–43.
Ball DJ (2002) Environmental risk assessment and the intrusion of bias. Environment Inter-
national 28: 529–44.
Ball DJ and Soby B (1995) Valuing consumer safety. International Journal for Consumer Safety
2(3): 117–31.
Bate R (1999a) Pollutants treaty condemns the poor. Chemistry and Industry, 1 March, p. 200.
Bate R (1999b) What Risk? Science, Politics and Public Health. Oxford: Butterworth-Heinemann.
HM Treasury (2003) The Green Book: Economic Appraisal and Evaluation in Central Government.
London: The Stationery Office. Also available on the internet.
HSE (2001) Reducing Risks, Protecting People: HSE’s Decision Making Process. Sudbury: HSE. Also
available on the internet.
Jones-Lee M (1984) The valuation of transport safety, in Proceedings of the Annual Transportation
Convention. Pretoria: Department of Transport.
Miller TR (1990) The plausible range for the value of life – red herrings among the mackerel.
Journal of Forensic Economics 3(3): 17–39.
Soby B and Ball DJ (1991) Consumer Safety and the Valuation of Life and Injury. Research report
No. 9. University of East Anglia Environmental Risk Assessment Unit. Brighton: UEA.
Stevens G, Wilson A and Hammitt JK (2005) A benefit-cost analysis of retrofitting diesel
vehicles with particulate filters in the Mexico City Metropolitan Area. Risk Analysis 25(4):
883–99.
Turner RK, Pearce D and Bateman I (1994) Environmental Economics: An Elementary Introduction.
Hemel Hempstead: Harvester Wheatsheaf.
The management of
9 ionizing radiation
Overview
In this chapter you will see how this framework is assembled, including how it
incorporates and builds upon the previous chapter on economic appraisal.
Learning objectives
Key terms
Collective dose The sum of the radiation doses to the individuals within a specified group,
measured in units called man-sieverts.
Deterministic effects Effects which can be directly associated with their causes and which
generally increase as dose increases. Normally a threshold exists.
Dose equivalent A measure of the amount of radiation absorbed by tissue combined with a
102 Rational action and environmental health policy
factor accounting for the ability of the type of radiation involved to cause damage, measured in
units called sieverts.
Dose limit The officially recognized maximum tolerable dose of ionizing radiation during a
specified time interval.
Effective dose equivalent The dose equivalent incorporating a further weighting to account
for the vulnerability to harm of the body part exposed, measured in units called sieverts.
Ionizing radiation Radiation that is sufficiently energetic to break the bonds that hold
molecules together to form ions.
Sievert (Sv) A unit of equivalent dose of radiation which relates the absorbed dose in human
tissue to the effective biological damage of the radiation. A millisievert (mSv) is one-thousandth
of a sievert.
Stochastic effects Effects that are governed by the laws of probability. The severity of harm is
independent of dose, but the probability of experiencing the harm is proportional to the dose.
There is no threshold dose below which effects do not occur.
Some knowledge of the nature, origin and effects of ionizing radiation is required
to get the most out of this chapter and the following three passages give a brief
overview.
Radioactivity is a natural part of the universe in which we live and life on Earth
would not exist without it. Nonetheless, it can be harmful, particularly at high
doses, and its association with health effects like cancer and human activities like
the manufacture of nuclear weapons has contributed to a widespread societal
concern. It is likely this concern, coupled with the scientific and philosophical
backgrounds of the people who have worked to understand ionizing radiation, has
led to the evolution of the framework for assessing and managing the associated
risks.
Ionizing radiation originates from atoms and their nuclei. For our purposes, atoms
may be considered as made up of negatively-charged particles called electrons
which orbit a small central nucleus which is made up of positively-charged protons
and neutrons which have no charge (Figure 9.1). Each substance or element is
characterized by the number of protons in the nucleus. For example, hydrogen has
just one proton in its nucleus (it also has no neutrons and one orbiting electron.
In its normal state the number of electrons equals the number of protons so that
the charge on the total atom is zero.) In the case of uranium, this has 92 protons
(and an equal number of orbiting electrons in its normal uncharged state). How-
ever, each element, hydrogen, uranium, oxygen, carbon or whatever, while being
defined by the number of protons on the nucleus, may have different numbers of
neutrons. So, for example, uranium comes in two well-known forms which are
uranium-238 and uranium-235, known as isotopes of uranium. Uranium-238 is so
called because it has 92 protons and 146 neutrons in its nucleus, while uranium–
235 has 92 protons and 143 neutrons. These atoms, described in this way, are called
‘nuclides’.
The management of ionizing radiation 103
Some nuclides of some elements are stable and never change, but most are not.
They lose chunks in the form of protons, neutrons, electrons, or pure energy in the
form of electromagnetic radiation, and are said to ‘decay’. Simply speaking, there
are three types of decay or emissions to consider, and these are known as alpha,
beta and gamma (α, β, γ). Alpha particles are made up of a group of two protons and
two neutrons, beta particles are electrons, and gamma emissions, known as gamma
rays, are pure energy in the form of high energy electromagnetic radiation (like
X-rays, but of higher energy). Figure 9.2 shows these emissions and also draws
attention to the fact that they have very different penetrating power. Thus α par-
ticles cannot even penetrate a sheet of paper, nor even the top layer of human skin,
so they are not dangerous to health unless the sources get inside the body through
being inhaled or eaten as a part of the diet. β particles are more penetrating and can
Figure 9.2 The three types of atomic radiation and their penetrating power
Source: UNEP (1985)
104 Rational action and environmental health policy
pass through a centimetre of living tissue, and γ rays can go through anything short
of a thick slab of lead or concrete.
ActivityDamage
9.1
to living tissue is caused by the energy deposited by the radiation as it passes
through and disrupts human cells and damages DNA, and this is called the ‘dose’. Dose
is, however, a complicated concept in radiological protection. Read the following pas-
sage based on the United Nations Environment Programme (1985) description, and
observe the different ways in which it is expressed.
Feedback
Doses are expressed in different ways to take account of how much of the body and
what parts of it are irradiated, whether one or more people are exposed, and the
duration of the exposure. The hierarchy is a little complicated but once understood it
can be seen to provide a coherent structure against which doses can be recorded
consistently and comparably.
Radiation is harmful to life even at low doses because it can start off a chain of
events which can lead to cancer or genetic damage (Figure 9.3), effects which may
not reveal themselves until decades later. At high doses it can kill cells and damage
organs, causing death within a few hours or days.
The acute effects of high doses occur after exposure above a certain threshold
which is in the region of 1 Gray. Environmental exposures, from natural or
man-made sources, are generally very much lower than this, by a factor of 1,000
or so. But in theory at least even the smallest dose poses an increased risk of
cancer and genetic harm because ionizing radiation is taken to be a non-threshold
agent.
Exposure can occur in two basic ways. First, people can be irradiated by external
sources. Second, people may be irradiated internally if they ingest or inhale dust,
food, or water which contains trace amounts of radioactive materials.
By far the greatest contribution to the radiation exposure of the world’s population
is from natural sources. These sources are mainly of terrestrial origin, with the
remaining roughly 10 per cent coming from outer space in the form of cosmic rays.
The Earth’s atmosphere shields us from most of the cosmic rays, but exposure
increases for people living at high altitude or who fly frequently. For people living
at sea level, the effective dose equivalent is about 0.3 millisieverts (one millisievert
1
is sievert) of cosmic radiation per year.
1000
Of the terrestrial sources, radionuclides naturally present in rocks give rise to an
average annual dose of about 0.35 millisieverts. There is some variation across the
planet, due to the make-up of the rocks, and ‘hotspots’ have been found at places
like Poços de Caldas in Brazil (250 millisieverts per year and thankfully uninhabited)
and parts of Kerala and Tamil Nadu in India (up to 17 millisieverts per year).
The above sources are all external and together contribute about one-quarter of
total exposure. The other three-quarters are down to internal irradiation which
results from naturally occurring radioactive substances in food, air and water. These
include potassium-40, lead-210 and polonium-210 which enter the body in food.
Although the average dose per person worldwide from such sources is about 0.3
millisieverts, some of these materials are concentrated in certain foodstuffs like
seafood, and kangaroo and caribou meat where these animals graze in areas natur-
ally rich in them, so local populations can receive higher doses. However, the larg-
est natural source of radiation exposure worldwide is the gas radon which exists as a
minor constituent of the atmosphere. Radon primarily originates from the radio-
active decay of naturally occurring uranium-238 in the ground and seeps from the
earth into the air. The concentration in the air depends on the amount of uranium-
238 in the ground, and it can be higher inside well-insulated, draught-proofed
buildings where it sometimes accumulates. Exposure therefore depends on the
kind of buildings in which people live and the ground on which they are built. It is
not so much the radon itself which is the problem, but that it undergoes quite rapid
radioactive decay into polonium-218 and polonium-214 which, if the radon gas
has been inhaled, can lodge in the lung and are α-emitters. The emission of α
particles in such a sensitive area of the body increases the risk of lung cancer. Doses
attributable to radon decay are typically about 1.3 millisieverts per year, but can be
hundreds or thousands of times higher in special circumstances.
Doses from man-made radiation are usually much smaller than from natural radi-
ation, but vary considerably. In countries that practise Western-style medicine, the
largest source of exposure to the public is the use of X-rays and radioactive
materials to diagnose and combat disease, and average doses to the public of
around 0.3 millisieverts in a year from this source are normal. Occupational
exposure, through working in hospitals or industry, is highly regulated and
exposures are typically in the range from 0.1 to 0.3 millisieverts per year. The
nuclear and other industries, hospitals and universities discharge some radioactiv-
ity to the atmosphere in routine operation, but the resulting public exposures are
generally low and below 0.001 millisieverts per year. One other source which
should be mentioned is nuclear fallout from previous but now ceased atmospheric
testing of nuclear weapons. Doses to the public were typically around 0.14
The management of ionizing radiation 107
millisieverts per annum in the early 1960s but have by now fallen to less than 0.005
millisieverts. Figure 9.4 shows the average relative importance of the various nat-
ural and man-made sources as pertaining for the UK population. Table 9.2 shows
worldwide average annual effective doses.
Figure 9.4 The main sources of radiation exposure in the United Kingdom
Source: HPA Centre (2006)
Table 9.2 Average radiation doses at year 2000 from natural and man-made
sources of radiation expressed in millisieverts (msv) per annum
Source Worldwide average annual effective dose
(msv/annum)
Natural background 2.4
Diagnostic medical examinations 0.4
Atmospheric nuclear testing 0.005
Chernobyl accident 0.002
Nuclear power production 0.002
Source: UNSCEAR (2000)
In the time since the health aspects of radiation have been a topic of research a
great deal of work has been conducted to try and identify the risks. A primary
interest has been the shape of the dose–response curve at low doses. The main
108 Rational action and environmental health policy
source of information has been the lifespan study of the survivors of Hiroshima
and Nagasaki which was most recently reported and assessed by UNSCEAR in
2000. As a result of this and other research, UNSCEAR estimates that the risk of
dying from cancer at low or moderate doses is in the range of 4–6 per cent per
sievert of radiation exposure. Research on the health effects of radiation is a con-
tinuing business and it is possible that this risk factor could change in the light of
new information.
ActivityUsing
9.2
information from the above passages:
1 What would be the additional lifetime risk of contracting fatal cancer for a
hypothetical person who lived at Poços de Caldas for one year?
2 What is the collective dose of the UK population of 60 million from radon exposure
and how many cases of fatal cancer would be expected annually from this hazard?
3 Identify an important assumption in making these estimates.
Feedback
1 The dose resulting from one year’s exposure at Poços de Caldas would be 250 msv.
Using UNSCEAR’s risk factor of 4–6 per cent risk per sievert of exposure, the
additional lifetime risk would be given by:
from (250 × 10−3) × 4/100 to (250 × 10−3) × 6/100
or from 0.01 to 0.015 (i.e. from 1–1.5%)
2 Taking the annual individual dose from radon to be 1.3 msv, the collective dose is
given by:
(1.3 × 10−3) × (60 × 106) = 78,000 man-sv
and the expected annual number of cases would be:
from (78,000 × 4/100) to (78,000 × 6/100)
or from about 3,000 to 5,000
3 These estimates, particularly for the UK example, are dealing with low doses and are
dependent on the validity of the LNT hypothesis discussed in Chapter 5. It is of interest
to note that if the calculations in 2 are correct, then as many people in the UK die from
exposure to background levels of radon as are killed in road traffic accidents or in the
workplace.
The aim of radiation protection is to provide a good standard of safety for people
exposed to ionizing radiation but without limiting its beneficial uses or disrupting
everyday life. In 1977, the ICRP issued the following recommendations for this
purpose:
The management of ionizing radiation 109
In many aspects of life, particularly those turning points where big decisions
have to be made, you will encounter the need to weigh up the advantages or
disadvantages of any particular course of action before deciding what to do. In
some professional circles the formal term for this procedure is ‘optimization’.
You have already come across it in Chapter 8 of this book, because cost–benefit
analysis is one method of optimizing decisions. In radiation protection, ALARA,
the ICRP concept that all radiation doses shall be kept As Low As Reasonably
Achievable, economic and social factors being taken into account, is consistent
with choosing the best course of action in given circumstances. Thus, in radiation
protection:
ActivityThe
9.3
following extract from Stokell et al. (1991) discusses the ALARA concept and its
origins. As you read it, consider:
1 Why did radiation protection switch from a system based purely upon exposure
limits to one incorporating optimization (ALARA)?
2 What were the driving forces behind the development of the ALARA principle?
3 What is the difference between deterministic effects and stochastic (probabilistic)
effects? Give an example of each.
4 Is the aim of radiation protection dose minimization?
material, incurred damage to tissue, but that at low doses no effects were observed. Based
on this, early systems of radiation protection were directed at keeping an individual’s dose
below dose limits that were defined with reference to the threshold for these effects.
In time it was realised that there were other health consequences from radiation
exposure, both somatic (in the exposed individual himself) and genetic (in his offspring),
that appeared to show no observable threshold dose. These effects are termed stochastic
or probabilistic, and it was assumed that an increased radiation dose implied an increased
likelihood of suffering one of these effects. It was also assumed that this relationship held
down to zero dose, i.e. even at very low levels of dose there remained a possibility,
however small, of sustaining one of these health effects . . .
Acknowledging that there was no completely ‘safe’ level of radiation exposure led to the
idea that the best one could do would be to reduce radiation exposures as far as possible.
However, radiation protection, like many other practices, is subject to the law of diminish-
ing returns – an initial expenditure on protection may result in a significant dose reduction,
but further expenditure produces progressively smaller reductions. Since there are never
infinite resources available to spend on protection, the question arises as to how far one
should go in reducing doses. Control has to be achieved on the basis of some sort of trade-
off between the reduction in radiation exposure and the cost of the protective measures
that could be taken. In other words, there is a need for some sort of optimisation of
radiation protection.
Feedback
1 The switch was made when it was discovered that for some effects, cancer and
hereditary consequences, radiation was a non-threshold agent.
2 The development of the ALARA principle is driven by two factors. These are the
absence of a threshold dose and the fact that resources for radiation protection are not
unlimited.
3 Living or working in a dusty environment which exceeds a certain threshold con-
centration will inevitably lead to a reduction in lung function, and this reduction
is likely to increase as dose increases. Such would be a deterministic relationship.
Asbestos workers may in this way suffer asbestosis and smokers may contract emphy-
sema. In contrast, both asbestos workers and smokers might contract cancer, though
this is not certain and can only be stated in terms of probabilities. Stochastic effects
have no threshold.
4 Radiation protection is not about dose minimization. It is about striking a balance
between dose limitation and resource allocation.
It is worth noting that although the concept of ALARA stems from the absence
of a demonstrable threshold for stochastic effects, that even if a threshold were
shown to exist, a trade-off between the benefits of dose reduction and costs of
achieving those reductions would still be required at levels above that threshold
(Figure 9.5).
The management of ionizing radiation 111
Figure 9.5 Even if there were a threshold (T) in the dose–response curve, the ALARA
principle would still apply at doses above the threshold
Source: Stokell et al. (1991)
The ICRP’s second principle requires you to decide how much effort should
reasonably be invested in reducing radiation exposure.
ActivityFrom
9.4
what you already know, can you outline a method for determining quantitatively
the levels of radiation protection which could be described as reasonably achievable in
any given circumstance?
Feedback
To answer this, look back at Chapter 8. If it were possible to put a monetary value on
the impairment to health resulting from radiation exposure (as was done for the value
of a life), the value of the health benefits of any exposure reductions could be calculated,
and these could then be compared with the costs of interventions used to bring them
about. Interventions satisfying the ALARA principle could then be identified as those
giving rise to potential Pareto improvements.
ActivityRead
9.5
the following extract based upon David Ball and Geoffrey Goats (1996) which
describes the approach in the UK to valuing the harmful effects of radiation exposures
and make notes of how it links up with Chapter 8.
Wales. The value of YOLL was found to be about 1.0 years per sievert. Thus, the implied
value of the man-sievert at the risk level of the WTP studies is about £30,000 to £60,000.
To calculate a base-line value of , it was necessary to bear in mind that the WTP studies
were carried out on populations facing an average risk of about 1 × 10−4 per annum. In
radiation terms this would be equivalent to a dose of a few mSv which, from Figure 9.6, is
associated with a risk aversion multiplier of about 5 to 7. Thus, the base-line value of the
man-Sievert which would apply at very low dose levels where the aversion multiplier was
close to unity would be in the range of £5,000 to £12,000.
Feedback
The NRPB’s approach is clearly consistent with using a welfare economics-based model
for health policy decision making. Such an approach would benefit from having recog-
nized monetary valuations of health detriments which result from the typical, low-level,
exposures that people receive, either in the environment or at work. Thus, they sought
to value the basic unit of radiation exposure, the man-sievert. First, NRPB used the
human capital (lost earnings) approach for valuing the man-sievert, but switched to
WTP when that went out of fashion. The Board did not have its own WTP survey data
and instead took a median value for a life from other WTP studies of non-radiological
risks. This was then crudely converted, without discounting, to a value of a life-year, so it
was then fairly straightforward to put a price on a man-sievert by using an estimate of
the average number of life-years lost per man-sievert of exposure. One obvious com-
plication is the use of the ‘risk aversion’ curve in Figure 9.6. NRPB felt that people are
more averse to risk, the higher the level of risk they are exposed to. They speculated
that the degree of risk aversion was as shown in Figure 9.6. They then considered that
most people who had answered the original WTP questions would have been influ-
enced by their personal level of risk from all sources. Typically this is taken to be about
1 in 10,000 risk of dying per year. They then deduced that this level of risk (using
UNSCEAR’s 4–6 per cent risk factor) was equivalent to a radiation exposure of about
2 millisv (msv), and from Figure 9.6 this can be seen to be equivalent to a risk aversion
factor of 5–7. NRPB were interested in the valuation of at much lower doses than this
(with an aversion factor of unity) so that a ‘baseline’ value could be calculated. This would
provide the minimum value of , from which all other values, relevant at higher doses,
could be calculated as required using Figure 9.6. The baseline value of was calculated
by dividing the value of at 2 msv (£30,000 to £60,000) by this factor of 5 to 7.
While the details of the above calculation are somewhat complicated, the key point
to note is that the ALARA approach is often closely connected with the use of CBA
which in turn is based in the theory of welfare economics. Also be aware that the
valuation of the man-sievert will, in addition to its dependence on dose, change
with time as GNP (gross national product) changes, may vary from place to place
depending on local GNP and other social factors, and also if methodologies or risk
factors change. Figure 9.7 shows the results of a survey of α values as used in differ-
ent countries in 2002 (ISOE 2003), all expressed in Euros per man-millisievert
(equivalent to α/1,000).
Figure 9.7 Alpha values adopted by regulatory agencies in different countries (*1997 survey)
Source: ISOE (2003)
The management of ionizing radiation 115
Dose limits
The third principle of the ICRP is concerned with ‘dose limits’. The aim of dose
limits is to set levels of exposure to ionizing radiation which should not be
exceeded for any individual.
ActivityReflect
9.6
upon why it is necessary to set dose limits when radiation exposures are
already constrained by the ALARA principle.
Feedback
ALARA is concerned with maximizing the overall welfare of a group of people. As with
Kaldor and Hicks’ modification of the Pareto principle, it is about potential Pareto
improvements. In these circumstances a change may make some individuals worse off
than others, yet still be seen as a good thing because the collective welfare of the group
is increased. The concept of dose limits is therefore introduced as a safety net to ensure
that no individual experiences risks which would, on ethical grounds, be deemed
unacceptable by society.
The setting of dose limits is only partially a scientific task because it relies upon
judgements. A widely used value for the dose limit for members of the public is 1
millisievert per year. This usually applies to all controlled sources, but not natural
radiation which, at a global average of 2.4 millisieverts per annum, is clearly higher.
For workers, a dose limit value of 20 millisieverts per year is commonly used. Where
do these values come from? Using the mid-point (5 per cent) of the UNSCEAR risk
factor of 4–6 per cent per sievert, it can be seen that the dose limit for workers is
equivalent to an annual risk of dying (from fatal cancer) of about:
(20 × 10−3) × 5% = 1/1000
Occupational risks of dying of 1 in 1000 per annum are fairly widely regarded as the
maximum risks that workers should be exposed to. Studies of the risks that workers
face in dangerous jobs, like mining, quarrying or deep sea fishing, generally show
that the levels of risk experienced are of this order and it is considered that this is
the maximum tolerable risk which workers should have to face. This then is the
justification for the dose limit of 20 mSv per annum for workers. It should be noted
that this by no means implies that workers can therefore be exposed to what some
people would regard as a high level of risk without further thought. This is because
the second principle, that all exposures shall be kept as low as reasonably achiev-
able (ALARA), economic and social factors being taken into account, is there to
continually drive down any risk to lower levels.
In the case of the public, the dose limit of 1 millisievert per annum is 20 times lower
than that for workers. The justification given for this is that the public is involun-
tarily exposed to these risks, whereas workers are paid and are aware of the risk.
116 Rational action and environmental health policy
Second, workers are generally fitter and healthier than the wider population and
may be more resilient.
An overview
Figure 9.8 sets out the components of the underlying philosophy of radiological
protection. It is possible to think of radiation dose, and its associated risk, as falling
into one of three domains. In Figure 9.8, the top domain of highest dose is deemed
to pose an unacceptable risk under any normal circumstances, and this region is
defined by dose limits. The central domain is one where risk is sufficiently high to
be of concern, but may be tolerated in exchange for benefits of activities or because
the cost and difficulty of control are too much. Where the correct balance of con-
trol lies is determined by application of the ALARA principle. Risks (doses) must be
reduced until as low as reasonably achievable. There is also potentially a third
domain of very low doses which are not of concern to people because the risks are
so small. You will read more about this in Chapter 10.
Summary
This chapter has introduced a basic framework against which decisions about
radiological protection can be made, taking account of the benefits of reducing
exposures to risk and the opportunity cost of doing this. The framework has two
essential ingredients: dose limits and ALARA. The concept of dose limits, or maxi-
mum tolerable risk, is there to protect all individuals from unreasonable exposure
to risk of harm. The second ingredient, ALARA, is an acronym for ‘all doses shall be
kept as low as reasonably achievable, economic and social factors being taken into
account’. In radiation protection circles, it is synonymous with ‘optimization’.
The ALARA principle is a direct consequence of the absence of a threshold for
health effects and the fact that resources are limited. Often decisions in radiation
protection can be made without recourse to complex calculation and are based on
experience, but a logical framework is always helpful in achieving uniformity. In
this chapter, cost–benefit analysis was used as a tool for identifying optimal
The management of ionizing radiation 117
control, but other techniques are available and will be introduced later. The ALARA
procedure is merely an aid to decision making and the final choice still remains the
responsibility of the decision maker.
References
Ball DJ and Goats G (1996). Risk management and consumer safety. International Journal for
Consumer Safety 3(3): 111–24.
Health Protection Agency Centre for Radiation, Chemical and Environmental Hazards (2005).
ICRP (1977) Recommendations of the International Commission on Radiological Protection,
ICRP Publication 26. Ann. ICRP 1(3). Oxford: Pergamon Press.
ISOE European Technical Centre (2003) Man-Sievert Monetary Value Survey (2002 update).
Information sheet No. 34. Paris: CEPN.
Stokell PJ, Croft JR, Lochard J and Lombard J (1991) Radiation protection: ALARA from Theory to
Practice. Luxembourg: Office for Official Publications of the Commission of the European
Communities.
UNEP (1985) Radiation: Doses, Effects, Risks. Nairobi: UNEP.
UNSCEAR (2000) Sources and Effects of Ionizing Radiation. New York: Report to the UN General
Assembly.
Environment and safety
10
Overview
In the previous chapter you were introduced to a framework for making health pol-
icy decisions about ionizing radiation. The framework was found to be based on risk
(or dose) limits, and exposure control through the process of managing risks accord-
ing to the principle of ALARA. In this chapter you will look at two other areas of
health policy: public and occupational safety from risk of injury, and environmental
health. The aim is to see if parallel approaches exist, and if so, the extent to which
there is a general framework for making environmental health policy decisions.
Learning objectives
Key terms
Best available technique not entailing excessive cost (BATNEEC) A pollution control
philosophy.
de manifestis When applied to risk, refers to levels that are manifestly intolerable.
de minimis When applied to risk, refers to levels that are considered trivial or negligible.
surge of interest. In practice, injuries rank among the leading causes of death and
burden of disease and occur in all regions and countries, affecting people of all ages
and income groups (WHO 2002). The hazards giving rise to injuries vary with age,
gender, income and region. For example, in the low- and middle-income regions of
the western Pacific, the leading injury-related causes of death are road traffic acci-
dents and interpersonal violence, while in the low- and middle-income countries
of Europe suicide and poisoning are more important.
An interesting question is perhaps why until now injuries have not received greater
attention in the public health sector, to which Judith Green (1997) provides a
fascinating response from a sociological perspective. The simple answer, though, is
that injuries have traditionally been regarded as random, unavoidable events –
accidents in the usual sense of the word – thus having little prospect of being
either understood or prevented. However, the epidemiology of accidents is now
well established and this shows that accidents appear not as unique misfortunes,
but as statistically predictable events with identifiable social, environmental,
psychological and biological risk factors (Green 1997). As a result of this shift in
perception, injuries and their health implications have increasingly attracted the
attention of decision makers worldwide and injury policy has found its place
on the public health agenda. The World Health Organization, for example, has
established a Department of Injury and Violence prevention.
ActivityWhile
10.1
reading the following passage from the World Health Organization (2002), con-
sider who should be involved in injury prevention programmes and what factors should
be taken into account in policy formulation.
When you have finished reading, make some notes in response to the following
questions:
1 Why do you think that WHO recommends involving non-government persons in
injury prevention?
2 Are prioritization and resource allocation recognized as important issues?
3 What factors might account for the uneven development of injury prevention
strategies around the world?
health, transport, justice or education, but ideally they should be developed in a multi-
sectoral fashion. It is also recommended that policy makers and planners take into account
at an early stage the human and financial requirements that will be necessary for their
implementation.
Most national injury prevention policies, strategies and/or plans of action currently in use
around the world originate in high-income countries. Few low- and middle-income coun-
tries have such policies, although more have been developed in recent years. Of those
which presently exist, some are comprehensive pertaining to all injury-related mortality
and morbidity, while others focus on a particular type of injury such as road traffic injuries
or violence-related injuries or a particular group of intended beneficiaries such as children,
youth or women. Much depends on the burden posed by these public health concerns in
the country and the country’s willingness and ability to recognize these as issues which
need to be addressed and to take action.
Feedback
1 Involving non-governmental parties is a modern trend in some societies and one
which we will consider more carefully in Chapter 17. There are three basic reasons for
it: (1) useful practical things might be learned by involving people outside of govern-
ment; (2) it is in the interests of democracy to involve people in policy making; and (3) if
people are invited to help in policy making, then they are more likely to accept the
policy, and support it, when it comes to implementation.
2 Once again, it can be seen that resource allocation, human or financial, is recognized
as an appropriate issue by the WHO, as it is by other national and international agen-
cies. Policy decisions, even about things as important as health and safety, cannot avoid
being scrutinized and prioritized.
3 There are many factors which might account for uneven development of injury
prevention strategies. They range from societal beliefs about the nature of accidents
and their preventability, to the agendas of professions and health protection agencies,
and the level of organization, type of infrastructure, and resources. Injury prevention
has only in the past few decades been recognized as a legitimate field of activity, and it is
therefore not surprising if some areas have yet to pick up the banner. Of course, they
might also have other more pressing concerns.
ActivityFigure
10.2
10.1 shows a number of interesting features. Each point drawn on the matrix
corresponds to a specific hazard, e.g. a slippery or uneven floor, a broken electrical
socket, a dangerous bend in a road, an unprotected drop, etc. The heavy line in Figure 10.1
can be compared with either the upper boundary of unacceptable risk or the lower
boundary of acceptable risk in Figure 9.8. Which is it?
Feedback
The line in question would have to correspond with the lower boundary of Figure 9.8
because if it were the upper boundary, the hazards above and to the right of the line
would normally require action to move them to lower risk and consequence, but the
implication in Figure 10.1 is that this is not necessarily the case, for example, if there is
limited scope for improvement. It is as if an achievability criterion, as in ALARA, is being
considered for the hazards shown.
The following edited passage from an article by David Ball and Geoffrey Goats
(1996) describes a framework, shown in Figure 10.2, and sometimes referred to as
the ‘Tolerability of Risk (ToR) framework’, against which occupational safety is
assessed in the United Kingdom. This approach is also seen as relevant in decision
making about risks to the public. The vertical axis in Figure 10.2 is the level of risk
of death by injury, with high risk at the top.
122 Rational action and environmental health policy
• whether a given risk is so great or the outcome so unacceptable that it must be refused
altogether (top zone);
• whether a risk is, or has been made, so small that no further precaution is necessary
(bottom zone);
• if a risk falls in the intermediate zone, that it has been reduced to the lowest level
practicable, bearing in mind the benefits arising from its acceptance and taking into
account the costs and difficulty of any further reduction.
Inherent within the scheme are a number of fundamental concepts. First, the idea of zero
risk has been rejected. Instead, the notion of tolerating risks in exchange for the benefits of
risky activities is introduced. Second, above a certain level a risk is regarded as intolerable
and cannot be justified under any ordinary circumstances. Third, below the intolerable risk
level an activity may take place provided that the associated risks have been reduced by all
available methods that are reasonably practicable, i.e. risks have been reduced until as low as
reasonably practicable (ALARP).
In considering whether further safety improvements are necessary to comply with the
ALARP criterion, formal cost–benefit techniques can be applied. This requires monetary
values to be placed on risk reductions.
At very low risk levels, referred to in Figure 10.2 as the ‘broadly acceptable region’, it may
not be worth allocating further resources provided there is confidence that these levels of
risk can be maintained in practice.
ActivityWhat
10.3
are the similarities and dissimilarities between this approach and that used in
radiological protection?
Feedback
They are comparable in a number of ways. The ALARA principle appears here as
ALARP. While there are some minor differences between these principles, they are not
relevant here. The principles are similar in that they require a balance to be made
between the benefits of an intervention, whether to reduce risk of injury or radiation
exposure, and the cost and difficulty of implementing the measures. Thus, ALARP and
ALARA are underpinned by techniques such as monetary evaluation of health states
and cost–benefit analysis for their formal implementation. The HSE’s framework also
sets upper and lower risk limits, analogous to the dose limits used in radiological
protection.
Risk limits
The risk limits relating to the upper and lower boundaries in Figure 10.2 come from
similar sources and by similar reasoning to that used in radiological protection.
Regarding the upper limit, the boundary between intolerable and just-tolerable
risk, the HSE observed that, broadly, an individual risk of death of 1 in 1,000
per annum is about the most that is ordinarily accepted under modern conditions
124 Rational action and environmental health policy
for workers in the UK and that it seemed reasonable to adopt this figure as the
dividing line between what is just tolerable and what is intolerable (HSE 1988). For
the public, a lower figure of 1 in 10,000 was proposed, largely because these risks are
involuntary from the public perspective.
As in the case of radiological protection, it could be argued that these are very high
risks to set as criteria for tolerability, but the point is that hazards must also comply
with the ALARP criterion. For hazards in the middle zone of Figure 10.2, all meas-
ures that are reasonably practicable must be implemented, and this will continue to
drive risks down, but not in a never-ending way that requires infinite resources.
So far as the level of individual risk which might be considered broadly acceptable
in Figure 10.2 is concerned, the HSE proposed that this could be taken as 1 in a
million per annum, for both workers and the public, since this would constitute
only a very small addition to the normal risks of life and was not a level of risk
which people ordinarily worried about or took action over. It is, for example,
roughly equivalent to the annual risk of being killed by a lightning strike.
International applicability
You have already noted that the approach adopted for radiological hazard man-
agement is international in scope through the participation of so many countries
in agencies like the UNSCEAR, ICRP and IAEA. The same can be said of the
broad details of the approach just described towards safety from accidents. Con-
sider, for example, the International Maritime Organization (IMO) and the SOLAS
(Safety of Life at Sea) Convention. IMO and SOLAS make use of the technique of
Formal Safety Assessment (FSA) to identify, by a rational and systematic process,
practicable control measures for implementation that increase safety and reduce
pollution risks on the oceans while giving good value for money to stakeholders.
Figure 10.3 shows the FSA methodology as described by the UK Maritime and
Coastguard Agency (2006), with its embedded use of cost–benefit analysis which,
in radiological protection, is called ‘optimization’.
Figure 10.3 The methodology of Formal Safety Assessment as used by the IMO
Source: Maritime and Coastguard Agency (2006)
Environment and safety 125
You have now encountered health policy frameworks in the areas of radiological
protection and injury prevention. Radiological hazards and hazards giving rise to
injuries both fall within the general domain of public health, but there are many
other areas too, including exposure to chemical and biological agents, natural or
man-made, in the environment. The question is, do similar frameworks for risk
management exist in these areas? The answer is, broadly, that they do. In fact, if
you read the literature on environmental pollution control, you will come across
terms such as BPEO (best practicable environmental option) and BATNEEC (best
available technique not entailing excessive cost), both concepts having similarities
to ALARA and ALARP.
The following edited extract from a document by David Kocher (1996) for the US
Department of Energy proposes a general framework for risk management of both
radionuclides and hazardous chemicals in the environment.
ActivityAs10.4
you read it, consider the following:
1 How do the concepts of de minimis and de manifestis risk fit in with the frameworks
which have been described for radiological protection and injury prevention?
2 Can it be inferred that there is, at least in outline, a consistent philosophy with
regard to health policy across the areas of radiological protection, hazardous
environmental chemicals, and injury prevention?
Table 10.1 General framework for categorizing risks from exposure to radionuclides and
hazardous chemicals in the environment
Severity of riska Characterization of risk Approach to risk reduction
De minimis Risks are so low that they are Action to reduce risk generally is
considered trivial or negligible. unwarranted.
Intermediate Risks are between de minimis and Feasibility of risk reduction generally
de manifestis levels. must be considered, but action to
reduce risk is required only if risks
are above levels judged as low as
reasonably achievable (ALARA).
De manifestis Risks are so high that they are Action to reduce risk generally is
considered manifestly intolerable. required, regardless of cost or any
other considerations.
Source: Kocher (1996)
Note: a Severity of risk increases from top to bottom of table.
unacceptable only if they are manifestly intolerable or they are below manifestly intolerable
levels but are not ALARA.
Feedback
1 The de minimis and de manifestis concepts fit neatly with the dose limit and risk limit
concepts of radiological protection and safety from injury as shown in Figures 9.8 and
10.2. So far, though, the numerical values of de minimis and de manifestis risk have not
been defined and we will consider that next. A point to note is that there is a lot of
sensitivity about de minimis risk in particular, because sometimes even very small risks
are perceived as undesirable and so the use of words like ‘acceptable risk’ may be
challenged. For example, a typical response might be ‘acceptable to whom?’ You will
learn more about this in Chapter 12, but for the time being, remember it is a sensitive
matter. As the engineer Samuel Florman said of risk assessment, but which has wider
resonance, ‘Responsible experts in the field, however, consistently warn that risk
assessment is a delicate tool that needs to be applied sparingly, not a machete to be
flailed against the supposedly overgrown regulatory jungle’ (Florman 1987).
2 It can be inferred that the vestiges of a consistent philosophy exist in the form of risk
or dose limits and concepts like ALARA and ALARP which have a great deal in com-
mon. What differs most, perhaps, is the extent to which the existence of these concepts
is recognized and understood in different professions, areas of application, and by the
affected public.
As noted above, although the utility of de minimis and de manifestis risk levels for
hazardous substances has been identified, the actual corresponding risk levels so far
have not been defined numerically. In fact, you have to dig quite deeply to discover
what they might be. To this effect, in 1987, Curtis Travis and colleagues conducted
a review of a large number of US federal regulatory decisions relating to the man-
agement of environmental carcinogens with a view to discovering if a consistent
pattern existed in terms of those risk levels which were seen as requiring, or not
requiring, regulatory action.
ActivityWith
10.5
reference to Figure 8.3, how would you describe the approach used by Travis and
colleagues to discover society’s position with regard to risks of environmental
carcinogens?
Feedback
This is an example of a willingness-to-pay (WTP) revealed preference study based on
regulatory/institutional decisions.
128 Rational action and environmental health policy
Travis et al. considered three measures of risk in their analysis. These were: (1)
individual risk defined as the upper limit estimate of the probability that the most
highly exposed individual in a population would develop cancer as a result of a
lifetime of exposure; (2) the size of the population exposed; and (3) the population
risk defined as the upper limit estimate of the number of additional annual inci-
dences of cancer in the exposed population. Figure 10.4 shows the raw results, in
terms of pre-regulatory levels of maximum individual risk, as obtained by Travis et
al. (1987).
Figure 10.4 Pre-regulatory levels of maximum individual lifetime risk reviewed by federal
agencies. = agencies did not act to reduce public risk; = agencies acted to reduce public
risk
Source: Travis et al. (1987)
ActivityExamine
10.6
Figure 10.4 carefully. What inferences can be drawn about the actions of
regulators in response to different individual risk levels?
Feedback
If you think quite carefully about it, there are two clear patterns in Figure 10.4. One is that
for every chemical with an individual chronic risk of developing cancer above 4 × 10−3
(four chances in a thousand), a regulatory action was taken. Second, with one exception,
no action was taken to reduce individual lifetime risk levels that were below 1 in 10−6.
Travis et al. also examined the data to see if the size of the population exposed had
an effect upon the level at which regulatory action was taken. Figure 10.5 shows the
results. Note that population risk has been calculated by multiplying individual
risk by population exposed and is expressed as cancer deaths per year.
ActivityThe
10.7
following passage extracted from Travis et al. (1987) interprets the data in Figure
10.5. What does this tell you about the actual levels of de manifestis and de minimis risk
for small populations, and how is the appropriateness of regulation for risks between
these boundaries assessed?
Environment and safety 129
Key: 䊊 Agencies did not act to reduce public risk; 䊉Agencies acted to reduce public risk.
Figure 10.5 The effect of individual and population risk on whether or not risk posed by a
chemical carcinogen in the environment is regulated or not
Source: Travis et al. (1987)
Line A of Figure 10.5 defines the de manifestis level: above this line, federal agencies always
acted to reduce risk. For exposures resulting in small-population risk, the de manifestis level
is approximately 4 × 10−3. As population risk approaches 250 cancer deaths (which would
occur in a population of 230 million as in the USA) the de manifestis level drops to about
3 × 10−4. Line B shows the de minimis level. Below this line, no action has ever been taken to
reduce risk. Line B indicates that for small-population effects, regulatory action was never
taken for individual risk levels below 10−4. For effects resulting from exposure of the entire
US population, the level of acceptable risk drops to 10−6. Line C is the area beyond which
no data can fall. It is determined by the maximum population which can be exposed,
230 million in this case.
Figure 10.5 raises two questions. First, what justification is given by regulatory agencies for
not regulating chemicals in the de minimis category of risk? Second, what justification is
given for regulatory decisions involving chemicals in the region between the de manifestis
and de minimis levels? The primary answer given to the first question is insignificant popula-
tion risk. In relation to the second, analysis of regulatory decisions involving chemicals in
the region between the de manifestis and de minimis levels indicates that cost-effectiveness
is the primary determinant of regulation . . .
In the region between the de manifestis and de minimis levels, substances with risk reduction
costs of less than $2 million per life saved were regulated; substances that cost more were
not regulated. This conclusion is based on limited data, but it is consistent with EPA
130 Rational action and environmental health policy
guidance suggesting that regulation is warranted if the cost per life saved does not exceed
$1.5 million.
Feedback
Travis et al. conclude that for small populations at risk the de minimis risk level is at
about 10−4 lifetime risk of dying from cancer. If you consider that most people live for
~50 to 80 years, this is equivalent to an annual risk of from 10−5 to 10−6, which is broadly
consistent with the figures proposed for the Tolerability of Risk framework for safety
from injury. Likewise, for small exposed populations, the de manifestis risk level is
equivalent to ~10−4 per annum which is the maximum value assigned to public risks in
the ToR framework. The appropriateness of taking action at intermediate risk levels
appears to be judged by examining the cost per life saved and comparing against
a yardstick of $1.5 million (at 1987 costs). This is clearly tantamount to the use of
cost–benefit analysis and is consistent with the philosophy of welfare economics.
The following passage by Kocher (1996) describes how the US Environmental Pro-
tection Agency (EPA) establishes RfDs for different chemicals. You will recognize
the approach from your reading of Chapter 5.
subpopulations (e.g. children and the elderly); a factor of 10 when extrapolating from
animals to humans to account for the possible interspecies variability in susceptibility; a
factor of 10 when a NOAEL is derived from a study involving sub-chronic exposures,
rather than longer-term chronic exposures; and a factor of 10 when using a LOAEL rather
than a NOAEL to account for the uncertainty in extrapolating from LOAELs to NOAELs.
In addition, the EPA may apply a modifying factor to account for any other uncertainties
and judgmental factors not addressed in the safety and uncertainty factors described
above.
Thus, the RfD generally is intended to be well below any threshold for adverse health
effects. In particular, when the RfD is based on animal studies, which usually is the case,
its value generally is set at least a factor of 100 and sometimes more than a factor of
1000 below the threshold, as represented by the NOAEL or LOAEL.
There are two considerations indicating that RfDs are properly interpreted as upper
bounds on de minimis doses for noncarcinogenic hazardous chemicals. The first is the use
of large safety and uncertainty factors in deriving RfDs from observed thresholds for
adverse health effects, as described above, which again are intended to ensure that doses
to individual members of the public would be well below any thresholds. Indeed, because of
the way RfDs are derived, there should be no evidence that doses somewhat above an RfD
would cause any adverse health effects.
The second consideration is that RfDs are presumed to be sufficiently far below any
thresholds for adverse health effects that action to reduce doses at levels below RfDs
generally is not required by the EPA. That is, RfDs are used to distinguish between doses
that are trivial and, thus, require no further consideration and doses that are sufficiently
high that consideration must be given to the feasibility of dose reduction. However, as is the
case with the upper bound on de minimis risk for carcinogens, reduction of doses above
RfDs generally is required only to the extent practicable.
As a cautionary note, there are suggestions that exposures to toxic substances at
RfD levels are not necessarily associated with minimal appreciable risk after lifetime
inhalation or ingestion (e.g. Castorina and Woodruff 2003), despite the use of large
safety margins.
Summary
Various systems are used to help policy makers make decisions about health, safety
and environmental protection on a consistent basis. These systems have much in
common, relying as they do on science and economics, and in this sense they are
sometimes referred to as ‘rational’ models. They are used worldwide, but are none-
theless not widely known, let alone understood outside of professional circles. The
systems all aim to maximize the use of resources in the interest of the public good,
while also providing safeguards so that distributional inequities (unfair risk
apportionments) are, to some extent at least, contained.
References
Ball D and Goats G (1996). Risk management and consumer safety. International Journal for
Consumer Safety 3(3): 111–24.
Castorina R and Woodruff T (2003) Assessment of potential risk levels associated with US EPA
reference values. Environmental Health Perspectives 111(10): 1318–25.
132 Rational action and environmental health policy
Overview
Now that you have studied, in Sections 2 and 3, the dominant paradigm of
environmental health policy making, it is necessary to consider some of the objec-
tions and counter-arguments. Policy making, by its nature, is controversial, and the
best way to deal with this is, first, to understand why.
This section starts with a consideration of the now familiar policy tools of environ-
mental risk assessment, Pareto optimization, and cost–benefit analysis, and how
they are regarded by those who are familiar with them. Arguments and counter-
arguments will be examined. You will find also that sociology actually provides a
number of quite different perspectives on health risk, besides the (rational) model
explored so far, which have different implications for policy formulation. In
Chapters 12 and 13 you will look more closely at two of these, notably, the
psychometric paradigm and cultural theory.
Learning objectives
Key terms
Cultural theory A social theory that proposes that social responses to risks are determined by
cultural belief patterns and not objective facts about risk.
Precautionary principle A principle that advocates the use of ‘prudent’ social policy in the
absence of empirical evidence in an attempt to solve a problem.
Psychometric paradigm An approach to understanding public attitudes to different risks based
on the use of quantitative techniques.
Rational actor paradigm (RAP) A sociological theory of human behaviour based on people
acting as self-seeking individuals and maximizing their personal utility.
136 Beyond the rational action approach
So far in this book you have read about an approach to environmental health
policy which is anchored in risk assessment, potential Pareto optimization (and
hence neo-classical welfare economics), and calls upon allied techniques such as
cost–benefit analysis even in applications involving public health. This is a particu-
lar approach which some theorists and sociologists describe as the rational actor
paradigm (RAP). Most people acknowledge that RAP is a powerful and in many
cases socially beneficial tool, but it is certainly not without its critics and it is at the
very least important to understand their concerns.
What exactly is meant by RAP? RAP is a way of thinking that can be traced
back to classical Greece and which gained in influence during the Italian Renais-
sance and the period in Europe (mainly in the eighteenth century) known as
the Enlightenment. At the centre of this essentially Western world-view is the idea
that humans are rational organisms and that the social world can be explained in
terms of the interaction of humans as atomistic entities, rather like the natural
world can be explained by the interaction of atoms. This world-view sees humans
as rational beings motivated by pure self-interest and consciously evaluating alter-
native courses of action (Jaeger et al. 2001). Welfare economics, including
willingness-to-pay methodologies, is reliant upon this assumption. Essentially, RAP
is about maximization, or, as you have also discovered, ‘optimization’.
However, sociologists, and others, argue that there is far more to life than that. It is
different and more complicated. There now follows two abridged passages by lead-
ing sociologists Carlo Jaeger and colleagues, and Michiel Schwartz and Michael
Thompson, which identify the nature of some of these concerns.
ActivityWhile
11.1
reading these extracts, think of what they might mean for the so-called RAP
which you have studied in earlier chapters. In particular, consider the following questions:
1 What is the main point being made by each of the authors?
2 What are the implications for policy making?
other individuals belong to a class of social phenomena that do not fit neatly into the
iron rule of RAP theory. Individuals may behave in accordance with the means–end
optimization process of RAP some of the time, but certainly not all of the time ( Jaeger
et al. 2001).
All policy debates revolve around people and ideas. Poverty, for instance, has to do with
some people’s lack of command over resources relative to their needs, and poverty policy
has to do with the ideas of distributional justice that define a more desirable state of affairs
(the policy goal) and with the ideas both of the processes by which poverty comes about
and of the ways we can intervene in those processes to modify the outcomes (the instru-
mental means towards the policy goal). But some policy debates, the conventional wisdom
tells us, revolve around things as well (nuclear power stations, chemical waste, drinking
water quality, and so on), and it is this quality of ‘thingness’ that has been used to separate
out technological decision making as a distinct kind of decision making, one in which the
debate revolves around a technical and factual core that simply does not exist in other
kinds of decision making.
It is with this starting point that we take issue. There is, we insist, no policy debate that
is devoid of things, though, of course, some are more thing-ridden than others. Poverty
policy, for instance, is not usually considered a part of technological decision making yet it
is nevertheless much involved with things as well as with people and ideas. Nor are the
facts concerning those things, and the uncertainties that often surround those facts, ever
merely technical. They are every bit as value-driven as are any of the other interactions
within this triangular interplay of people, ideas and things. Where others have assumed the
interplay of just people and ideas to be the natural habitat of policy, and have then had to
designate a special category of decision making for those environments where things are
involved as well, we begin by assuming that this ‘special case’ is the normal (indeed, the
inescapable) state of affairs, and then set out to explore the continuum of variation in
policy debates as the relative contributions of these three essential elements – people,
ideas and things – vary (Schwartz and Thompson 1990).
Feedback
1. The main point being made by Carlo Jaeger et al. stems from the fact that RAP
emerges from a theoretical basis which assumes that people behave in particular
ways for particular reasons, but that this is only partially reflective of the truth.
People are more than profit or utility maximizers. They aspire to (nobler) things
which (for all we know) may have been instrumental in the survival of the human
race.
2. It follows that, if the theoretical background is faulty, then a policy process
derived from that theory may not reflect human aspirations. Were this so, it
could spell trouble.
Michiel Schwartz and Michael Thompson are also concerned that policy debates
which involve the more easily recognizable technical issues have, for no good rea-
son, been shunted off into a distinct kind of decision making process (which we
may identify with RAP). This process aspires not to be value-driven by relying
upon facts, welfare economics, PPI and the like, but all of these actually involve
138 Beyond the rational action approach
values in some way or another. For example, the issue of distributional justice is not
a part of neo-classical welfare economics, having been delegated by economists to
philosophers and politicians, but the mere act of omitting it implies the making of
a value decision.
This is not to say that these authors are necessarily antagonistic to the RAP
approach, rather, that it has strengths and it has weaknesses too. Environmental
health policy makers will want to understand this very clearly.
So what, then, are the main categories of criticism overall which have been levelled
at risk assessment and the associated decision techniques as a tool for environ-
mental health policy? The following extract from a paper by environmental scien-
tist Ken Sexton (2000) provides a concise and useful summary under six headings,
with responses.
Procedural critique
Whether it is more rational, the process of relying exclusively on expert judgment to
evaluate risks is not fair because citizens and communities have a right to participate in
decisions directly affecting their health and well being. Response: it does not have to be one
way or the other. An integrated approach, which involves the public in identifying and
evaluating risks, is emerging as a middle-of-the-road alternative.
In reading the above extract, you will have discovered reference to two drivers
for change which seek to turn RAP, and traditional risk assessment, into forms
more universally acceptable to policy making. These are the reference to ‘know-
ledge and dread’ as dimensions of risk which must be considered, and to the
desirability of public participation in health risk decision making. There is also
reference to an alternative paradigm – the Precautionary Principle. In the remaining
chapters you will read more about these developments. First, though, as a means of
injecting some order into the process, it is helpful to consider a classification, by
the sociologist Ortwin Renn, of the major sociological perspectives on risk.
Alternatives to RAP
corner, and there are clearly a number of other models on offer also shown in
the diagram. You will be reading about two of these in particular, the psychomet-
ric paradigm and cultural theory, in the following chapters. It is important to
understand the meaning of the dimensions. This will become clearer when you
have read about the other models, but a short explanation will be given here.
The horizontal axis ranges from individualistic to structural. This refers to the base
unit of the model. In the case of RAP, this is an atomistic model for which the base
unit is the individual. According to this model, society is made up of a myriad of
individuals who independently try to maximize their personal utility. For this rea-
son, RAP is located on the left of Figure 11.1. However, other theories take a differ-
ent perspective. People are seen not to act individually but as part of a social group,
a subculture or a society. Theories, like neo-Marxism and cultural theory, which
explain risk behaviour from this perspective, are located on the right.
The vertical axis ranges from objective to constructivist. Whereas the objectivist
concept implies that risk is an objective, measurable entity, which can be system-
atically prioritized, the constructivist concept denies this. Instead, it sees risks as
‘socially constructed’. The social construction idea is based on the observation that
people somehow agree to ignore most of the potential dangers around them, but
interact so as to concentrate only on risks which they select for their attention for a
variety of reasons (Douglas and Wildavsky 1983), even though these may carry a
minimal actual risk. On this dimension, RAP clearly belongs at the bottom of this
axis, being anchored in attempts to quantify risk, and hence its overall position is
in the bottom-left corner of Figure 11.1.
In 1996, a group of leading American economists spoke out in defence of the use of
benefit–cost analysis in policy decisions affecting the environment, health and
safety (Arrow et al. 1996). They noted that the use of benefit–cost analysis in the
fields of environmental protection, health and safety was controversial and sought
to develop a consensus by setting out principles in a straightforward fashion. They
advise that their pamphlet should be required reading for policy makers who wish
to understand both the strengths and limitations of benefit–cost analysis in the
development, design and implementation of policy.
ActivityThe
11.2
key principles, from the perspective of environmental health policy, set out by
Kenneth Arrow and colleagues (1996) are summarized below in two parts (some
minor modifications have been made, but only to adapt them for the purposes of this
book). In reading them, consider and make notes on the extent to which the principles
address and deal with the criticisms identified earlier in this chapter.
An introduction to alternative theories of risk 141
Arrow’s principles
1 Guidance for decision makers on using economic analysis to evaluate proposed policies
(a) A benefit–cost analysis is a useful way of organizing a comparison of the favourable
and unfavourable effects of proposed policies.
(b) Economic analyses can be useful in designing regulatory strategies that achieve a
desired goal at the lowest possible cost.
(c) Benefit–cost analysis should be required for all major regulatory (or policy) decisions.
(d) Policy makers should not be bound by a strict benefit–cost test, but should be
required to consider available benefit–cost analyses. For regulations (or policies)
whose expected costs far exceed expected benefits, a clear justification should be
required.
2 Suggestions for improving the quality of economic analysis used in decision making
(a) While benefit–cost analysis should focus primarily on the overall relationship between
benefits and costs, a good benefit–cost analysis will identify important distributional
consequences of a policy.
(b) It is important to identify the incremental costs and benefits associated with different
policies.
(c) Benefits and costs of proposed policies should be quantified wherever possible. Best
estimates should be presented along with a description of uncertainties.
(d) Not all impacts of a decision can be quantified or expressed in dollar terms. Care
should be taken to assure that quantitative factors do not dominate important qualita-
tive factors in decision making.
(e) The more external review analyses receive, the better they are likely to be.
(f) A core set of economic assumptions should be used in calculating benefits and
costs associated with environmental, health, and safety policies. Key variables include
the discount rate, the value of reducing risks of dying and accidents, and the value
associated with other improvements in health.
(g) Information should be presented clearly and succinctly. Transparency is necessary if
benefit–cost analysis is to inform decision making.
(h) Whenever possible, values for monetizing benefits and costs should be based on
trade-offs that individuals would make, either directly or, as is often the case, indirectly
in labor, housing, or other markets.
(i) Given uncertainties in identifying the correct discount rate, it is appropriate to employ
a range of rates.
Feedback
Principles 1(a) to 1(c) maintain the status quo, but 1(d) seemingly goes some way to
addressing the critiques. Thus, it says, benefit–cost analysis (BCA) should be used, but
should not be absolutely binding. And if the BCA shows a proposed policy to have a very
poor benefit to cost ratio, then a good justification would be required if it were being
recommended for implementation. This appears reasonable.
Principle 2(a) addresses head-on the distributional issue and is a clear signal that this
is important. 2(c) and 2(i) are important in that they stress the need to identify
uncertainties, something which has not been a strong point in economics, and test
their effect upon policy options. 2(d) is very important regarding the critiques – many
concerns that people have about policies will relate to aspects that can only be
142 Beyond the rational action approach
described qualitatively. There has long been a fear that analyses that include quantita-
tive and qualitative factors will tend to devalue the qualitative ones. Whether this can
be achieved in practice, however, remains to be seen. 2(e) and 2(g) are helpful in that
they potentially open up the analysis so that there can be wider scrutiny. This is a
small step in the direction of the opposition. Principles 2(f) and 2(h), however, which
imply adherence to contingent valuation, look set to continue to be regions of
contention.
Summary
References
Arrow KJ et al. (1996) Benefit–Cost Analysis in Environmental, Health, and Safety Regulation: A
Statement of Principles. Annapolis: The Annapolis Center.
Douglas M and Wildavsky A (1983) Risk and Culture. Berkeley, CA: University of California
Press.
Jaeger CC, Renn O and Rosa EA (2001) Risk, Uncertainty and Rational Action. London:
Earthscan.
Renn O (2005) personal communication. An earlier version appeared in Krimsky S and
Golding G (eds) (1992), Social Theories of Risk, chapter 3. Westport, CT: Praeger.
Schwartz M and Thompson M (1990) Divided We Stand: Redefining Politics, Technology and
Social Choice. New York: Harvester Wheatsheaf.
Sexton K (2000) Socioeconomic and racial disparities in environmental health: is risk
assessment part of the problem or part of the solution? Human and Ecological Risk
Assessment 6(4): 561–74.
Risk perception and the
12 psychometric paradigm
Overview
Learning objectives
Key terms
Affect The automatic feeling of goodness or badness generated in response to some event.
Experiential learning Learning from experience.
Psychometrics The science of psychological measurement.
Risk aversion Here, a precautionary attitude towards risk taking.
Stakeholder An individual or group with a substantive interest in an issue (i.e. interest group),
including those with some role in making a decision or its execution.
144 Beyond the rational action approach
In the early days, by which is meant until some 30 years ago, it was common
practice for risk decisions to be made by specialists who frequently saw little need
to engage with those who would be affected. As time passed, this normally well-
meaning but autocratic approach ran into increasing difficulty. Decisions made
according to the so-called rules of ‘rational decision making’ were increasingly
challenged by parties who had an interest in policy choices.
Table 12.1, based on information from the Centre for Environmental Justice in Sri
Lanka in 2006, illustrates the breadth of issues which can give rise to controversy
in just one part of the world. There are numerous other examples worldwide
where the public, employees, and other stakeholders have felt it necessary to raise
issues and even challenge risk decisions by professionals or governments. The con-
sequences can be important – they might derail a programme aimed at improving
public health, incur significant costs, or deter innovation. This is not to imply that
these challenges to authority are necessarily wrong or unjustified. Without needing
to take sides, it is clear that perceptions of risk do have immense consequences for
society, just as they do for individuals. Environmental health policy makers there-
fore need to understand why this is so in order to have any prospect of resolving
these issues.
Table 12.1 An example of environmental and health issues causing current concern in one
region of the world, Sri Lanka
The issue Nature of the concern
100 metre no construction zone in the coastal belt Public unrest
(a tsunami counter-measure)
The Sri Lankan civil war No studies of the environmental
consequences
Genetically modified food Uncertainty over risks and benefits
Sulphur dioxide level in Columbo Effect on sensitive groups
Menik Ganga River diversion Water reduction for Yala and
Kataragama villages
Persistent organic pollutants: (aldrin, dieldrin, DDT, Cancer and other health effects
chlordane, endrin, heptachlor, mirex, toxaphene,
hexachlorobenzene, PCBs, dioxins, furans)
Incineration Waste of resources and contamination
Uncontrolled logging Conservation
Human–elephant conflict leading to 150 to 200 Public concern
elephant deaths per year
900 MW coal-fired power station to be built Pollution
at Kalpitiya
Columbo to Kandy highway Will relocate many families and destroy
paddy fields
Source: Based on the Sri Lanka Centre for Environmental Justice (2006)
Risk perception and the psychometric paradigm 145
ActivityIdentify
12.1
from your own experience (or from what you have seen in your local media),
two issues where different perceptions of environmental health risk have resulted in
controversy. What reasons can you think of that might underlie these conflicts?
Feedback
You should have no difficulty in responding to the first question because the range of
possibilities is immense. Of the second question, it can be said that controversies may
be ignited by anything from different objectives, values, or perceptions of risk held by
affected groups, and perhaps least likely but still plausible, disputes over technical
assessments of risk. Or it may be that there has been a lack of political process in the
form of consultation and communication.
In 1969, the American scientist Chauncey Starr authored what is now seen as a
landmark paper that compared the benefits of selected technological activities with
one particular cost, namely, the cost of accidental deaths associated with the activ-
ity. His aim was to find if there was a relationship between the level of risk of an
activity which society would tolerate and its benefits. He used as examples a num-
ber of accepted activities, such as the use of motor vehicles, electricity generation,
etc. (Starr 1969). Referring back to Figure 8.3, you should be able to identify this
as an example of the ‘revealed preference – individual choice – consumer market’
method.
Starr concluded that the public is willing to accept ‘voluntary’ risks that are roughly
1,000 times greater than ‘involuntary’ risks giving the same benefit. He also con-
cluded that the acceptability of risk appeared to depend on the benefits (real or
imagined) of the activity. These findings seem eminently sensible. Yet these factors
might not have been acknowledged in a strictly ‘rational’ (as defined earlier)
decision making process.
Further influential research of a quite different nature was reported by Tversky and
Kahneman in 1974, who found that when people are confronted with large
amounts of information on some topic and have to form an opinion, they use
mental short cuts, known as ‘cognitive heuristics’. These help simplify information
and reduce the amount of mental effort and time required to make a decision.
Every time you cross a road you have to make a risk decision almost instant-
aneously. The use of heuristics focuses attention on the most prominent issue of
the moment. A second factor they discovered was that of ‘availability’. That is, if
you have an available image in your mind which relates to the issue in hand, it will
influence your decision. Available images can be very powerful and long-lasting –
see Figure 12.1.
146 Beyond the rational action approach
Figure 12.1 Anyone witnessing this scene of people fleeing for their lives from a tsunami
would be likely to retain a vivid image in their memory.
Activity112.2
What available images do you personally recall that influence your decisions?
2 What available images can you think of which have influenced societal decisions?
3 Is the effect of these available images on decisions beneficial?
Feedback
1 Available images come in many shapes and forms, depending upon your experiences.
They may be of happy associations, or of frightening events, as in Figure 12.1.
2 Images of the December 2005 tsunami have encouraged society to set up a seismic
monitoring network in the Indian Ocean, even though these events are exceedingly
rare.
3 Psychologists believe that available images are one of the ways in which human beings
have been able to make decisions and to survive. However, they can be misleading and
where important decisions are being made, it would be inadvisable to be over-reliant
upon them. Analysis, too, should be used.
In the 1970s, Sarah Lichtenstein and her co-workers decided to take forward Starr’s
work using quantitative expressed preference techniques. They did this by asking
ordinary people to estimate the risk of dying from a wide range of causes (e.g. heart
disease, cancer, botulism, tornadoes, etc.) and comparing their answers against
actual risk as deduced from official statistics. They also put the same questions to
experts.
Risk perception and the psychometric paradigm 147
ordinary people to estimate the risk of dying from a wide range of causes (e.g. heart
disease, cancer, botulism, tornadoes, etc.) and comparing their answers against
actual risk as deduced from official statistics. They also put the same questions to
experts.
The results of the first part of this work are found in Figure 12.2. This shows (verti-
cal axis) the annual number of deaths in the USA, where the study was made, as
estimated by a sample of Americans, versus actual statistical estimates (horizontal
axis). The important thing about this graph is that if the judgements of probability
were accurate, they would fall on the straight line at 45 degrees to each axis, but
they don’t. It looks instead as if the frequency of rare events is being overestimated
and that of common events underestimated. The reason why the probability of rare
events is overestimated might in part be down to the ‘availability heuristic’ if
images of these events have lodged in the memory.
The results of the second part of this study, the comparison of public assessments
of risk with those of experts, are summarized in Figure 12.3. It can be seen (from
the slopes of the lines) that the expert ratings are more closely associated with
the actual risk of death. Note that it is not said that the expert ratings are more
‘accurate’. The reason for this will become apparent shortly.
Figure 12.3 The top graph compares expert perceived ratings of risk against technical
estimates and the bottom graph does the same for lay people. Note that the experts’ ratings
are more closely associated with actual fatality rates.
Source: Slovic (2000)
ActivityThe
12.3
above difference between expert and lay estimations of risk is often quoted and
has been very influential. It has led some people to propose that the lay public are
rather ignorant about risk and that what is needed is education so that they think like
experts. What do you think of this proposition?
Feedback
This is a controversial area which we will discuss in more detail later. There is truth in
the proposition that experts are more knowledgeable about the numerical risks posed
by hazards than the public. But on the other hand, the public may have legitimate
concerns which are not encapsulated by the experts’ opinions where these are based
solely upon considerations of risk magnitude.
Risk perception and the psychometric paradigm 149
The above question, over the difference between expert and lay perceptions of risk,
was reframed by the American psychologist Paul Slovic and co-workers. What, they
asked, do non-experts actually mean when they say that some situation or some
activity is risky? Further research showed that whereas experts, in rating risks, were
primarily focusing on associated fatality rates, lay people had a much wider inter-
pretation of risk. This was demonstrated by asking lay people to rate hazards against
a number of qualitative dimensions besides pure risk of harm. Figure 12.4 shows the
results for one pair of hazards, both being sources of ionizing radiation, and hence
posing a risk of cancer. What is clear is that the lay perceptions of these two hazards
are quite different in terms of these qualitative characteristics. If lay people were
incorporating factors of this kind into their overall rating of hazards, this could
explain the differences between public and expert opinion in Figure 12.3.
Figure 12.4 Qualitative characteristics of perceived risk for two hazards giving rise to ionizing
radiation
Note: Each profile is based on subjective ratings against nine dimensions on a seven-point
scale.
Source: Slovic (2000)
ActivityDo12.4your own rating on a seven-point scale of two quite different hazards with which
you are familiar against the following five qualitative characteristics:
1 Voluntary (1) to involuntary (7).
2 Familiar (1) to unfamiliar (7).
3 Not fatal (1) to fatal (7).
4 You know when you are exposed (1) to you don’t know (7).
5 It is understood by science (1) to it is not understood (7).
What conclusion do you draw about the importance of these characteristics in your
overall assessment of the relative risk?
150 Beyond the rational action approach
Feedback
If you allow yourself to think about the hazards in general terms you will probably find,
if the hazards are sufficiently different, that the pattern of scoring differs significantly
against the five criteria. Whether or not this influences your decision about their
overall relative importance will depend upon your perspective. For many people,
factors like these are influential.
Paul Slovic and his co-workers have greatly extended their work since those early
days and it has by now been replicated in countries all around the world. To under-
stand why some hazards are perceived as more risky than others they employed the
‘psychometric paradigm’ where psychometrics is a term referring to the measure-
ment of mental attitudes. This used a questionnaire which asked participants to
rate a selection of hazards against the following 15 characteristics:
The methodology is in fact very similar to the one you used in Activity 12.4. A
typical question was: ‘How do you rate this hazard (e.g. chemical contamination
of river water) in terms of its voluntariness? Give a score from 1 to 7, where 1
means it is entirely voluntary to 7 meaning it is entirely involuntary.’ Partici-
pants were also asked to rate the overall perceived risk of each hazard. The term
‘risk’ was left deliberately vague, so that participants could interpret it as they
wished.
Slovic and co-workers found that each hazard had its own unique profile when
scored against these attributes (such as X-rays and nuclear power in Figure 12.4).
Because it is not possible to display these results in 15 dimensions, a statistical
method called Factor Analysis was used to simplify the results. It turned out that
many of the qualitative characteristics were closely inter-related. For example, a
hazard such as riding a bicycle was seen as both voluntary and individual, and one
such as genetic modification of crops was regarded as a new risk and unknown to
science. Factor analysis showed that there were two dominant composite factors in
Risk perception and the psychometric paradigm 151
Recently psychologists have begun to pursue a further line of inquiry into how we
make choices. This is known as ‘the affect heuristic’, where ‘affect’ refers to the
specific quality of goodness or badness which you experience as a feeling (with or
without consciousness) when presented with some object or situation. Affective
responses occur very quickly and automatically. Images like those in Figure 12.1,
for example, automatically generate affective responses, some positive and some
negative.
In Section 3 of this book we discussed rationalistic decision processes, based
on cognitive thought processes, rather than upon affect-driven processes. Note
that the dictionary defines cognition as ‘conceiving of something by mental pro-
cesses as distinct from emotion and volition’. Both of these processes occur in
the brain, so it’s as if the brain has two ways of arriving at decisions (no doubt
this will turn out to be an over-simplification). Most of the risk decisions we
have to make have to be made quickly, hence relying on what might be called
an instinctive reaction. This need would favour, even require, the use of the
affect heuristic because it is quick. The other approach is analytic – with time to
spare, we may spend much time analysing the pros and cons of alternative
strategies.
ActivityGive
12.5
an example of a decision you have made based on the affect heuristic and one
based on a cognitive process.
Figure 12.5 Slovic’s characterization of 81 hazards
Source: Slovic (2000)
Risk perception and the psychometric paradigm 153
Feedback
As is often said, ‘Is it your heart or your head that makes decisions?’ The heart is
associated with emotion and intuition, and the head with analysis and wisdom. Human
beings rely upon both. You should not find it difficult to answer this question.
There is no dearth of evidence in everyday life that people apprehend reality in two
fundamentally different ways, one variously labelled intuitive, automatic, natural, non-verbal,
narrative, and experiential, and the other analytical, deliberative, verbal, and rational.
The rational system is a deliberative, analytic system that functions by way of established
rules of logic and evidence (e.g. probability theory). The experiential system encodes
reality in images, metaphors, and narratives to which affective feelings have become
attached.
This intuitive–experiential system of decision making to which Epstein refers is
intimately associated with affect. It is believed to work as follows: when you
encounter an emotionally significant event you automatically search your memory
for similar events including their emotional accompaniments. If the feelings gen-
erated are pleasant, actions to reproduce the feelings are activated, and vice versa.
While there is a growing belief that cognitive heuristics, including the affect heur-
istic, are important aids to decision making, it is also felt that too much or too little
affect can do disturbing things to your decision making processes. For example,
there is evidence that the affect heuristic responds poorly to numerical information
and this can have very real consequences in decision settings where resources need
to be allocated.
the affect heuristic enables us to be rational actors in many important situations. But not in
all situations. It works beautifully when our experience enables us to anticipate accurately
how we will like the consequences of our decisions. It fails miserably when the con-
sequences turn out to be much different in character than we anticipated.
The affect heuristic is reliant upon experiential learning. This type of learning is
and has been important for human survival but, like rational thinking, is vulner-
able to various biases and shortcomings. Richard Eiser (2004) describes an example
wherein a doctor is faced with a patient expressing some symptoms of potentially
serious disease. Suppose a treatment is available which would benefit a patient who
154 Beyond the rational action approach
truly has the disease. The doctor has to decide whether to treat or not and may elect
to do so on the basis of personal clinical experience. Now she may prefer to err on
the side of safety and prescribe the treatment. This would lead to a situation in
which experience of treating patients by some medical intervention was gained.
However, the feedback which the doctor receives from this decision is selective,
because the results of not treating the patient are unknown. For example, it could
be that the symptoms would have gone away of their own accord. However, had
this happened and the patient had been prescribed drugs, say, the doctor’s belief in
the efficacy of the drugs would have been falsely strengthened.
If the drugs are inexpensive and have no adverse effects, this might not matter. But
suppose the treatment also has negative side-effects (as many do). Then the reliance
on personal experience could lead to unnecessary prescription. In fact, there are
four possibilities as shown in Table 12.2. A risk-averse doctor is likely to treat and
gain experience of treated patients, but is less likely to have experience of patients
who do not receive the treatment, leading to biased decision making.
As Eiser (2004) points out, this is a general problem in risk decision making where
we rely upon experience. False alarms and risk aversion may be a consequence of
the incomplete feedback we receive from everyday experiences. We only witness
the consequences of our actions and not the alternatives.
Another difficulty with experiential learning is that in many situations feedback
following some decision may be delayed. This has several implications. One is that
causal pathways may be hidden. An example is the link between asbestos exposure
and mesothelioma. Asbestos was used because of its valuable properties, some
aimed at reducing risks, but knowledge and experience of long-term effects like
mesothelioma were lacking and it was not until much later that causality between
asbestos exposure and mesothelioma was determined. Likewise, the emission of
carbon dioxide into the atmosphere was for centuries not a source of worry, but
science now tells us it is linked to climate change.
that event are powerful, but very few of us understand the full, and still emerging,
story and thus, in forming an opinion on the basis of Chernobyl, we may over-
generalize.
Eiser has produced a helpful summary model of what you have read here about risk
perception and the way in which the brain makes decisions (Figure 12.6). In this
model, risk perception starts with some event which triggers memories and/or
emotional reactions (affect heuristic). These result in an initial interpretation of the
event in a relatively automatic way. At the same time, if the event is public and others
experience it too, it will trigger responses which might lead to social reinforcement.
At this point, it may be that the individual will commence a more measured evalua-
tion of the event by use of cognitive heuristics. Further information will also be
received from experts which will be assessed for trustworthiness as well as meaning.
These estimations, derived from the affect and cognitive heuristics, will then some-
how be combined into a summary.
Individuals will then decide what to do. However, if they decide to treat the danger
as real and avoid it, they will often not actually know if the danger was real
(because they avoided it) nor if their actions were effective. Thus, there may be no
evidence in the way of feedback so the apparent need for the possibly unnecessary
action will be strengthened along with a potential increase in risk aversive tenden-
cies. Alternatively, a decision to treat the situation as safe and approach the poten-
tial risk will provide the individual with useful feedback about the correctness of
the decision, so updating experience and enhancing future judgement capabilities.
However, the possibility will still exist that the feedback is misinterpreted, espe-
cially if it is inconsistent or delayed.
ActivityThe
12.6
work of Slovic and his colleagues demonstrates that public concerns about
environmental hazards have, after all, a rational basis if you accept that factors such as
‘dread’ and ‘voluntariness’ are legitimate issues, and that affect-based choices may also
have validity. Write down your thoughts on the implications of this for environmental
health policy makers.
Feedback
The matter is far from simple. Policy makers will, on the one hand, feel compelled to
maximize public health gains which implies a strictly rational decision making process.
On the other hand, public concerns should be heeded in a democracy, but if this means
allocating extra resources to an issue which is really a low priority in health terms, lives
will be lost. The issue is further complicated by the fact that failure to legitimize public
concerns may place obstacles in the way of purely rational policies, even though these
would do the most good.
Figure 12.6 Flowchart representing processes involved in assessing the presence or absence
of a risk
Source: Eiser (2004)
Risk perception and the psychometric paradigm 157
Summary
Research into risk perception points to two ways by which the brain makes
decisions; either on the basis of rational analytic processes or affective response.
The prevailing view is that both processes are necessary, one providing a check on
the other. Research also reveals that the public tend to view risk in a more complex
way than experts. Many qualitative factors contribute to the public perception of
risk, whereas experts tend to rely upon rational, analytic assessments which in
turn focus on the one-dimensional probability of actual harm. It is implied that if
the public disagree with experts over the importance of a hazard, that this is not
necessarily down to lack of knowledge (though this is possible), but may be because
they have a ‘richer’ understanding of the meaning of risk.
References
Eiser RJ (2004) The Public Perception of Risk. London: Office of Science and Technology.
Epstein S (1994) Integration of the cognitive and psychodynamic unconscious, American
Psychologist, 49: 709–24.
Lichtenstein S et al. (1978) The judged versus actual risk of particular harms. Journal of
American Psychology 4: 551–78.
Slovic P (2000) The Perception of Risk. London: Earthscan.
Starr C (1969) Social benefits versus technological risk: what is our society willing to pay for
safety? Science 165: 1232–8.
Tversky A and Kahneman D (1974) Judgment under uncertainty, Science 185: 1124–31.
Cultural theory
13
Overview
While the psychometric paradigm has deepened the understanding of risk contro-
versies considerably, it suffers from the same micro-orientation as does RAP. For
this reason, cultural theory takes a very different approach in its attempt to under-
stand why people perceive and react to risk the way they do. Unlike RAP and the
psychometric paradigm, it starts out with the supposition that people have one
of four different rationalities. Depending upon their ‘choice’ of rationality, they
will react differently to the risks which they encounter. Because cultural theory
permits this diversity, it is useful in understanding why risk issues are often debated
and even contentious, for if four different rationalities exist, then four different
interpretations of the same situation must be expected. This message can be very
troubling for decision makers, who normally are seeking a single interpretation of
the ‘facts’, and a unique and uncontested solution. However, the insights gained
from cultural theory are nonetheless very useful for decision makers, including
environmental health policy advisers, who will inevitably encounter a diversity of
responses to environmental health issues and must therefore be prepared, like it or
not, for a less than straightforward existence.
Learning objectives
Key terms
Objectivism The idea that risks can be measured and that we can distinguish between their real
magnitude and what people variously and perhaps erroneously believe them to be.
Social constructivism This approach to knowledge denies the existence of reality prior to
human engagement and the validity of ‘truth’ in the sense of a corresponding representation of
reality. Instead, it poses that reality is whatever is known, and that all knowledge is socially
produced. Constructivism can thus lead to relativism, as it allows no distinction between true
and untrue statements.
Cultural theory 159
The story begins, or at least one thread of it, with natural science; the study of
ecosystem management to be precise, which, by chance, is not a far cry from
environmental health. From this work emerged the idea that all institutions (or,
for that matter, policy makers), faced with exactly the same kind of situation,
invariably adopted strategies based upon one of four different interpretations of
the stability of the ecosystem (or policy issue) with which they were concerned.
A further contention is that these four world-views, as we can call them, can be
observed in numerous walks of life besides ecosystem management, certainly
including environmental health policy formulation.
ActivityRead
13.1
the following abridged account by the geographer John Adams (1995) of what
are called the four ‘myths of nature’, corresponding to the four world-views
mentioned above, each of which is said to capture some essence of human experience
and wisdom.
1 Observe that the myths of nature are clearly depicted by the simple model of a ball
in one of four possible landscapes as in Figure 13.1.
2 Think of an example of an environmental health issue which is managed according
to each of these four myths or world-views.
3 How might the world-views help understand environmental health controversies?
Nature benign: nature, according to this myth, is predictable, bountiful, robust, stable,
and forgiving of any insults humankind might inflict upon it; however violently it might
be shaken, the ball comes safely to rest in the bottom of the basin. Nature is the
benign context of human activity, not something that needs to be managed. The
management style associated with this myth is therefore relaxed, non-interventionist,
laissez-faire.
Nature ephemeral: here nature is fragile, precarious and unforgiving. It is in danger of being
provoked by human carelessness into catastrophic collapse. The objective of environ-
mental management is the protection of nature from humans. People, the myth insists,
must tread lightly on the Earth. The guiding management rule is the precautionary
principle.
Nature perverse/tolerant: this is a combination of modified versions of the first two myths.
Within limits, nature can be relied upon to behave predictably. It is forgiving of modest
shocks to the system, but care must be taken not to knock the ball over the rim. Regulation
is required to prevent major excesses, while leaving the system to look after itself in minor
matters. This is the ecologist’s equivalent of a mixed-economy model. The manager’s style
is interventionist.
Nature capricious: nature is unpredictable. The appropriate management strategy is again
laissez-faire, in the sense that there is no point to management. Where adherents to the
myth of ‘nature benign’ trust nature to be kind and generous, the believer in ‘nature capri-
cious’ is agnostic; the future may turn out well or badly, but in any event it is beyond his
control. The non-manager’s motto is que sera sera.
Feedback
1 It should be clear that the world-view ‘nature is benign’ can be represented by a ball
in a deep bucket. However hard you whack it, it will never escape. On the other hand, if
you believe in ‘nature perverse/tolerant’ (which should sound familiar and remind you
of ALARP/ALARA and the philosophy that risks, up to a point, are tolerable), a ball in a
modestly deep bucket is what you have. In ‘nature ephemeral’ the ball is clearly deli-
cately poised, and ‘nature capricious’ is like a marble on a flat surface – it rolls wherever
it likes and is unpredictable.
2 Examples abound, but here are a few. For nature perverse/tolerant you might
think of the general approach to the management of human exposure to toxic
chemicals. Providing dose is managed so as to be below a certain threshold, then it
is presumed that there is no harm. For nature benign, you might think of the past
attitude of some industries and many nations to global carbon dioxide emissions.
The atmosphere was deemed to be so large as to be beyond significant influence by
humankind. For nature ephemeral, those who maintain that there are no ‘safe’ limits
for, say, radiation exposure, adhere to this world-view. And, finally, for nature capri-
cious, if you believe that some health conditions are induced by supernatural causes,
or by events just too complicated to understand or control, then they would fit this
model.
3 The world-views all exist at the same time. For every issue that you can identify, you
will find some people who adhere to each world-view. In the case of personal injuries,
Cultural theory 161
for example, some people believe that they are caused by accidents and are beyond
control, and you might say that they subscribe to the world-view ‘nature capricious’.
Others believe, however, that accidents can and should be eliminated, even to the
extent that the word ‘accident’ should no longer be used. These people could be
aligned with a ‘nature ephemeral’ ideology, because their belief is that accidents are so
serious that you would strive to eliminate them. Yet others, who have a ‘nature benign’
perspective, believe that the world is a hard place, and that sometimes we must expect
to be injured in the quest for personal fulfilment. These people see nature as essentially
benign in that it is not out to get them, but to be exploited, and for them, risks must be
accepted and ‘he who dares wins’. Finally, most professionals working in injury ‘control’,
as they might call it, are trying to reduce injury risk, but only until as low as reasonably
practicable (ALARP) – they are applying a nature perverse/tolerant model. In this way,
cultural theory offers an insight into why these four very different ideas about, in this
case, injury control, co-exist.
The understanding and utility of the above ‘cultural theory’ have also developed
along another route by anthropologist Mary Douglas and her co-workers (Douglas
1985). They concluded that people’s social relationships could also be classified
into four types, and that these coincided neatly with the world-views in Figure 13.1.
ActivityIn13.2
reading the following passage adapted from Schwartz and Thompson (1990), which
outlines the thinking which gave rise to the identification of these four types of rational-
ity (which are given the names ‘hierarchist’, ‘egalitarian’, ‘individualist’, and ‘fatalist’),
ask yourself if these four ‘archetypes’ really exist. It might help if you try to think of
examples of types of people who seemingly fit these descriptions, and do you fit in
there?
Feedback
In some senses at least, these archetypes, with their associated ways of operating, can
be observed throughout any society and at any level. Individualists, for example, can be
likened to individual street or market traders who seek out the freedom to bid for and
bargain with their wares, and for whom the important issue is their ability to ‘make the
books balance’. In contrast, hierarchists belong to orderly social groups who attempt to
manage things according to established rules. You might find them in places ranging
from offices, to government bureaucracies, to the organized religions, or the army.
Egalitarians, in contrast, are characterized by a sense of community but with a tendency
to be critical of both the status quo (loved by the hierarchist) and the unrestricted
freedom and its impugned lack of collective responsibility (of the individualist). In the
environmental arena, members of the Green pressure groups are often felt to be
egalitarian in outlook. But not everyone who is unconnected to organized social groups
can be described as an individualist, since that would mean they had freedom to act. In
fact, they might be in a very weak position to influence events and thus their situation
is more akin to one of fatalism, for they cannot influence the things which will happen
to them.
Having encountered the four myths of nature (world-views or rationalities) and the
four behavioural types, it is possible to make a further connection and see how the
one maps onto the other. Individualists, by their nature, find the view that nature is
benign the most hospitable, and so the two tend to go hand-in-hand, as in Figure
13.3. Likewise, egalitarians, who are questioning and concerned, find nature
ephemeral to suit their world-view. It provides a raison d’être for their perspective.
Hierarchists, in turn, like to organize and manage, and this fits the mould of nature
Cultural theory 163
The implications of cultural theory, if accepted, are profound. It does not occupy
the opposite corner to RAP of Ortwin Renn’s classification of theories of risk for
nothing (Figure 11.1). RAP is anchored in ‘objectivism’, the view that risks can be
measured, whereas under cultural theory, this has to give way to ‘constructivism’,
the idea that risk is inherently subjective – something that people project onto
whatever they observe. This follows because risks are viewed differently, according
to which of the four world-views or cultures you subscribe.
Cultural theorists then tell us that because of the fourfold plurality of world-views,
it is wrong to impose upon people a decision process framed around one view only
(e.g. the hierarchist’s RAP), because this disenfranchises all those with other views,
and in any case could not win their support since they would see the problem as
incorrectly framed. However, they also say that the prospect is not one which rejects
science, nor one which accepts extreme forms of relativism, according to which,
anyone’s opinion is as good as anyone else’s (Thompson et al. 1998). In fact, they
argue that the fourfold typology is sufficient to encompass much of the diversity of
people’s views.
164 Beyond the rational action approach
ActivityIn13.3
order to understand more fully the insights which can be gained from cultural theory,
consider the example of global climate change. The following article by John Adams
and Michael Thompson (2002) describes how this important challenge for humanity is
understood and interpreted by three of the four world-views. As you read the article,
reflect upon the following questions:
1 Which world-view is not represented? Why, and does it matter?
2 Which world-view is correct?
3 How should the policy maker react to the different world-views?
Since everything is connected to everything else, this story continues, we cannot properly
understand environmental degradation unless we see it as a symptom of this wider social
malaise. The way humans pollute, degrade, and destroy the natural world is merely a very
visible indicator for the way they treat each other and particularly the weaker members of
society. The logic that allows us to fell thousands of square kilometres of rainforests, to
dump toxins in waterways, or pollute the air, is precisely the same logic that produces
racism, misogyny, and xenophobia. Tackling one problem inevitably implies tackling all the
others.
The heroes of the profligacy story are those organizations and individuals who have man-
aged to see through the chimera of progress in advanced industrial society. They are those
groups and persons that understand that the fate of humans is inextricably linked to the
fate of Planet Earth. The heroes understand that, in order to halt environmental degrad-
ation, we have to address the fundamental global inequities. In short, the heroes of the
profligacy policy argument are those organizations of protest such as, most prominently,
Greenpeace or Friends of the Earth. These organizations, we need hardly point out, are
strongly biased towards the egalitarian social solidarity.
What, then, is the moral of the profligacy story? Its proponents point to a number of
solutions. In terms of immediate policy, the profligacy tale urges us to adopt the pre-
cautionary principle in all cases: unless policy actors can prove that a particular activity is
innocuous to the environment, they should refrain from it. The underlying idea here is that
the environment is precariously balanced on the brink of a precipice.
The story further calls for drastic cuts in carbon dioxide emissions; since the industrialized
North produces most of these emissions, the onus is on advanced capitalist states to take
action. Of course, this policy argument calls for a total and complete ban on chlorofluoro-
carbons. Yet none of these measures, the story continues, is likely to be fruitful on its own.
In order to really tackle the problem of global climate change we in the affluent North
will have to fundamentally reform our political institutions and our unsustainable life-
styles. Rather than professionalized democracies and huge centralized administrations,
the advocates of the profligacy story suggest we decentralize decision-making down to the
grassroots level. Rather than continuing to produce ever-increasing amounts of waste,
we should aim at conserving the fragile natural resources we have: we should, in a word,
move from the idea of a waste society to the concept of a conserve society. Only then can
we meet real human needs. What are real human needs? Simple, they are the needs of
Planet Earth.
Population: a hierarchist story
This policy argument tells a story of uncontrolled population growth in the poorer
regions of the world. Rapidly increasing population in the South, this story argues, is
placing local and global eco-systems under pressures that are fast becoming dangerously
uncontrollable: more people means more resource consumption which inevitably leads to
environmental degradation. The setting of the population policy story differs slightly, but
significantly, from the settings in the other two diagnoses. Like the protagonists of the
profligacy story, the population policy argument maintains that global climate change is a
moral issue.
Human beings, due to their singular position in the natural world, are the custodians of
Planet Earth; since civilization and technological progress have allowed us to understand
the natural world more than other species, we have a moral obligation to apply this
knowledge wisely. Unlike the profligacy story, the population tale assumes that humans
166 Beyond the rational action approach
have a special status outside natural processes. The population story, like that of the
proponents of the pricing argument (see next story), contends that human actions are
rational. However, unlike the pricing argument, the population story tells us the sum of
individual rational actions can lead to irrational and detrimental outcomes. The popula-
tion story, then, is set in a world that needs rational management in order to become
sustainable. Yet, while the motive of rational management is an ethical duty to preserve
the planet, the means of management are technical. Economic growth, and the socio-
economic system that underpins that growth, are necessary components in any global
climate change policy response. However, economic growth in itself is no solution: it
must be tempered, directed, and balanced by the careful application of knowledge and
judgement.
The villain in the population tale is uncontrolled population growth. Since each individual
has a fixed set of basic human needs (such as food, shelter, security, etc.) and these needs
are then standardized at every level of socio-economic development, population increase,
other things being equal, must lead to an increase in the aggregate demand for resources.
Humans, the story insists, satisfy their basic human needs by consuming resources. It
follows that population growth must lead to an increase in resource consumption: more
people will produce more carbon dioxide to satisfy their basic needs. Given the limited
nature of most resources, population growth must invariably lead to over-consumption
and degradation of natural resources.
The heroes of the population story are those institutions with both the organizational
capacities (that is, the technical knowledge) and the ‘right’ sense of moral responsibility. In
short, the global climate change issue should be left to experts situated in large-scale, well-
organized administrations. In terms of our typology of organizational types, the population
story emerges from hierarchically structured institutions.
The moral of the population story is to rationally control population growth. In particular,
this means the introduction of family planning and education in the countries most likely
to suffer from rapid population growth. Here, the onus for action is quite clearly on the
countries of the South. Rapid population growth has eroded societal management capaci-
ties; if we are to tackle the global climate change issue, we must first establish the proper
organizational preconditions.
Prices: an individualist story
This story locates the causes of global climate change in the relative prices of natural
resources. Historically, prices have poorly reflected the underlying economic scarcities;
the result, plain for all to see, is a relative over-consumption of natural resources.
The setting of the prices tale is the world of markets and economic growth. Unlike the
profligacy story, the prices diagnosis sees no reason to muddy the conceptual waters with
extraneous considerations of social equality. Yes, it says, global climate change is an import-
ant issue, but it is an issue that is amenable to precise analytical treatment. It is, in short, a
technical issue to which we can apply a technical discourse.
Economic growth, far from being a problem, is the sole source of salvation from environ-
mental degradation. Environmental protection, the proponents of this policy argument
contend, is a very costly business. In order, then, to be able to foot the huge bill for adjusting
to a more sustainable economy, societies will have to command sufficient funds. These
funds, in turn, will not materialize from thin air: only economic growth can provide the
necessary resources to tackle the expensive task of greening the economy.
Cultural theory 167
In sum, the prices tale takes place in a world determined by the Invisible Hand (a
fundamental premise of classical economists). Here, individuals know and can precisely
rank their preferences. In the world of the prices story, individual pursuit of rational
self-interest (economic utility) leads, as if by magic, to the optimal allocation of resources.
If market forces are allowed to operate as they should, then resource prices will
accurately reflect underlying scarcities; the price mechanism then keeps environment-
degrading consumption in check. However, if someone (usually the misguided policy
maker) meddles with market forces, prices cannot reflect real scarcities; this gives rise
to incentives for rational economic actors to over- or under-consume a particular
resource.
The villain in the prices story is misguided economic policy. Barriers to international trade,
subsidies to inefficient national industries, as well as price and wage floors, introduce
distortions to the self-regulatory powers of the market. These distortions have historically
led markets to place a monetary value on natural resources that belies the true market
value. The result, the protagonists of this policy argument maintain, has been wholesale
over-consumption and degradation of the natural world.
The heroes of the prices story are those institutions that understand the economics of
resource consumption. In the global climate change debate, these institutions comprise
players such as the Global Climate Coalition and trans-national energy companies. In
terms of the cultural theory typology, the heroes of this story are those institutions that
are strongly permeated by the individualist solidarity.
The moral of the prices story is as simple as its prognosis: in order to successfully face the
challenge of global climate change, we have to ‘get the prices right’. Unlike the profligacy
story, the prices tale sees no necessity to restructure existing institutions. If it is the
distortions of global, national and regional market mechanisms that undervalue natural
resources, then any climate change policy that fails to remove these distortions is ‘funda-
mentally flawed’. Policy responses must work ‘with the market’. Here, concrete policy
proposals consist of both general measures, such as the liberalization of global trade, as
well as more specific measures, such as carbon taxes or tradable emission permits.
Feedback
1 You should quickly spot that the fatalists’ world-view is absent. This is because
fatalists would not be involved in the debate, because they would not expect to be
listened to, and their position would be undefined. Whether this matters is a matter of
opinion. You could argue that if they have nothing to say or contribute, then why listen?
On the other hand, if they can be approached, and convinced that their views will be
taken seriously and responded to, then a better policy process will result. Nor should
you overlook the fact that even the fatalists can, if strongly motivated by a perceived
injustice, take up arms quite effectively.
2 Cultural theorists would argue that if you ask the question, ‘Which view is correct?’
then you have made a mistake! The three stories tell plausible but conflicting tales
of climate change. All three tales use reason and logic to argue their points. None of
the tales is ‘wrong’, in the sense of being implausible or incredible. Yet, at the same
time, none of the stories is completely ‘right’; each argument focuses on those aspects
of climate change for which there is a suitable solution cast within the terms of a
particular world-view.
168 Beyond the rational action approach
Adams (1995) likens it to ‘moving up the insight axis’ as shown in Figure 13.4. This
is where policy makers must try to locate themselves – above the fray so that they
can discern and be detached from the competing explanations and concerns, and
thus be better placed to take an even-handed approach which inspires mutual
trust.
Figure 13.4 The insight axis and ‘rising above the fray’
Source: Adams (1995)
Cultural theory 169
Summary
This third model of risk is radically different from those which have gone before.
While not at all rejecting the roles for science and economics in policy formulation,
it opens up new vistas, demanding that recognition be given to alternative world-
views, all of which are seen as legitimate, and each of which points the finger at
different causes and different solutions. The task of the policy maker thus now
encompasses a powerful need to communicate and listen to stakeholders and any-
one affected by policy decisions. The aim of the policy maker must be to rise above
the fray; to listen, communicate and respond; and to try to accommodate as many
concerns as possible into the policy process in the quest for sensible and equitable
solutions.
References
Overview
In Section 2 you read about what is nowadays regarded as the classic quantitative
method of environmental health risk assessment. However, you are now aware
from Section 3 that despite its rapidly expanding global presence, it does not go
unchallenged. By no means should this be interpreted as a death knell, for all
things, including good things, have their limitations and their detractors. In this
section of the book you will read about some alternative approaches to environ-
mental health policy, commencing in this chapter with environmental, social
and health impact assessment. This is not at all to say that these alternatives
should necessarily usurp the classic method, but that you should be aware of their
existence, their merits, and their limitations.
Learning objectives
Key terms
Environmental impact assessment A technique and process by which information about the
environmental effects of a project is collected.
Health impact assessment A method which aims to identify the likely changes in health risk of
a policy, programme, plan, or development action on a defined population.
Social impact assessment The process of assessing or estimating, in advance, the social
consequences that are likely to follow from specific policy actions or project developments.
Strategic environmental assessment The formalized, systematic and comprehensive process
of evaluating the environmental effects of a policy, plan or programme and its alternatives.
Sustainability Development which meets the needs of the present without compromising the
ability of future generations to meet their own needs.
174 Other approaches
The growing interest of the public in the state of the environment has resulted in
international initiatives to safeguard the environment and promote sustainability.
An early example was the requirement in some countries for environmental impact
assessments (EIA) to be carried out on new development projects to ensure that
their impacts are taken into account. While it is true that healthy environments
and healthy populations are inter-related, it nonetheless came to be felt that EIAs as
such did not fully integrate health concerns, which tended to be dealt with
through the provision of health care services and discrete public health legislation.
For this reason, there have been moves to introduce specific environmental man-
agement tools which focus upon health – health impact assessments (HIA) – or to
more fully integrate health issues into EIAs (HEIA). For those whose interest is
ecosystems, ecological risk assessment is a further approach, though very much in
its infancy.
However, just as there exists potential for development projects to give rise to
health impacts, benign or malignant, the same must also be true at the policy level.
One example is agricultural policy which can have health impacts through food
quality, access to food, nutritional status of the affected population, environmental
disruption, the supply and nature of jobs, or other implications such as the intro-
duction of new varieties such as rape seed with their allergenic properties (BMA
1998). In fact, health consequences may be associated with all levels of activity,
from policies to plans, programmes and projects. This has given rise to a further
suite of strategic-level management tools for assessing health impacts, including
strategic environmental assessment (SEA), social impact assessment (SIA), and the
like. This chapter discusses these initiatives, starting with EIA.
A technique and process by which information about the environmental effects of a pro-
ject is collected, both by the developer and from other sources, and taken into account by
the planning authority in forming their judgements on whether the development should go
ahead.
(DoE 1989)
The generic process is outlined in Figure 14.1. You should note in particular that
consultation with stakeholders, and their potential participation, is seen as an inte-
gral part of the process. This is an issue which will be revisited many times in this
book. For the moment, it is useful to be reminded that three main theoretical
justifications have been given for increased participation. Academics refer to these
reasons as normative, epistemological, and instrumental, grand terms whose meanings
are relatively simple – see Table 14.1.
Environmental, social and health impact assessment 175
ActivityIn14.1
contemplating the pervasive role for stakeholder consultation and participation in
EIA as proposed in Figure 14.1, make some notes on how this sits with the various
models of environmental decision making that you have encountered earlier (e.g. RAP,
the psychometric paradigm, and cultural theory).
176 Other approaches
Table 14.1 The cited advantages of broadening participation in the EIA process
Nature of goal Description
Normative Broadening participation is in the interests of democracy because
environmental policy decisions are not value-free
Epistemological Decision makers do not know everything and can benefit from knowledge
outside of traditional regulatory structures
Instrumental Broadening participation helps ensure the political acceptability of resulting
policy decisions
Source: Adapted from Rothstein (2004)
Feedback
A strict RAP model might well not see the need for this consultation. After all, if risks
have been properly assessed, if stakeholders are rational actors (individual utility
maximizers), and if their values have been properly assessed by WTP techniques, then
RAP should incorporate everything they care about, and Pareto optimization will
determine the best path. On the other hand, the psychometric paradigm might well
say, ‘Hold on, the concerns of the stakeholders may be legitimate even if they are not
strictly “rational” in RAP’s terms. We must elicit their concerns and at least consider
factoring them into the decision process.’ And, as you might guess, cultural theorists
would be appalled at RAP’s arrogance, demanding that consultation permeate the
entire process.
Many countries now have a legal requirement that new, substantive programmes,
such as dam-building, must undertake an EIA; this includes major development
projects funded by the World Bank. This is directly related to the declaration at the
UN conference on Environment and Development that ‘human beings were at the
centre of concern for sustainable development’ and so development projects now
must prepare EIAs.
As shown in Figure 14.1, EIAs will in part follow a similar pattern to quantitative
risk assessment, with hazards being identified, dose–response relationships sought,
and emissions assessed from the project’s technical specifications. To assess
exposure and hence impacts, this primary data must be combined with secondary
data on population and geography and possibly other factors too. These processes
are necessary for every aspect of interest, for example, noise, air quality, or water
supply. As this process is supposed to occur prior to the commencement of the
project, there is little opportunity for new research and so the assessment has to be
based on existing scientific knowledge. In practice this means that data must be
extrapolated from other situations and assumptions. As reported by the Ministry of
Environment and Forests of India (2006), EIA studies require not only a lot of
primary data (such as air and water quality) but also much secondary environ-
mental data on demography, forestry, watersheds, land use, geology and ecology,
and this has often been a major bottle neck in achieving the full benefits of EIA.
Environmental, social and health impact assessment 177
ActivityWrite
14.2
down as many elements as you can think of that could be included in an
environmental impact assessment. Use a specific local or regional example with
which you are familiar. The following categories will help you to structure your
answer:
• environmental pollution and ecological considerations
• use of natural resources
• social effects
• economic factors.
Feedback
Look at Table 14.2 for a comprehensive list of the variety of possible environmental
impacts of a development project, and compare it with the items you noted down.
ActivityIf14.3
an EIA had been required before the project started, and the significance of
malaria-infected workers understood, what could have been done to mitigate the risk?
Feedback
Malaria-infected workers could have been excluded or treated, and any new cases
could have been monitored and treated. New housing in the towns could have been
Environmental, social and health impact assessment 179
built without the possibility of standing water in the basements. Proper provision
would have been made to house seasonal agricultural workers away from the shanty
towns on the water’s edge, and industrial workers could have been provided with
accommodation of an adequate standard.
EIAs have been criticized because there has been a tendency to concentrate on
immediate, biophysical consequences of programmes or projects, omitting con-
sideration of social effects of development projects. They have also been criticized
for a lack of health expertise, with conclusions being drawn on the basis of com-
parison with environmental quality criteria (for example, soil, air and water quality
standards) rather than fully-fledged health risk assessments. Another perceived
shortcoming has been that they are normally applied at the project level, and
therefore miss out the cumulative effects of broader initiatives at the policy level.
This has given rise to the technique of strategic environmental assessment (SEA).
The simple definition of SEA is that it is the environmental assessment of any stra-
tegic action, whether at the level of policy, plan or programme. More specifically, it
has been defined as:
the formalised, systematic and comprehensive process of evaluating the environmental
effects of a policy, plan or programme and its alternatives, including the preparation of a
written report on the findings of that evaluation, and using the findings in publicly account-
able decision-making.
(Thérivel and Partidário 1996)
SEAs differ from EIAs in that they offer a much better opportunity to address the
cumulative impacts of many individual projects, any linkages to other policies, and
sustainability issues. Despite their strategic nature, SEAs can be applied at every
level from national and international, to regional or local.
As an example of a local application, consider the Bara Operational Forest
Management Plan (OFMP) of the Kingdom of Nepal (Khadka et al. 1996). The
OFMP was believed to be necessary to permit commercialization of timber and fuel
wood production in the Bara District forest. It had many apparent advantages,
including better management of dwindling forest species, provision of jobs, forest
regeneration, and protection of sensitive areas. However, it was clear that the
implementation of such a plan would have an array of adverse as well as beneficial
impacts associated with the biophysical, social and economic aspects of the area. It
was thus important to analyse the plan prior to implementation so that negative
consequences could be avoided or minimized cost-effectively. Approximately 150
impacts were identified through stakeholder consultation and these were discussed
in a series of workshops open to all. The following edited extracts from an article by
Ram Khadka and colleagues provide information on the main issues which were
identified by this SEA.
180 Other approaches
forests are being managed. The OFMP recognizes this, but needs to create a well-planned
biodiversity strategy.
Tenure rights of local people
The plan needs to acknowledge more strongly the issue of tenure and how it will be
resolved.
Legal and institutional arrangements
Legislation requires the plan not to affect the environment significantly. The plan needs to
show sufficient grounds that this is the case.
Timber harvesting methods
The plan needs to outline how the potential negative impacts of timber harvesting will be
avoided. A series of harvesting guidelines would be useful in this respect.
Silvicultural practices
The OFMP describes silvicultural activities but more information is needed on negative
impacts and effects on animal and plant habitats.
ActivityWhat
14.4
was the coverage of the study and what were the implications of its findings for
the true nature of the OFMP?
Feedback
The SEA study was commendable for its comprehensiveness in terms of socio-
economic, ecological and political impacts, but, although it did receive some consider-
ation, health impacts are not directly mentioned in this list. Regarding the OFMP,
Khadka et al. (1996) actually maintain that this more closely resembles a timber man-
agement plan than a forest management plan, and lacks sufficient consideration of
biophysical, social and cultural aspects related to forest resource utilization.
Despite criticisms, SEA has many potential benefits as identified by Sadler and
Baxter (1997). SEA does the following:
Project EIAs have generally been formulated with the physical rather than the
social environment in mind, with the emphasis on conservation of nature, ecol-
ogy, and sustainability. Typical considerations include traffic, air pollution and
climate. Social impacts are often under-estimated. So apart from the insufficiency
of EIA at the strategic level, which SEA attempts to rectify, this further deficit
needed to be addressed. This contributed to the emergence of the concept of social
impact assessment (SIA).
As a potential example of the utility of SIA, the following edited extract from a New
Zealand government website (New Zealand Government 2006) describes concerns
over the social impacts of the proposed provision of broadband internet access to
rural and urban areas.
ActivityAs14.5
you read it, make notes about the author’s attitude to the following:
1 Why policy makers should be cautious in what outwardly appears to be a valid way
of extending democracy.
2 The reasons why these arguably important social impacts are often neglected. Add
your own thoughts about this too.
Feedback
1 The author surmises that rather than being an extension of democracy, that the
move will exacerbate social inequalities because a practical consequence of the provi-
sion of broadband would be a loss of face-to-face contact which is the preferred
method of communication by some social groups. It therefore needs to be thought
through in more detail.
2 As the author says, although recognition of social and cultural impacts of technology
is not new, it is often discarded as eccentric and minimalized. Why should this occur?
Social and cultural theorists might well argue that it is in part attributable to the
fragmentation of society. Technical decisions about things like broadband are generally
delegated to technical people whose strength is seldom in the social implications of
their technologies.
In the following extract, Susan Joyce (2001), a Canadian researcher, notes that
attitudes to the desirability of SIA also extend to the meaning of SIA. In this account,
Joyce refers to a ‘technocratic approach’ which is clearly a reference to what we
have previously described as RAP.
ActivityIn14.6
reading the extracts, note and comment on the following:
1 the role being assigned to technical experts and the method of evaluating project
proposals;
2 the way in which the successfulness of SIA was diagnosed.
Feedback
1 The role of technical experts is still regarded as essential though not as dominant as
once it might have been. This shift is reflected in the perceived nature of the assessment
criteria used for project evaluation, which were seen to be socio-political rather than
technical.
Environmental, social and health impact assessment 185
2 There is some inference that the success of SIA was measured by the outcomes of
the inquiries, namely that both projects failed to win approval. However, this is not
strictly proven because it may be that, for example, the economic cases were weak.
Only history can tell if the decisions were actually for the best and even then, it might, as
discussed by Eiser (p 154), be imponderable.
The spur for the development of HIA has been a perception that while human
health has often been taken into account in EIAs and SEAs, assessments tend to
estimate only the negative effects resulting from expected changes in (physical)
environmental media, neglecting the effects of modifications on other health
determinants, such as socio-economic ones, and the possibility of promoting
health benefits (UNECE 2004). The effective management of health and other
environmental risks also requires medical input, often lacking in traditional
EIAs. This requirement arises because of the complexity of health impacts attrib-
utable to synergistic effects of different hazards, cumulative effects of chronic
exposures, and the differing vulnerabilities of members of the community to
adverse effects.
The British Medical Association (1998) says that the failure to fully integrate, or
even give attention to, the relationships between the biophysical environment and
human health remains a major deficiency in a number of countries, including the
UK, and proposes as a solution an approach which overtly integrates HIA into EIA.
Figure 14.2 shows how the EIA process (as in Figure 14.1) could be adapted. During
scoping and screening the point is made that all five main categories of health
detriment should be specifically considered. These are described by the BMA as
follows:
• communicable disease, e.g. sexually transmitted diseases, infections, micro-
biological food contamination;
• non-communicable disease, e.g. chronic poisoning, lung disease associated with
dust, noise, vibration;
• inappropriate nutrition, e.g. deprivation, access to food, pesticide or other chem-
ical contamination;
• injury, e.g. traffic injury, violence, occupational injury;
• mental disorder, e.g. stress, chronic depression, substance abuse.
By doing this, a medical model is imposed upon the process. Further, rather than
attempt to quantify risks associated with identified hazards, which the BMA regards
as a difficult and distant goal whose validity is further brought into question
because people perceive risks differently, they recommend a simple ranking pro-
cedure as more realistic. This might take the form of identifying, for each of the
health hazards associated with the activity, the health change as a result of carrying
it out. This, the BMA proposes, might be done simply, for example, in terms of: no
apparent change, increased risk, or decreased risk. Table 14.3 is suggested as one
way of presenting such data.
Figure 14.2 Integrating HIA into EIA
Source: BMA (1998)
Environmental, social and health impact assessment 187
You have been asked to carry out a health impact assessment as part of a feasibility
study for the provision of water standpipes in each street in a poor urban area. The
previous water supply came from contaminated wells which were some distance
from the main dwelling locations. Communicable diseases have already been iden-
tified as the main health issue in this area, but the other forms of health impact
should not be discounted.
ActivityUse
14.7
Table 14.3 to record the potential health impacts of the new proposal. State whether
they are positive or negative impacts, and make a note of your recommendations to the
planning committee on whether or not the development should go ahead.
Feedback
Table 14.4 lists some responses. You might have recommended the provision of
adequate drainage for the standpipes to prevent vector breeding conditions in stagnant
water. Overall, the positive health benefits seemingly outweigh the negative impacts,
providing straightforward safeguards are put in place, so it could be that you would
recommend that the development should go ahead provided there are no other
anticipated adverse effects.
188 Other approaches
Summary
Local, regional, national and international concerns for the environment have led
to the generation of a range of techniques for assessing the environmental impacts
of developments. Initially the focus was on the environmental impact of projects.
This gradually expanded to the policy level via the use of strategic-level assess-
ments. More recently there have been moves to include health, socio-economic
and political dimensions. In one sense, these trends may be a reflection of acknow-
ledged deficiencies of strict RAP approaches. Also, the issue of sustainability is
firmly on the agenda, and this requires a new perspective, one which links grass-
roots actions with a ‘grand plan’. The techniques themselves have their own
shortcomings, and this is unsurprising given that some are novel. A noticeable
feature is that all, though some more than others, shift the emphasis towards
consultative processes and stakeholder engagement and away from what some see
Environmental, social and health impact assessment 189
References
British Medical Association (1998) Health and Environmental Impact Assessment. London:
Earthscan.
Department of the Environment (1989) Environmental Assessment: A Guide to the Procedures.
London: HMSO.
Joyce SA (2001) Social Impact Assessment in the Mining Industry: Current Situation and Future
Directions. Report No. 46. London: International Institute for Environment and Develop-
ment and World Business Council for Sustainable Development.
Khadka R, McEachern J, Rautiainen O and Uttam SS (1996) SEA of the Bara Forest Management
Plan, Nepal, in Thérivel R and Partidário MR (eds) The Practice of Strategic Environmental
Assessment. London: Earthscan.
Ministry of Environment and Forests of India (2006) http://www.eicinformation.org/
New Zealand Government (2006) http://www.e.govt.nz/archive/services/authentication/
tikanga-200408/chapter8.html
Rothstein HF (2004) Precautionary bans or sacrificial lambs? Participative risk regulation and
the reform of the UK food safety regime. Public Administration 82(4): 857–81.
Sadler B and Baxter M (1997) Taking stock of SEA. Environmental Assessment 5: 14–16.
Thérivel R and Partidário MR (1996) The Practice of Strategic Environmental Assessment. London:
Earthscan.
UN Economic Commission for Europe (2004) Human health in the EIA convention and SEA
protocol, in Proceedings of the Espoo Convention on Transboundary Environmental Impact
Assessment. http://www.unece.org/env/eia/health.html
WHO/CEMP (1992) Environmental and Health Impact Assessment of Development Projects: A
Handbook for Practitioners. London: Elsevier Applied Science.
Environmental risk ranking
15
Overview
Learning objectives
Key term
Human exposure – rodent potency index (HERP) An index devised for ranking the risk of
potential human carcinogens based on average human exposure and rodent toxicity data.
ActivityTo15.1consolidate what we have learned, describe and distinguish between the ranking
methods in Figure 7.1 and Figures 10.1 and 12.5.
Feedback
Figure 7.1 provides either a semi-quantitative or qualitative ranking based on the
probability of risks and their consequences, and is usually based on expert knowledge.
Figure 10.1 goes one step further by thinking also about the cost-effectiveness of
potential risk reduction measures. Figure 12.5 is very different, producing a ranking
based on the quantified public perceptions of risks which are encapsulated in dimensions
of ‘dread’ and ‘familiarity’. Those in the top-right quadrant are potentially ‘prioritized’
for action.
In 1987, Bruce Ames and colleagues devised a technical method of ranking the scale
of possible human hazard associated with a wide range of chemical carcinogens to
which people are exposed via diet or workplace. A sample of their calculations is
shown in Table 15.1. This method is based on working out the ratio (expressed as
a percentage) of the average daily human intake, expressed in mg/kg, divided by
TD50 which is the daily dose rate (in mg/kg) which halves the percentage of tumour-
free animals by the end of a standard lifetime. Thus, the higher the number in the
left-hand column of Table 15.1, the higher the possible health hazard.
ActivityThe
15.2
HERP hazard factors for alcoholic drinks and DDT exposure have been omitted
from Table 15.1. Work out what they should be using the data in Table 15.1.
192 Other approaches
Table 15.1 Ranking possible human cancer hazards with the HERP index
Possible Average daily Human dose of Potency of carcinogen
hazard human exposure rodent carcinogen TD50 mg/kg/day
HERP % (USA) per person (70 kg)
per day Rats Mice
0.00008 PCBs (1984–86) PCBs 98 ng 1.74 (9.58)
0.0008 Tap water 1 litre Chloroform 51 g (262) 90.3
(1987–92)
0.001 Bacon 19 g Diethylnitrosamine 0.0266 (+)
19 ng
*** DDT (pre-1972 ban) DDT 13.8 g (84.7) 12.8
0.003 Home air (14h/day) Benzene 155 g (169) 77.5
*** Alcoholic beverages, Ethyl alcohol 22.8 ml 9110 (—)
all kinds
6.1 Tetrachloroethylene Tetrachloroethylene 101 (126)
(dry cleaners 1980– 433 mg
90)
140 High exposure EDB Ethylene dibromide 1.52 (7.45)
workers (pre-1977) 150 mg
Source: Ames et al. (1987) with updates from Gold et al. (2003). See Gold et al. (2002) for full text.
Note: Parentheses indicate TD50 not used in calculation, this being data for the species with the less potent
TD50. (−) = negative in cancer test(s); (+) = positive cancer test(s) but not suitable for calculating TD50.
Feedback
For alcoholic drinks, taking as an approximation the density of alcohol to be equal to 1.0
and noting that 1ml weighs 1g, the calculation is as follows:
HERP (%) = (22,800/70) ÷ (9,110) × 100% = 3.6%
For DDT:
HERP (%) = (13.8 × 10−3/70) ÷ (12.8) × 100% = 0.002% (after rounding)
In 1987, concerns were expressed in the USA about whether the Environmental
Protection Agency (EPA) was making the best use of its resources, that is, ‘were its
activities properly prioritised with the most serious hazards receiving the greatest
attention?’ As a consequence, the National Comparative Risk Project was insti-
gated, the aim of which was to assess 30 hazards for which EPA had responsibilities
(Table 15.2) against four risk criteria (US EPA, 1987). The risk criteria were: human
cancer risk, human non-cancer risk, ecological risk, and welfare risk. Each hazard
was assessed and ranked against each criterion, but a combined ranking was not
attempted. Despite a number of recognized limitations, the study was sufficient to
show that there was a mismatch between rankings by experts and those reflected in
public opinion.
Environmental risk ranking 193
Table 15.2 The EPA’s 1987 list of environmental issues for ranking
1 Primary air pollutants (sulphur dioxide, ozone, etc.)
2 Hazardous/ toxic air pollutants
3 Other air pollutants
4 Radon gas (indoors)
5 Other indoor air pollution
6 Radiation
7 Substances suspected of depleting stratospheric ozone
8 Carbon dioxide
9 Direct point-source discharges to surface waters
10 Indirect point-source discharges to surface waters
11 Non-point source discharges to surface waters
12 Contaminated sludge
13 Discharges to estuaries and oceans
14 Drinking water supplies
15 Active hazardous waste sites
16 Inactive waste sites
17 Municipal waste sites
18 Industrial waste sites
19 Mine wastes
20 Accidental releases of toxics
21 Accidental oil spills
22 Leaks from storage tanks
23 Other groundwater contamination
24 Pesticide residues on food
25 Pesticide application
26 Other pesticide risks
27 New toxic chemicals
28 Biotechnology
29 Consumer product exposure
30 Occupational exposure to chemicals
The matter was revisited, in more detail, by the US Strategic Advisory Board
(SAB). The SAB set up three committees – on ecology and welfare, human health,
and strategic options – to take the work forward. The committees were comprised
of scientists, engineers and environmental managers. The ecology and welfare
committee assessed and rated the hazards, including some new ones, against the
following criteria: spatial extent of problem, importance of affected ecosystem,
Table 15.3 Summary of SAB’s ecological risk ranking work
Key: HHH > HH > H > M > L where HHH = highest etc.
Hazard Extent of stress Medium Recovery time
potential to cause ecological effects, intensity of exposure and potential for recov-
ery. Table 15.3 shows the ratings achieved and how they were expressed. Note that
the hazards are rated in terms of their harmfulness and not their potential for
amelioration.
The human health committee identified primary air pollutants, hazardous air
pollutants, indoor radon, indoor air pollution excluding radon, drinking water
pollutants, pesticide application and occupational chemical exposure as deserving
high rank. However, they were cautious about developing an aggregate ranking of
agents because the attributable health effects were so disparate.
ActivityRead
15.2
the following passage by SAB and note the nature of the problem they experienced.
Can you suggest ways by which SAB’s problem could be overcome?
Comparing harms
. . . to attempt a relative ranking in terms of severity of such disparate health outcomes
as birth defects in infants compared to paralysis in older persons, requires consideration
of many dimensions of the values we place on various members of society, families and
the utility of specific physical and mental functions for individuals and society. Such a
comparison requires that the impact of each effect be scored for severity, a process
necessitating selection of suitable measures and scales of severity, as well as appropriate
weighting factors. In addition, the current disparity in risk assessment approaches for
carcinogens and systemic toxicants makes it exceedingly difficult to construct a universally
acceptable aggregate ranking.
Feedback
There is a major difficulty in comparing health detriments (harms) of different kinds.
You touched on issues of this kind in Chapter 9 when radiological harms were being
considered. There are many dimensions of harm which would need to be considered,
including severity of pain, duration of harm, loss of life-years, loss of ability to function,
and so on. A great deal of work has now been done on establishing multi-dimensional
scales for comparing health states, however. These include scales which go by the name
of QALY (quality-adjusted life years), and DALY (disability adjusted life year).
ActivityBefore
15.3
reading the SAB Committee’s list of eight criteria, try drawing up your own list
of criteria for evaluating risk reduction interventions. It might help to think of a few
examples of interventions and work from there.
196 Other approaches
Feedback
The eight criteria are shown in Table 15.4. Note that cost is considered from two
perspectives – that of the EPA itself, and that of society. Dependability, or certainty, is
another key factor along with implementability. And, as you would expect, the amount
of risk reduction which the measure is expected to achieve is listed. There is little point
in investing in measures which provide little benefit in terms of risk reduction. In fact,
SAB elected to look at this through the fractional, or relative, change in risk, and that
resulted in a nationwide discussion about the use of risk-based measures as a means
of setting priorities. Table 15.4 also includes cost-effectiveness. There is no mention of
cost–benefit, but it is inescapable that this would not surface, explicitly or implicitly,
when the viability of control options was debated.
The EPA’s pioneering work was aptly entitled ‘Unfinished business’. Since that time,
a number of agencies, regional authorities and communities around the world have
sought to prioritize their activities just as EPA set out to do (Clarence-Davies 1996;
HSE 1997). All systems for ranking or prioritizing need to be purpose-designed for
the institutional and cultural setting at hand, but certain characteristics, and issues,
can be expected to occur often. The general procedure is in fact to carry out the
following sequence of steps (Morgan et al. 1995):
1 List the risks to be ranked (prioritized).
2 Identify the risk attributes (characteristics) to be considered.
3 Describe the risks in terms of the attributes.
4 Decide who is going to do the ranking.
5 Perform the rankings and combine the rankings for the different attributes.
6 Summarize the results.
Table 15.2 is one example of a Stage 1 output, albeit for a large agency with a very
broad remit. Ranking exercises need not be as demanding as this. Stage 2 is about
the assessment criteria (attributes/characteristics). An important issue here is who
should identify the criteria. If left to technical experts, there will be, as you have
seen in Chapter 12, a tendency to focus on fatality rates and possibly little else, but
Environmental risk ranking 197
if the aim is to engage the wider community, other criteria too may be appropriate.
Also, what is included depends on the aim of the ranking process. For example,
consider Table 15.5 which originates from the government of the State of Vermont
(1991) and which aimed to answer the question: ‘What environmental problems
pose the most serious risks to Vermont and Vermonters?’
The criteria listed are known as ‘Vermont’s quality of life criteria’, and are a consoli-
dation of what the people of Vermont said were important to them. Alternatively,
Table 15.6 shows a much simpler, four-dimensional, ranking system which was
devised by Granger Morgan and colleagues (1995) for use by government agencies.
ActivityComment
15.4
on the attributes listed in Tables 15.5 and 15.6. In particular, how do they
seek to accommodate public and expert concerns?
Feedback
The Vermont attributes were provided by the public in consultations. One feature of
interest is that health risks are not listed as a top level criterion although they are
included through ‘peace of mind’ and ‘economic well-being’, which are both linked with
the overall health of the population. There is a considerable emphasis on economic
factors, and on equity in the form of the distribution of costs and benefits in the current
generation and between it and future generations. The criteria in Table 15.6 were
clearly designed with something quite different in mind. They were in fact devised by
decision experts and their content suggests they are aimed at building a bridge between
health and environmental experts, who would mainly be concerned about threats to life
(and ecology), and public opinion which, as Slovic’s work shows, is influenced by qualita-
tive factors summarized in Table 15.6 as ‘knowledge’ and ‘dread’.
Having agreed upon the assessment criteria, Stages 3 to 5 of the process require
each of the risks to be rated against each of the assessment criteria. This requires,
first, a scale, qualitative or quantitative, to have been devised for each criterion. It
also needs to be decided who will do the rating. Normally, though, this stage is seen
as a task requiring expertise of some kind: health, ecological, economic, ethics, etc.
Stage 5 also refers to combining the scores across the criteria. To do this, it is
necessary to decide upon weighting factors for the criteria, unless all have equal
weight, which is unlikely. This is a value-laden task and it may be felt appropriate
to engage the affected community in this task if not to place it entirely within their
jurisdiction. The method for computing the final score, and hence ranking, for
each risk can be accomplished by a variety of means. A simple and commonly used
approach is to use the following formula:
Combined score for a hazard = (score on attribute A × weighting factor for A)
+ (score on B × weighting for B) +. . . .
for as many attributes as are described. Though the process appears outwardly
simple, it has many subtleties and needs care and thought in its application (Keeney
and Raiffa 1976; Kiker et al. 2005; DTLGR 2001).
At the beginning of this chapter the idea was advanced that environmental risk
ranking might overcome or avoid some of the objections which have been associ-
ated with the other approaches to decision making and prioritization described
earlier. The abridged passage that follows, by US attorney Frederick Anderson
(1996), gives reasons why the process of comparative risk assessment is viewed as
necessary and why nonetheless it still encounters resistance.
Environmental risk ranking 199
ActivityWhile
15.5
reading, focus on and think about:
1 the arguments for and against comparative risk assessment (referred to as ‘com-
parative risk analysis’ in this passage);
2 whether comparative risk assessment truly represents an advance or is merely a
revamping of the status quo;
3 by what means benefits of comparative risk assessment might be realized.
Feedback
1 The argument for comparative risk analysis is, as with benefit–cost analysis, that it
forces you to be honest about resource utilization and the implicit trade-offs. The
arguments against are not new either. If your motivation is to ban something outright, or
to be a scientific sceptic, then comparative risk analysis, which furthers compromises, is
not for you. Furthermore, if you concede that risk analysis has something to offer, then
the balance of power might shift away from environmentalism and towards science.
200 Other approaches
2 Is comparative risk analysis something new, or just something old in fresh clothes?
The answer is probably that it depends. It is clear that the technique can take on board
other dimensions that people care about, but, like all ‘sophisticated’ tools, it needs to be
handled skilfully and with care. It could either assist decision making, including making it
better both technically and more democratic, or it could hinder it.
3 Steps to help realize this dream would include: the use of credible, neutral science in
comparative risk assessment; carrying out the process transparently; involving affected
parties early; and trying to build consensus.
Summary
References
Ames BN, Magaw R and Swinsky Gold L (1987) Ranking possible carcinogenic hazards. Science
236: 271–80.
Anderson FR (1996) CRA and its stakeholders: advice to the Executive Office, in Clarence-
Davies J Comparing Environmental Risks: Tools for Setting Government Priorities. Washington,
DC: Resources for the Future.
Clarence-Davies J (1996) Comparing Environmental Risks: Tools for Setting Government Priorities.
Washington, DC: Resources for the Future.
Department for Transport, Local Government and the Regions (2001) Multi-Criteria Analysis: A
Manual. London: DfTLGR.
Gold LS, Slone TH, Manley NB and Ames BN (2003) Misconceptions about the causes of cancer.
Vancouver: The Fraser Institute.
Health and Safety Executive (1997) Risk Ranking. Report No. 131/1997. Sudbury: HSE Books.
HM Treasury (2005) Managing Risks to the Public: Appraisal Guidance. London: HM Treasury.
Keeney RL and Raiffa H (1976) Decisions with Multiple Objectives: Preferences and Value Trade-Offs.
Cambridge: Cambridge University Press.
Kiker GA, Bridges TS, Varghese A, Seager TP and Linkov I (2005) Application of multicriteria
decision analysis in environmental decision making. Integrated environmental Assessment
and Management 1(2): 95–108.
Morgan MG, Fischhoff B, Lave L and Fischbeck P (1995) A proposal for ranking risk within
federal agencies, in: Clarence-Davies J Comparing Environmental Risks: Tools for Setting
Government Priorities. Washington, DC: Resources for the Future.
Strategic Advisory Board (1990) Reducing Risk: Setting Priorities and Strategies for Environmental
Risk. Report No. EPA-SAB-EC-90-021. Washington, DC: US EPA.
US EPA (1987) Unfinished Business: A Comparative Assessment of Environmental Problems.
Washington, DC: US EPA.
Vermont Agency of Natural Resources (1991) Environment 1991: Risks to Vermont and Vermonters.
Waterbury, Vermont: VANR.
Alternatives assessment
16 to precaution
Overview
In the previous two chapters you have read about a number of alternatives, or
supplements, to risk assessment. In this chapter you will review another approach,
known as alternatives assessment, and consider how it stands in relation to those
already considered. You will also explore the nature and significance of the fre-
quently cited ‘precautionary principle’.
Learning objectives
Key term
Alternatives assessment A process that involves looking at the risks, benefits and other pros
and cons of a broad range of environmental health policy options.
Environmental risk ranking and comparative risk assessment are, as you saw in
Chapter 15, new initiatives aimed at helping society prioritize environmental
actions while taking on board broader concerns than those normally considered by
conventional risk assessment and cost–benefit analysis. But do they go far enough?
ActivityThe
16.1
following passage by Mary O’Brien (2000), consultant on alternatives to risk
assessment, criticizes the sufficiency of these initiatives. In reading the passage, think
about these questions:
202 Other approaches
Feedback
1 The argument is similar to ones you read elsewhere in this book, namely, that the act
of doing risk assessments, comparative or otherwise, could be seen as a move which
will necessarily lead to the legitimization of some hazards. It begs the question of
whether the hazard, whatever its size, could be avoided in its entirety, and thereby
undermines more thoughtful approaches that might be used.
2 There is no shortage of examples of where a narrowly selective approach to assessing
individual risks could result in people taking their eye off the wider possibilities. What
Mary O’Brien is proposing is a broader-based initiative that would permit a search for
common roots of multiple problems, which might thus have a common solution. The
problems associated with pesticide use in agriculture, for example, give rise to occu-
pational hazards, chemical residues in foodstuffs, public health hazards, and ecological
risks. All of these could be addressed by consideration of alternative agricultural prac-
tices which avoided or minimized pesticide usage, such as organic farming or integrated
pest management.
3 It might well be that in some circumstances there is a preference by some stakeholders
Alternatives assessment to precaution 203
for a narrower view to be taken. Some environmental health policies which would
emerge from an O’Brien-style approach would require much more than, say, ‘straight-
forward’ technological fixes. They might even require significant social change, and
some would say, disruption. This could run into difficulties with all kinds of interests –
professional, commercial, institutional and political. This is not to say that environ-
mental health policy makers should avoid contemplating such options, though they
should be aware that changes of that kind might introduce new health risks. As the
British Medical Association (1998) has said, policies themselves may introduce their
own health risks.
Alternatives assessment
In place of risk assessment, Mary O’Brien advocates what she calls alternatives
assessment. Her argument is that while many of those who employ risk assessment
for health policy purposes are doing their utmost to bring more evidence, candour,
and realism to the decision process, the ‘frame’ may not be large enough. As she
puts it:
Are you being allowed the room to learn about and calculate the pros and cons of all
reasonable alternatives to the activity whose risk you are calculating? Risk assessment is, to
some degree, dependent on the motives, assumptions, data bases, and intent of the risk
assessors, but not as much as many people would hope. This is because risk assessment is
fundamentally played out on a highly structured turf. The game played on this turf is to
estimate, with more or less information and candour, how much of a hazardous activity is
safe, insignificantly harmful, or acceptable. Alternatives assessment is a different game,
played on different turf. The goal is to gather information from many people about the pros
and cons of a variety of different paths to a goal. The goal itself is conceived as broadly as
possible. If you are a skilled, honest, committed risk assessor (or policy adviser), you will be
able to do much more to evoke needed information for good decisions about public health
and the environment over on the turf of alternatives assessment.
Table 16.1 Some benefits and disadvantages of the use of rbGH versus rotational grazing
Issue rbGH strategy Rotational grazing strategy
Animal health increased incidence of mastitis better health than in a confined
feeding system
Human health more antibiotics needed to treat no new health risks
mastitis
Economics competitive under conditions of competitive under conditions of
high milk prices, low feed costs, low milk prices, high feed costs,
low interest rates high interest rates
Viability of rural benefits larger units promotes community self-reliance
communities
Environment higher use of nitrates and herbicides less demand for fuel and chemicals
Source: Based on O’Brien (2000)
ActivityRead
16.2
the following edited passage from O’Brien which summarizes the pros and cons
of rbGH and a radically different means of achieving similar results, rotational grazing,
and also consider Table 16.1. In so doing:
1 Make a note of which issues might be dealt with in a conventional risk assessment
and which might not.
2 Think of other issues which might be important to farmers and communities.
3 Identify any lessons about stakeholder consultation.
Feedback
1 Some of the questions raised, like those about the incidence of mastitis and the
transfer of traces of veterinary drugs to human consumers are the types of questions
upon which a typical risk assessment would focus. However, issues of the effect upon
farmers’ lifestyles, the long-term impact upon pasture quality and ground water, and
traditional community would likely be omitted. Changes in pesticide usage might also
be overlooked, since they are an indirect consequence of rbGH usage.
2 Farmers and their dependants would probably also be concerned about the disrup-
tion of practices which may have shown themselves to be sustainable over long periods
of time, and the potential threat this would pose to traditional lifestyles. Farmers would
also become more dependent upon and therefore vulnerable to external suppliers, and
this would introduce a quite different form of risk in a process sometimes known as
‘risk substitution’ (considered in Chapter 20).
3 The example draws attention to the fact that interventions of this kind – the poten-
tial substitution of traditional practices by modern technology – can have very diverse
effects upon the ecosystem and upon community lifestyles and hence health and well-
being. The implication is that consultation needs to be wide, in order to draw in as many
stakeholders and affected parties as possible, so that these issues may be recognized
and understood, and all available knowledge brought to bear.
The debate over the relative merits of alternatives assessment and risk assessment
also impinges upon cost–benefit analysis. Nicholas Ashford and Charles Caldart
(1991), whose field is technology and public policy, have described three main
advantages which are frequently assigned to cost–benefit analysis as a decision-
aiding tool:
ActivityIn16.3
the following abridged extract, Mary O’Brien (2000) describes how alternatives
assessment differs from this view of cost benefit analysis. As you read it, consider the
following questions:
1 What are the strengths of the argument?
2 How might the argument be challenged?
3 Overall, where in your opinion does this leave risk assessment and cost benefit
analysis?
206 Other approaches
Alternatives assessment
Alternatives assessment, like cost–benefit analysis, clarifies choices among alternatives
by evaluating consequences in a systematic manner. The consequences (the costs and
benefits) of alternatives assessment, however, can include issues of democracy, aesthetics,
spiritual values, ethnic values, uncertainty, sense of community, and personal feeling as
well as monetary consequences. What constitutes a ‘systematic evaluation’ of these con-
sequences, of course, is a judgment. What may be crucially important to some may be
marginal to others. Considering and comparing monetary consequences with aesthetic or
cultural consequences may not appear as ‘systematic’ as comparing dollars to dollars, but it
is more realistic than assuming that all significant consequences can be systematically
translated into dollars. To speak of making decisions ‘rationally’ on the basis of money is to
deny the reality that much that matters has no price tag.
Furthermore, like cost–benefit analysis, alternatives assessment fosters an open and fair
policy-making process by making explicit the estimates of costs and benefits of the options
being considered and the underlying assumptions. The costs and benefits considered in
alternatives assessment, however, are more comprehensive. In alternatives assessment,
costs and benefits are both monetary and non-monetary. An alternatives assessment to
which the public contributes will be much more candid than a cost–benefit analysis,
because many kinds of relevant pros and cons will be displayed for public consideration.
The third ‘advantage’ of cost–benefit analysis, i.e. the representation of the total policy or
action by a ‘common metric’ of dollars, is not shared with alternatives assessment, because
social and environmental realities cannot be reduced to a common metric such as dollars.
Again, the tidy appearance of a common metric can be created only by denying social,
political, spiritual, and environmental realities.
How, then, can we decide among the alternatives if we have to consider numerous groups
and numerous factors, some of them financial, some cultural, some spiritual? The same way
each of us makes decisions each day. Our decisions are sometimes based on sheer survival
instinct, sometimes on overriding economic reasons, and sometimes on personal principles.
Occasionally we choose one alternative because another simply doesn’t seem the right
one. Sometimes we choose an alternative extremely systematically, on the basis of explicit
factors, after careful consideration of numerous options and after consultation with others.
Although both ‘objective’ cost–benefit analysis and messier alternatives assessment are
political, cost–benefit analysis covers up the political nature of analysis with monetary
numbers. Alternatives assessment forces the decision maker to assume responsibility for
choosing among various explicit political and value tradeoffs. At the heart of good public
decision making, governmental, business and corporate decision makers must take
responsibility for their decisions that affect the public and the public’s environment. They
must be prevented from hiding behind ‘dollar’ numbers, just as they must be prevented
from hiding behind ‘risk’ numbers.
Feedback
1 The thrust of the argument is that, for some environmental policy decisions at least,
there are wider, non-monetizable issues like spiritual and ethnic values that are important
to people and which therefore need recognition and evaluation, and would benefit from
public consultation. It is undeniable that things such as these are beyond monetization.
Alternatives assessment to precaution 207
ActivityMake
16.4
notes on the following:
1 The overall primary intention of the Precautionary Principle as revealed by the
statements.
2 Any clauses that moderate the primary intention.
Alternatives assessment to precaution 209
Feedback
1 The common intention is to tackle significant environmental threats, in advance,
without waiting for complete scientific proof of cause and effect. The onus is placed
upon polluters to provide evidence that their actions will not cause harm.
2 The UN World Charter for Nature contains the qualifying clause ‘expected benefits
outweigh potential damage’, the Rio Declaration refers to the precautionary approach
being applied by States ‘according to their capabilities’, and the UK Government mod-
erates the principle with ‘if the likely balance of costs and benefits justifies it’. All of
these imply, with varying levels of intensity, that the Principle will still be subject to
rational, economic analysis, involving cost–benefit considerations. It is also interesting
to note that there is reference to ‘absolutely clear scientific evidence’ (North Sea
Conference) and cause-and-effect relationships that are ‘not fully established scientific-
ally’ (Wingspread Conference). This opens up an additional debate as to the meaning of
the Principle. What qualifies as absolute evidence? Indeed, does science set out to
produce absolute evidence? Even Newton’s laws of motion and gravitation are not seen
as absolute statements of unshakeable truth, but as hypotheses, and ones which have
had to be modified as a result of work by Einstein.
Per Sandin (1999) goes on to analyse the Principle, as stated by the 17 agencies, in
detail. He does this on the basis of the following general formulation of the
Principle: ‘If there is (1) a threat, which is (2) uncertain, then (3) some kind of
action (4) is mandatory.’ Sandin observes that the phrases in the 17 statements
which express these four dimensions vary in (a) precision and (b) strength, and
that the weakest phrase is the one which determines the strength of the entire
principle. Thus, in the formulation of the UK government the clause ‘if the likely
balance of costs and benefits justifies it’ seriously weakens the ‘action’ dimension
and hence the Principle itself. The Rio Declaration says nothing about what
actions are to be carried out, but only that uncertainty is not a justification for
inaction, amounting, as Sandin says, to a rule for legitimate discourse rather than
for policy advice.
ActivityThe
16.5
following abridged passage from Cass Sunstein (2002) describes the obvious
emotional appeal of the precautionary principle, but also sounds a warning for policy
makers. What is the basis of that warning and what is being implied as the remedy?
There is some important truth in the precautionary principle. Sometimes it is much better
to be safe than sorry. Certainly we should acknowledge that a small probability (say, 1 in
100,000) of a serious harm (say, 100,000 deaths) deserves extremely serious attention. It
is worthwhile to spend a lot of money to avoid that risk. The fact that a danger is unlikely
to materialise is hardly a good objection to regulatory controls. But everything depends
on the size of the investment and the speculativeness of the harm. Unless the harm would
be truly catastrophic, a huge investment makes no sense for a harm that has a one in
one billion chance of occurring. Taken literally, the precautionary principle would lead
to indefensibly huge expenditures, exhausting our budget well before the menu of options
could be thoroughly consulted. If we take costly steps to address all risks, however improb-
able they are, we will quickly impoverish ourselves.
But there is a larger problem. The precautionary principle can provide powerful guidance
only if we blinker ourselves and look at a subset of the harms involved. In real-world
controversies, a failure to regulate will run afoul of the precautionary principle because
potential risks are involved. But regulation itself will cause potential risks, and hence run
afoul of the precautionary principle too. Hence the precautionary principle is literally
paralyzing. It bans every imaginable step, including inaction itself.
Feedback
Cass Sunstein’s warning to policy makers is that even though precaution is a worthy
consideration, it is still necessary to think carefully about resource allocation in the
interests of public health and also to be aware that even precautionary interventions
will, if thoroughly examined, be seen to generate secondary risks, just as medicines
have side-effects, which introduces a logical inconsistency. The implication is that while
being ‘precautionary’ has obvious political appeal and may at times be very sensible,
analysis of costs and benefits is still required. In a nutshell, one might adapt Samuel
Florman’s (1987) aphorism, ‘the precautionary principle is a sometimes useful tool that
needs to be applied thoughtfully, not a machete to be flailed against every conceivable
harm’.
Summary
Alternatives assessment seeks to extend the scope of risk assessment, which is still
seen as necessary although of reduced importance, to cover the risks and benefits
of a range of options to control target risks, some of which might involve social
rather than technical change. It also seeks to encompass broader issues than those
normally considered quantifiable in cost–benefit analyses. There are competing
arguments about the novelty and utility of this – supporters of traditional cost
benefit maintain that they do not at all preclude consideration of broader issues,
nor seek to confine the agenda of environmental health policy makers.
Yet another challenge to conventional thinking comes from advocates of the
‘precautionary principle’. Thoughtful analysis of this principle shows that it is
interpreted in many different ways, some interpretations falling back on cost–
benefit analysis, and some being much more demanding of evidence of no harm. It
is evident that useful though the concept undoubtedly is in some circumstances, it
needs to be handled carefully by policy makers.
Alternatives assessment to precaution 211
References
Arrow K et al. (1996) Benefit-Cost Analysis in Environmental, Health, and Safety Regulation.
Annapolis: The Annapolis Center.
Ashford N and Caldart C (1991) Technology, Law and the Working Environment. New York: Van
Nostrand Reinhold.
British Medical Association (1998) Health and Environmental Impact Assessment. London:
Earthscan.
Florman SC (1987) The Civilized Engineer. New York: St Martin’s Press.
Graham JD and Wiener JB (1995) Risk Versus Risk: Tradeoffs in Protecting Health and the Environ-
ment. Cambridge, MA: Harvard University Press.
London Dumping Convention (1972) Convention on the Prevention of Marine Pollution by Dump-
ing of Wastes and Other Matter. http://www.londonconvention.org/
O’Brien M (2000) Making Better Environmental Decisions: An Alternative to Risk Assessment.
Cambridge, MA: MIT Press.
Sandin P (1999) Dimensions of the precautionary principle. Human and Ecological Risk Assess-
ment 5(5): 889–907.
Sunstein CR (2002) Risk and Reason: Safety, Law and the Environment. Cambridge: Cambridge
University Press.
SECTION 6
Making policy
From risk communication
17 to participatory
decision making
Overview
As the story unfolds, you will have noticed that more and more emphasis is being
given to factors and concerns which some consider to have fallen outside the pur-
view of traditional health policy decision making. One way of redressing this bal-
ance would be to find out what people care about in affected communities, what
their problems are, and what insights they might have. In this chapter you will
learn about how these ideas have impacted upon the traditional approach to deci-
sion making, and in particular the now perceived central role and importance of
risk communication.
Learning objectives
Key terms
Analytic-deliberation A process of decision making that involves both technical analysis and
engagement with affected and interested parties.
Deliberation A reciprocal exchange of ideas between the public, interest groups and policy
makers.
As you have progressed through this book you will have noticed an undercurrent
of change in the way risks are perceived and managed and this is also increasingly
reflected in the ways in which policies are formed. In fact, in many quarters, there
is a strong trend towards involving stakeholders and other interested persons in
decision making and policy formulation. This is reflected in the growing use of the
term ‘governance’ over the last decade in many countries and international organ-
izations. Governance refers to structures and processes that support collective deci-
sion making involving stakeholders and other interested parties. This trend has
been brought about by a number of factors. Partly it is because there has been a
growing feeling that many risks have not been well managed, and this has resulted
in a loss of trust in conventional, political and technocratic approaches to decision
making.
Additionally, a very public battle has been fought over how risk management, and
policy decisions, should be made. As you read in Chapter 12, psychological studies
have shown us that the wider public perceive risks in different ways from profes-
sionals, and in Chapter 13, on cultural theory, you encountered the notion of the
legitimacy of the different world-views that are held by different groups. These
disparate interpretations of the evidence, culminating in different ideas about
what should be done, further reflected themselves in the proposed alternative
strategies for addressing health risks, such as social impact assessment, strategic
environmental assessment, health impact assessment, alternatives assessment,
precautionary approaches, and so on. These alternatives seek, in part, to change the
agenda, making it more encompassing of these other perspectives.
In 1995, Baruch Fischhoff, of Carnegie Mellon University, produced a very insight-
ful account of how risk management, in its attitude to the wider community, had
changed over the preceding 20 years. This trend is summarized in Table 17.1.
ActivityExamine
17.1
Table 17.1 and make a note of what you consider to be its key features.
Feedback
Table 17.1 suggests that within the constituency being considered by Baruch Fischhoff,
decisions about risk management had initially been made on the basis of technical
considerations without a perceived need to engage with the wider community, a mani-
festly technocratic approach which could be described as paternalistic. As time passed,
however, it became increasingly necessary to inform, explain, convince and win over
those affected by the decision, culminating in a need to involve them, on an equal
footing, as partners in the policy process. The last line of Table 17.1 is important too – it
shows that the intention is that whatever the level of public involvement deemed
appropriate, the technical input is undiluted and essential.
William Leiss (1996) has described the evolution of risk communication as having
occurred in three phases. The first phase emphasized the necessity of conveying
numerical information on risk to the general public and educating them over the
need to accept the risk management practices of the respective institutions. If
doubts remained, the main strategy was to argue that if people were willing to
accept a voluntary risk of a certain magnitude, then they should accept an equal, or
smaller, imposed risk. Of course, we now know from the psychometric paradigm
that this is unlikely to work, and so a second stage of risk communication emerged
in which the emphasis switched to persuasion and public relations exercises. This,
essentially one-way communication process, was also found to be unconvincing
for many people, and led to phase three which is often described as two-way com-
munication in which experts are expected to learn from the public as well as the
public from experts.
This shift in opinion has had a very real impact upon the way in which the policy
process is perceived in many countries and organizations. You will remember, from
Figure 3.2, the classical approach to risk assessment and policy formulation as
advised by the US National Research Council (NRC) in 1983, and which was fairly
universally accepted at the time. This approach is best known for the way in
which it drew a sharp distinction between risk assessment, seen as a technical
function, and risk management, seen as a policy function which transcends sci-
ence. However, as noted above, times and perspectives have changed, and the
NRC has revised its proposed methodology accordingly, and this is illustrated in
Figure 17.1.
ActivityExamine
17.2
the process set out in Figure 17.1 and compare it with that in Figure 3.2. What
is your opinion on the following?:
1 In what ways has it changed?
2 From what you have read in this book so far, what might be the justifications for the
changes?
218 Making policy
Feedback
1 There are several striking differences. Figure 17.1 clearly identifies the process of
decision making as involving two interconnected threads, of which scientific analysis is
one, and deliberation is the other (this is why the process is termed analytic-
deliberation). Second, inputs to the process are shown as coming, not just from labora-
tory and field measurements and other technical data, as indicated in Figure 3.2, but also
from public officials and interested and affected parties. Furthermore, all parties are
involved throughout the entire decision process in the exchange of ideas, information
and opinions. Thus, everyone can (and should, according to this view) learn from every-
one else at every stage of the process from framing to decision making.
2 Arguments in favour of this shift are of several types. In fact, Table 14.1 set out one
way of looking at it. In Table 14.1 three justifications (normative, epistemological, and
instrumental) were given for broadening participation, in that case in the EIA process,
but the same can be applied to risk and policy decisions more generally. To recap, the
justifications are that it is in the interests of democracy and because decisions are not
value-free; that decision makers can learn from affected communities and stakeholders;
and that broadening participation helps win acceptability of resulting policy decisions
(Löfstedt 2005).
The NRC also redefines risk characterization. Their new definition is as follows:
Risk characterization is a synthesis and summary of information about a potentially haz-
ardous situation that addresses the needs and interests of decision makers and of inter-
ested and affected parties. Risk characterization is a prelude to decision making and
depends on an iterative, analytic-deliberative process.
This definition too would appear to heed the concerns expressed by sociology,
psychology, and exponents of the wider vision about how to go about engaging in
policy decision making on health and the environment. It brings the affected
communities into the decision process from the very beginning (the issue framing
stage) and permits values to enter into the process.
This vision is reaffirmed in a recent White Paper on risk governance by the Inter-
national Risk Governance Council (IRGC 2005) in Geneva (Figure 17.2).
From risk communication to participatory decision making 219
ActivityStudy
17.3
Figure 17.2 and consider the following:
1 The positioning and nature of risk communication.
2 How the standard model of risk assessment relates to the scheme.
3 The content of the pre-assessment phase.
4 How wider concerns are fed into and assessed by the process.
Feedback
1 Risk communication is two-way and permeates the entire cyclical process, from
pre-assessment to decision making.
2 The standard model, as described in Chapter 3, is located in the risk appraisal phase.
3 The pre-assessment phase assumes greater importance and takes a broader view of
issues and concerns of all those involved and potentially affected by the decision.
4 Concern assessment is an integral feature of the process, appearing as a part of the
risk appraisal stage, and thus takes its place in this model alongside scientific, economic,
social and cultural aspects. Concern assessment here refers to insights from risk per-
ception studies and interdisciplinary analyses of the target risk’s indirect social and
economic implications.
220 Making policy
All of this is not to say that every policy decision requires copious public engage-
ment. This could be expensive, time-consuming, unwarranted and even counter-
productive. Risk communication, especially dialogue-based risk communication, is
not necessarily the be-all and end-all of risk management (Löfstedt 2005). It is
necessary to proceed on a case-by-case basis, and the optimum approach will be
influenced by the level of trust which exists between the affected community and
the policy maker.
ActivityNonetheless,
17.4
the following passage, adapted from the NRC (1996), explains in more
detail the intention behind the new vision of decision making. As you read it, note how
NRC distinguishes between different types of decision for the purpose of deciding
upon the appropriate level of public and stakeholder engagement. Do you think that
this would meet the objections of proponents of social impact analysis and alternatives
assessment?
ADifferent
new vision of decision making
kinds of decisions require different kinds and levels of analysis and deliberation in
support of risk characterization. For a series of similar risk situations, one might establish
routines for risk analysis, characterization, and decision making that embody clear and
consistent expectations about how the problem is defined, which options are to be con-
sidered, what kinds of evidence are to be considered, who is to participate in the process,
and so forth. For novel, complex, or highly controversial risk situations – which often
involve questions about major potential impacts and the equity of the distribution of risks
and benefits – routines are likely not to be satisfactory. It is likely to be necessary to
develop unique procedures for characterizing risk in these situations.
Some examples of procedures involving repetitive decisions are those for re-approving
existing permissions for discharge of pollutants from industry, for testing new drugs prior to
approval decisions, for issuing pre-manufacturing approval for the industrial production of
new chemicals, and for deciding whether to exclude an individual from receiving a vaccine or
giving blood. We do not mean to imply that all of the current procedures for these and
similar decisions are appropriate; only that it is often appropriate to develop standard pro-
cedures. In fact, situations and knowledge change even for routine decisions, and standard
procedures for risk analysis and characterization should be re-evaluated from time to time.
Risk situations need to be accurately diagnosed to determine whether existing standard
procedures should be applied, whether new procedures need to be devised, whether
additional information is needed to decide which approach to follow, and what extent and
type of analytic and deliberative effort may be needed to reach such decisions. In medicine,
experienced clinicians use a combination of knowledge, experience, and judgment to make
diagnoses. The situation is closely comparable for those who must diagnose a risk situation
and prescribe the appropriate kinds and level of analysis and deliberation needed and the
appropriate breadth of participation.
From risk communication to participatory decision making 221
Feedback
The NRC’s recommendation is essentially that for routine, simple and uncontroversial
hazards, the ‘old’ form of technocratic decision making, based upon routine pro-
cedures founded in experience and expertise may suffice, though periodic review
of practices should still be undertaken. In the case of controversial or complex
hazard situations, more emphasis should be placed upon engagement, consultation and
deliberation.
It remains to be seen whether this recipe will satisfy everyone, including critics of the
former risk assessment and EIA processes. However, it certainly looks like a step
towards meeting those concerns which have been expressed. It should also be expected
that not everyone will welcome this shift. For example, some people are firmly wedded,
and believe very strongly, that the so-called rational approach to decision making is still
the best way to make public policy decisions and that procedures which could dilute it
will ultimately waste resources and cost lives (e.g. Breyer 1993). This is a dilemma with
which environmental health policy makers will continually be confronted and to which
there is not an easy answer. We will return to this.
Much thought has been put into the further refinement of this issue of when and
how much engagement is desirable in policy decisions of various kinds. There are
no hard and fast rules, every hazard and its context being unique in some way, but
sociologist Ortwin Renn has produced a typology which is worthy of consider-
ation. This is shown in Table 17.2, and in Figure 17.3 where it is referred to as the
‘risk escalator’. Renn categorizes risk problems into one of four kinds: ‘simple’,
‘complex’, ‘uncertain’ and ‘ambiguous’ defined as follows:
1 Simple. Risk problems are ones for which the potential negative consequences are
obvious, uncertainties are low, and the values involved are non-controversial. They
are more-or-less straightforward to deal with and traditional methods of statistical
analysis, use of goals determined by legislation, and conventional risk manage-
ment techniques are applicable.
2 Complexity. Refers to the difficulty of identifying and quantifying causal links
and specific observed effects. This may be due to interactions between agents (syn-
ergisms), delayed manifestation of effects, inter-individual variability of response,
etc.
3 Uncertainty. Uncertainty is different from complexity but often results from an
incomplete or inadequate modelling of cause–effect relationships. There are vari-
ous sources: different vulnerabilities of targets; systematic or random errors in
modelling; randomness of events; factors which are outside of the boundary of the
modelled system but which might still be important; lack or absence of knowledge.
4 Ambiguity. This differs from the first two categories (which are epistemic in
nature and can be reduced by improving existing knowledge and improving mod-
elling tools). Ambiguity refers here to the different interpretations which may be
placed upon accepted risk assessment results, a phenomenon described by cultural
theory. There are two kinds of ambiguity. Interpretive ambiguity refers to different
interpretations of a given result, whereas normative ambiguity refers to the matter
of what is, or is not, tolerable.
222 Making policy
Table 17.2 Risk characteristics and their implications for decision making
Knowledge Management Appropriate instruments Stakeholder
characterisation strategy participation
1 ‘Simple’ risk Routine-based: →Applying ‘traditional’ Instrumental
problems (tolerability/accept- decision-making discourse
ability judgement) • Risk-benefit analysis
• Risk-risk trade-offs
(risk reduction) • Trial and error
• Technical standards
• Economic incentives
• Education, labelling,
information
• Voluntary agreements
2 Complexity-induced Risk-informed: (risk →Characterising the Epistemological
risk problems agent and causal available evidence discourse
chain) • Expert consensus seeking
tools:
• Delphi or consensus
conferencing
• Meta analysis
• Scenario construction,
etc.
• Results fed into routine
operation
Robustness-focussed: →Improving buffer capacity
(risk absorbing of risk target through:
system) • Additional safety factors
• Redundancy and diversity
in designing safety devices
• Improving coping capacity
• Establishing high reliability
organisations
3 Uncertainty-induced Precaution-based: →Using hazard Reflective
risk problems (risk agent) characteristics such as discourse
persistence, ubiquity etc. as
proxies for risk estimates
Tools include:
• Containment
• ALARA (as low as
reasonably achievable) and
ALARP (as low as
reasonably practicable)
• BACT (best available
control technology), etc.
Resilience-focussed: →Improving capability to
(risk absorbing cope with surprises
system) • Diversity of means to
accomplish desired
benefits
From risk communication to participatory decision making 223
ActivityWrite
17.5
down some examples of hazards which fall into each of the four categories.
Feedback
Examples of hazards with complex characteristics would include large-scale industrial
facilities and the synergistic effects of potentially toxic substances. Those with the
characteristic of uncertainty would include volcanic eruptions, earthquakes, and the
long-term effects of introducing genetically modified organisms into the environment.
Examples of interpretive ambiguity include the possible effects of low doses of ionizing
radiation and the use of antibiotics in animal husbandry. Normative ambiguities are
associated with passive smoking, nuclear power, and genetically modified food.
To return to Table 17.2 and Figure 17.3, what is being proposed here is that for ‘simple’
risks which are relatively free of the above attributes, the management strategy may
be routine and draw upon traditional decision making techniques of the types listed,
with risk communication having an instrumental aim, that is, educational or to
induce the desired response in affected parties. For ‘complex’ and ‘uncertain’ risks a
different approach is needed, which may be directed either at the hazard itself, or
those who are vulnerable. The following excerpt from IRGC (2005) explains:
For ‘complex’ and ‘uncertain’ risk problems it is helpful to distinguish the strategies
required to deal with a risk agent from those directed at the risk-absorbing system (those
affected or the environment): complex risks are thus usefully addressed on the basis of
‘risk-informed’ and ‘robustness-focussed’ strategies, while uncertain risks are better man-
aged using ‘precaution-based’ and ‘resilience-focussed’ strategies. Whereas the former
strategies aim at accessing and acting on the best available scientific expertise and at
reducing a system’s vulnerability to known hazards and threats by improving its buffer
capacity, the latter strategies pursue the goal of applying a precautionary approach in order
to ensure the reversibility of critical decisions and of increasing a system’s coping capacity
to the point where it can withstand surprises. Finally, for ‘ambiguous’ risk problems the
appropriate strategy consists of a ‘discourse-based’ strategy which seeks to create toler-
ance and mutual understanding of conflicting views and values with a view to eventually
reconciling them.
Figure 17.2 shows the centrality of risk communication in decision making.
ActivityWrite
17.6
down what you believe to be the purpose(s) of communicating about risk.
Feedback
Risk communication could have a number of purposes: enabling the public and stake-
holders to understand the thinking behind decisions when they are not formally part of
From risk communication to participatory decision making 225
the process; helping people who are faced with risks by giving them factual information
so that their choices are informed; fostering understanding of and hence tolerance for
conflicting points of view; creating trust; enabling society to cope with and react to
hazards of all kinds, whether acute or chronic in nature.
Risk communication is not a science but nonetheless, much has been learnt about
its proper (and improper, i.e. how not to do it) conduct over the years, sometimes
from bitter experience. Regina Lundgren (1994) has provided a valuable guide
which should be consulted. This identifies three primary forms of risk communica-
tion which she calls care communication, consensus communication and crisis
communication (Figure 17.4), each of which requires its own approach.
Care communication is about things like safety and environmental health. It can
be subdivided into health risks such as AIDS, smoking, and exposure to environ-
mental toxins, and, on the other hand, industrial exposure to chemicals or the risk
of accidents. Consensus communication is about informing and encouraging
groups to work together to decide on how to deal with a more complex hazard
situation such as the Nepalese Bara Forest plan described in Chapter 14. Crisis
communication refers to risk communication in the aftermath of major events
such as the 2005 earthquake in northern Pakistan, the Indian Ocean tsunami of
2004, or major industrial accidents such as those at Bhopal, Chernobyl and Prince
William Sound (the Exxon Valdez).
Of these, care and consensus communication are more likely to be of relevance to
environmental health policy makers, and crisis communication will not be exam-
ined further here. Lundgren has described many of the factors which would-be care
communicators should consider in preparing their communications:
• Principles of process
• Know your communication limits and purpose.
• Whenever possible, pretest your message.
• Communicate early, often and fully.
• Remember that perception is reality.
• Principles of presentation
• Know your audience.
• Don’t limit yourself to one form or style of communication.
• Simplify language and presentation but not content.
• Be objective and not subjective.
• Communicate honestly, clearly and compassionately.
• Listen to specific concerns and deal with them.
• Convey the same information to all interested parties.
• Deal with uncertainty.
• Principles for comparing risks
• Don’t trivialize.
• Use ranges where there is uncertainty.
• Compare to Standards.
• Compare to other persons’ estimates of the same risk.
226 Making policy
Figure 17.4 Three different realms of risk communication requiring different styles
Source: Lundgren (1994)
Activity117.7
What kind of communication is this primarily referring to?
2 In what circumstances might it be effective, and in what circumstances might it be
unproductive?
Feedback
1 The recommendations primarily apply to what might be called ‘top-down communi-
cation’. Scope is clearly permitted for interaction with audiences, but the emphasis is
more on presenting and listening than on deliberating.
From risk communication to participatory decision making 227
2 Top-down communication might well suffice and be the optimal approach in situ-
ations where there is trust between affected communities and policy makers, and
where the hazards are familiar and non-controversial. Where trust has been lost, or
hazards are complex or in some way ambiguous, the approach could lead to a further
deterioration in relations.
ActivityIn17.8
reading the following edited passage from his book, note the justifications given for
encouraging stakeholder and public involvement, and reflect on their compatibility with
the recommendations of Ortwin Renn as set out in the risk management escalator
(Figure 17.3).
Feedback
The justifications, which would also apply to policy makers, are to increase trust,
gain information on the interests of those affected, and secure legitimacy. These
ideas are compatible with the right-hand column of Renn’s risk management escalator
(Figure 17.3), which deals with hazards involving questions of ambiguity and personal
values.
Wouter Poortinga and Nick Pidgeon (2003) have analysed the nature of trust in
some detail and proposed that it is more clearly understood if divided into a
fourfold typology as shown in Figure 17.5, based on two dimensions, one being
the general level of trust in policy makers and the other the level of belief, or
scepticism, which surrounds a policy issue. They argue that it is important to dis-
tinguish between these different types of trust in any given situation. A potentially
important matter for environmental health policy makers is that ‘critical trust’, for
example, should not be confused with ‘distrust’, and may in fact be the healthiest
state of affairs to exist between policy makers and their constituencies!
228 Making policy
Deliberative approaches
Deliberative approaches
. . . the deliberative process is a relative failure. There are several reasons for this. Research
has shown that more deliberation in the policy-making process leads to public mistrust of
policy-makers and the government as a whole; they only become more aware of how staid
and ineffective bureaucrats can be. Some scientists and experts criticize deliberation, as
they see no reason why the public, whom they view as under- or misinformed, lost,
bewildered, overtly self-interested or simply apathetic, should participate in policy-making
at all. In addition, the popularity of the deliberative process in the 1990s has resulted in
more work for officials and a less efficient practice. Bureaucrats concerned about trans-
parency have now begun to retreat from public meetings, favouring memos and informal
discussions. The participatory process is also criticized for requiring the public to assimi-
late a lot of information about the particular issue being deliberated, which can lead to
divergent rather than convergent views.
Henry Rothstein (2004) has examined a number of participatory decision processes
in the United Kingdom, concluding that the broadening of participation per se
does not necessarily produce more democratic or robust policy outcomes than
From risk communication to participatory decision making 229
closed processes, although it may have some limited value in improving public
confidence in the regulatory regime.
As you will have surmised, there is no simple recipe in this chapter for environ-
mental health policy makers. Although techniques of risk communication are
in vogue in some countries, the matter of when, where and how to apply them
finds no simple answer. At the extremes, and to use a boxing analogy, in the
red corner we have people who espouse ‘technocratic risk management’ and
in the blue corner we have those who see any such approach as arrogant and
inherently undemocratic. Once released into the ring, pitched battles are apt to
break out.
US Supreme Court Justice Stephen Breyer (1993) is often cited as a defender of the
‘red corner’. In his 1993 book, Breaking the Vicious Circle, he expressed concern
about patchy and inconsistent approaches to risk assessment, and arguments
involving the public who, he felt, were unable to judge risks reliably. The Breyer
recipe was for a specific kind of group to take the lead in health policy decisions.
This group would be ‘mission oriented, seeking to bring a degree of uniformity and
rationality to decision making in highly technical areas, with broad authority,
somewhat independent, and with significant prestige’. To find an occupant of the
opposite corner, one need look no further than Mary O’Brien (of Chapter 16) who
describes Breyer’s vision as ‘nightmarish and undemocratic’.
With whom should the environmental health policy maker align herself or him-
self? Without wishing to raise your hopes unduly, perhaps the next chapter will
provide some ideas, if not answers.
Summary
Risk communication is now firmly on the agenda of health policy decision makers,
and the recognized need for it has influenced the decision making style of many
countries and agencies. It is not, however, a panacea, and it is necessary to think
carefully about the appropriate style of communication, whether top-down, par-
ticipatory, or something in-between. An important factor in deciding what is
necessary in a given circumstance will be the nature and complexity of the hazard
being considered, and the issue of trust. Trust is itself not a straightforward concept
and needs to be examined carefully on a case-by-case basis. Although the current
fashion in some sectors is very much communication-oriented, not everyone is
convinced and important questions remain unanswered.
References
Breyer S (1993) Breaking the Vicious Circle: Towards Effective Risk Regulation. Cambridge, MD:
Harvard University Press.
Fischhoff B (1995) Risk perception and communication unplugged: twenty years of process.
Risk Analysis 15(2): 137–45.
230 Making policy
IRGC (2005) Risk Governance: Towards an Integrated Approach. Geneva: IRGC. Available at:
www.irgc.org
Jaeger CC, Renn O, Rosa EA and Webler T (2001) Risk, Uncertainty and Rational Action. London:
Earthscan.
Leiss W (1996) Three phases in risk communication practice, in Kunreuther H and Slovic P
(eds) Annals of the American Academy of Political and Social Science: Challenges in Risk
Assessment and Management. Thousand Oaks, CA: Sage.
Löfstedt R (2005) Risk Management in Post-Trust Societies. New York: Palgrave Macmillan.
Lundgren R (1994) Risk Communication: A Handbook for Communicating Environmental, Safety,
and Health Risks. Columbus, OH: Battelle Press.
National Research Council (1996) Understanding Risk: Informing Decisions in a Democratic
Society. Washington, DC: National Academies Press.
Poortinga W and Pidgeon NF (2003) Exploring the dimensionality of trust. Risk Analysis 23(5):
961–72.
Rothstein H (2004) Precautionary bans or sacrificial lambs? Participative risk regulation and
the reform of the UK food safety regime. Public Administration 82(4): 857–81.
Philosophy, politics
18 and prejudice
Overview
By now you will be aware, if you were not before, that all is neither simple nor calm
within the world of environmental health policy. There are sharp divides between
those who advocate so-called ‘rational’ models of policy formulation and those
aligned to models with a wider ‘social’ emphasis. There is also disagreement over
who should make policy decisions, and especially the extent to which the public
should be invited to participate. While the near-term existence of a formula for
resolving these disputes is doubtful, and hence you might think there are grounds
for pessimism, an alternative strategy is to try to understand the roots of these
disagreements. Already you are familiar with two sociological theories, cultural the-
ory and the psychometric paradigm, which offer valuable insights. In this chapter
you will further examine the rifts from a philosophical and political perspective.
The aim is to achieve maximum insight into these fundamental differences so you
are better equipped to recognize available choices. You will also think about the
matter of ‘prejudice’ and its contribution to decision making and, in conclusion, the
essential and unforgiving challenge of environmental health policy formulation.
Learning objectives
Key terms
Communitarianism A philosophy that values communities and resists the idea that
individuals and free market exchanges are the key social institutions.
Utilitarianism (welfarism) Based on the notion that society’s interests are best served through
the maximization of individual utilities.
232 Making policy
Wider reflection upon how society functions with respect to decision making and
policy formulation has pointed to four basic needs: these are for efficiency, effective-
ness, social cohesion, and legitimacy (Renn 2005). Efficiency is achieved via eco-
nomic balancing; effectiveness is about ensuring that measures adopted will, with
reasonable expectation, yield the desired result, and derives from expert knowledge
and science (the technocracy); social cohesion refers to the need to incorporate
human values and fairness, and, as you saw in the previous chapter, can be addressed
by consultation and deliberation. It concerns the communication and exchange
of interests such that disparate values are recognized and fairness is achieved.
Legitimacy is about compatibility with legal requirements and political culture and
emanates from the political system. Within modern societies, these four systems
are maintained by economics, science, social systems and politics (Figure 18.1).
Most environmental health policy decisions take input from all four of these sys-
tems. This is partly at least because health policies designed to help one community
are likely to have impacts, possibly adverse, on others (BMA 1998; Graham and
Wiener 1995). In real societies all things are coupled together in some way, people
and institutions alike, and cannot be considered as independent atomistic entities
as in the idealized RAP model. It is also because each system is regarded as having
its own strengths and weaknesses, as summarized for three of the systems, in
Table 18.1, so by using them all there is more chance of success. As you read
through Table 18.1 you should recognize all the entries. This is because you have
already studied these three systems: in Chapters 3 to 7 you encountered the
technocratic approach to policy formulation as taken by science; in Chapters 8
Table 18.1 A comparison of the benefits and disadvantages which have been attributed to
three different policy approaches
Policy Cited benefits Cited disadvantages
approach
Technocratic – increases benefits of environmental health – sometimes seen as arrogant and
policies undemocratic
– gives credibility to policies – unlikely to work if trust is low
– reduces dependence on moral and legalistic – not all issues are about purely
arguments science (e.g. Bara Forest)
– the scientific approach has produced huge – knowledge is far from perfect and
health benefits uncertainty may be significant
– there is public support for the use of science – can be seen as a way of permitting
in policy decisions the continuance of risks rather
than avoiding them
– experts are fallible and results can
be manipulated
– even scientific decisions involve
value judgements
– risk assessment tends to
introduce conservative bias (by
repeatedly erring on the safe side)
Economic – the economic (market-based) approach is – the process is elitist and answers
more efficient than command and control ‘how safe is safe enough?’ without
– resources are saved direct recourse to the affected
– compliance costs are reduced public
– provides an incentive to develop better – equity issues are not considered,
control mechanisms nor are many other factors which
– exposes the conservative biases to which people care about, mainly because
risk assessments are prone they cannot be quantified
– exposes the biases to which the public and – it assumes that economists are
policy makers are vulnerable (e.g. that there right in the way they measure
is more concern for highly visible and public preferences (WTP etc)
dramatic risks than those which cause most – cost–benefit analysis downplays
harm etc) unquantifiable risks
– ignores social values e.g. altruism
– tends not to handle uncertainty
well
– the monetary valuation of health
is challenged
– measuring and aggregating public
preferences is difficult, even for
economists
Consultative/ – strengthens democracy by informing and – open to exploitation
deliberative involving the public – may subvert democracy
– gives legitimacy to decisions – can diminish trust
– brings on board issues of equity and other – may lead to relativism
non-quantifiable but important factors – costly and time consuming
– does not subjugate the issue of what level of – experimental and unproven
risk is tolerable to a mathematical formula – requires the public to assimilate
– makes use of publicly-held knowledge as well large amounts of technical
expert knowledge information
– dialogue is better than top-down risk
communication or no communication at all
– increases trust
Source: Based partly on Löfstedt (2005)
234 Making policy
to 10, the economic approach; and in Chapters 11 to 16, social factors increas-
ingly played a role including, in Chapter 17, the extra step to participatory and
deliberative decision making.
What has not so far been discussed is the political-philosophical dimension. In dis-
cussing this fourth system, the emphasis is going to be upon exploring how different
political philosophies underlie different approaches to decision making. You will
recall from Chapter 13 how cultural theory offered an insightful explanation of why
different groups respond differently to similar information. Another window on
this phenomenon can be elicited by examining the political roots of policy making.
First though, you need to think about the role of values in decision making. The
philosopher David Seedhouse, whose primary interest is health care, has this to say:
Health promotion is thought, by many theorists and practitioners, to be based primarily, or
even entirely, on evidence and facts. But this is a crucial mistake. Health promotion is much
more to do with values (and politics) than even the most radical health promoters realise.
(Seedhouse 2000)
Since health care policy and environmental health policy have much in common,
what would be true for one would presumably be true for the other. But is it true?
Seedhouse proposes that while most policy makers believe that when it comes to
policy it is the evidence which comes first, the truth is in fact the reverse. There is
thus a false perception that environmental health policies are formed in this order:
1 There is evidence of preventable environmental health problems (this evidence
is either factual or highly probable – and it is the main driver of policy).
2 There are strategies designed to deal with the preventable problems (there is
usually a choice of strategy, and values may be one of the factors influencing the
choice).
And in recent years:
3 There are additional strategies meant to protect the environment itself even
where there is no evidence of human health problems.
But instead, environmental health policies are formed in this order:
1 Policy makers hold particular values and political philosophies.
2 There is evidence of preventable environmental health problems (this evidence
is selected according to values and political philosophy – even the decision to
call something an environmental problem or not is thus inspired though this is
not to say the evidence is entirely shaped by prejudice – there can be better and
worse reasons to intervene.
3 Strategies designed to deal with preventable environmental problems are
selected according to values, political philosophy and evidence – the key
questions are: do we think this ought to be done and will it work?
And in recent years:
4 There are additional strategies to safeguard the environment even where there
are no existing human health problems.
Philosophy, politics and prejudice 235
ActivityIn18.1
policy decisions, is it the case that ‘facts always speak for themselves’? Or is it true
that policy makers interpret evidence according to their particular values and political
philosophies?
Feedback
It is evidently the case that environmental health promotion is not factual but essentially
value-based. This is demonstrated by the fact that priorities differ between different
administrations, between different groups, in different geographical locations, and at
different times, even when essentially the same problems are faced.
In fact, within this book, you have encountered in very general terms just three
political bases. Table 18.2 summarizes, in a much simplified form, what these are
about, under the three headings of ‘rational health policy’, ‘environmental health
policy’, and ‘social health policy’. Rational health policy would be anchored in
techniques like risk assessment and potential Pareto optimization, and is generally
assumed to be objective and evidence-based because it aims to improve measurable
aspects of people’s lives. On the other hand, social health policy sees life (society)
as more complex and seeks to improve people’s lives by social change. It is particu-
larly concerned with social inequalities, whereas rational health policy is more
interested in getting an appropriate health return for its investment. The third type
is environmental health policy. This comes in many shades, from relatively mild
to ‘deep Green’, and is directed more towards the protection of the ecosystem as a
whole than to people, though it may also be a part of the intention that people
will benefit if the ecosystem is healthy. Of course, the true picture is much more
complicated than this, nevertheless, it can be used as a starting point for under-
standing some of the controversies which permeate environmental health policy
formulation.
Table 18.2 indicates how each of these three policy types can be traced to an
underlying political philosophy. For example, you can trace how rational health
policy emanates from a political philosophy of prudence, utilitarianism, and
preservation of the status quo. Essentially, it is conservative in nature. Similarly,
links can be made between social health promotion and different varieties of
social democracy and egalitarianism. This political outlook is inspired by the
essential equality of all people, and the importance of communities as well as
individuals.
236 Making policy
Table 18.2 A simplified analysis of the possible political roots of three different approaches to
environmental health policy
‘Rational’ health policy ‘Environmental’ health policy ‘Social’ health policy
Environmental health exists in Environmental health is Environmental health exists
the absence of measurable about the well-being of the in the absence of threats to
disease, illness, injury and ecosystem health
handicap
Environmental health problems If the ecosystem is healthy Environmental health
are bad of themselves people will also have the threats are bad in
greatest chance to be healthy themselves
too
Environmental health problems Threats to the ecosystem are Environmental health
are also disruptive and incur also bad because they pose threats are also bad
losses of working time and risks to other creatures because they pose risks to
medical costs peoples’ potentials
Environmental health risks can Environmental health
be managed by appropriate threats are unequally
science-based interventions distributed across social
groups
The causes of
environmental health
threats are manifold but
sometimes they are the
result of how people have
to live
Environmental health The avoidance of threats to Where environmental
interventions should be the environment is our health problems are the
instigated where health responsibility. We should result of broader social
benefits exceed costs ‘tread lightly on the earth.’ issues health policy should
consider social change
PRUDENCE ENVIRONMENTALISM EGALITARIANISM
UTILITARIANISM SOCIAL DEMOCRACY
PRESERVATION OF THE SOCIALISM
STATUS QUO
CONSERVATISM
Source: Adapted from Seedhouse (2000)
ActivityAs18.2you read the following edited passage by John Graham (2001) on the economic
evaluation of health interventions and the political roots of decision making (specifically,
liberalism, which includes egalitarianism and utilitarianism) think carefully about the
following questions:
1 In Chapter 9 you encountered a framework for decision making in the context of
the control of radiation exposure, summarized in Figure 9.8, and in Chapter 10 you
read about the ‘tolerability of risk’ framework, summarized in Figure 10.2. Have
these frameworks been influenced by political philosophies, and if so, which?
Philosophy, politics and prejudice 237
2 In Chapters 14 and 16, you read about several alternatives to the Rational Actor
Paradigm for determining environmental health policy, including health impact
assessment, social impact assessment, and alternatives assessment. Can the evident
contest between these models and, say, RAP, be attributed to different underlying
political philosophies and, if so, how?
Feedback
1 Both of these frameworks for decision making contain a strong element of utili-
tarianism. This comes in via the concepts of ALARA and ALARP which are about
optimization and seeking the greatest good for the greatest number i.e. economic
efficiency. However, the frameworks also incorporate dose and risk limits above which
no person should be exposed, and in this way an element of liberalism-egalitarianism is
238 Making policy
introduced. This is why, in the right-most column of Table 8.1, where these frameworks
lie, they are referred to as ‘hybrid’ approaches.
2 It should be clear that whereas RAP is rooted in the left-hand column of Table 18.2
and is therefore underpinned by utilitarian beliefs and conservatism, social impact
assessment is more readily associated with the right-hand column and hence egali-
tarianism and social democracy. Health impact assessment, at least as described in
Chapter 14, remains essentially a RAP-style approach, merely aiming to incorporate
health considerations more overtly into the environmental impact assessment process.
Alternatives assessment, on the other hand, is much more concerned with social,
political, spiritual and environmental priorities than with economic optimization, and
seeks to take a wider societal view of environmental health policy and challenge the
status quo. It is therefore more strongly associated with the right-hand column of
Table 18.2, and, as expressed in Chapter 16, may have some anchorage in the central
column too.
Just as tools for policy makers may carry with them a political ideology, so do all
environmental health programmes.
Table 18.3 lists 18 targets of the Kazakhstan Ministry of Health. Although informa-
tion is limited, you might conclude from this list that these targets are of different
kinds, being anchored in different political philosophies and hence requiring dif-
ferent approaches to implementation.
ActivityExamine
18.3
the list and make a note of the more indicative tendencies of this kind, i.e.
whether some objectives are more clearly motivated by utilitarianism as opposed to
egalitarianism or environmentalism and vice versa.
Feedback
The first targets on the list are primarily egalitarian in nature, whereas from No. 9
onwards the tendency is for a ‘rational’ approach to health policy. Thus, the early
targets on the list talk about ‘reducing’, ‘preventing’ and ‘improving’ health without
reference to efficiency and item 8 refers to the intention to ‘develop new population
behaviour patterns and healthy lifestyles’, a clear indication of a preference for social
health policies. Items 11, 13 and 16, in contrast, are clear indicators of an interest in a
‘rational’ style of health policy, and some of the other items may be too, depending
upon how they are implemented.
This chapter has emphasized that people’s choices about which health policies to
pursue and how, are based upon values as well as, and arguably more than, facts.
Further, whereas you have read about the importance of psychology and culture in
shaping opinions, here the emphasis has been on the role of political philosophies.
This is not at all to say that scientific evidence is unimportant, for most people
would agree that it is. To deny the contribution of science would be to descend
into the fruitless morass of relativism. So, to make sense of the world, it is neces-
sary to collect the evidence and interpret it, the latter requiring a belief system or
philosophy against which to perform this task. However, it will often be the case
that better or worse interpretations can be identified, and sometimes it will be
possible to say that interpretations are completely right or completely wrong, in
either an analytic or a moral sense. The implication for policy makers is that they
should not be unaware of the roots of their choices and that, from time-to-time
at least, they should re-examine them. As John Adams might say, if you aspire to
making policy, you should endeavour to move up the insight axis (of Figure 13.4),
and that means seeing through the political biases or prejudices of others as well as
your own.
ActivityThe
18.4
following adapted passage from David Seedhouse describes three principal types
of prejudice. In reading, make a note of which is an appropriate form and which is not
for an environmental health policy maker. Does this offer any explanation of the rifts
between rational, social and environmental health policies?
240 Making policy
Feedback
Sooner or later prejudice has to enter into any deliberation about environmental
health policy, so environmental health policy will always be open to dispute. Thus, the
contest between rational, social and environmental policy models is a fact of life and not
just a temporarily unresolved difference of opinion. Cultural theory tells the same
story. Ask yourself: how can anyone seek to promote environmental health without
holding a prejudice of some kind? Can anyone promote health without having an
opinion about whether one way of living, and one way of intervening, is better than
another? It is not possible. The evidence does not speak for itself, but, remember, neither
is it mute.
It should also be obvious that only the first and third of the above forms of prejudice
should be countenanced by participants in environmental health policy.
Philosophy, politics and prejudice 241
Aside from the prejudices of policy makers, what about those of the public?
Environmental health policy makers are often in the position of making decisions
which affect the well-being of possibly many people to a far greater extent than
they affect themselves. These decisions can be ethically challenging because the
stakes are high and the value issues complicated. In this book you have encountered
a number of reasons why policy makers should incorporate public values and per-
ceptions into their decisions. However, while it may well be that incorporating
public input into policy decisions will usually lead to better decisions, this is not
guaranteed.
What should be done if the public object to the use of Decision Analytic techniques for policy
formulation?
People may object to the formal structuring and codification of complex decisions, but this
should be rejected firmly because social decisions are made under conditions of resource
constraints and trade-offs, like it or not, must be acknowledged even if it is claimed that
they are morally repugnant.
What risk attitude (risk averse, prone or neutral) should policy makers take?
Policy makers should be avowedly risk neutral because the more compelling moral obliga-
tion is to save as many lives as possible. If there are additional costs associated with
catastrophic events, these should be incorporated as additional factors in an analysis and
not as some form of risk aversion.
What if the individual preferences of the public differ from the societal policy perspective?
Again, this is a difficult scenario and one which would have to be analysed on a case-by-case
basis.
You would be correct in deducing from this that there are still very real challenges
for environmental health policy makers. The following passage, adapted from their
paper, acknowledges the difficulties and provides a supportive message:
Our purpose is to encourage others to join us in thinking about the ethical problems faced
by policy makers in making their decisions when their views differ from those of the
affected public. We feel compassion for the decision makers, struggling to do the right thing
in difficult circumstances. We want our policy maker to be intellectually well armed with an
understanding of decision analysis sufficient to ensure that complex social problems can be
viewed from a broad, consistent perspective. We also want our policy maker to have a
backbone, able to go against (while never ignoring) public desires, and a heart, caring for
and respecting (but not always acquiescing to) public views. This is a tall order, we admit.
These are tough problems, unavoidably so.
Summary
Environmental health policy decisions are driven as much if not more by values
than by evidence. These values can be shown to be anchored in different political
philosophies, and in turn offer an explanation of the on-going contests between
rational, social and environmental models of health policy. The selection of any
particular environmental health intervention will, in part, be based upon preju-
dice. Of the various forms of prejudice around, only some should be harboured
by policy makers. These are necessary prejudice and reasoned prejudice, and even
these should be reviewed periodically. Environmental health policy makers should
expect to continue to be confronted with many technical and ethical dilemmas.
It is the nature of the job and is unlikely ever to go away.
References
British Medical Association (1998) Health and Environmental Impact Assessment. London:
Earthscan.
Graham JD (2001) Economic evaluation of injury control: alternative perspectives, in Proceed-
ings of the Third International Conference on Measuring the Burden of Injury. Baltimore, MD: US
Department of Transportation.
Lichtenstein S, Gregory R, Slovic P and Wagenaar WA (1990) When lives are in your hands:
dilemmas of the societal decision maker, in Hogarth RM (ed.) Insights in Decision Making.
Chicago: University of Chicago Press.
Löfstedt RE (1995) Risk Management in Post-Trust Societies. New York: Palgrave Macmillan.
Renn O (2005) The challenge of integrating deliberation and expertise: participation and
discourse in risk management, in MacDaniels T and Small M (eds) Risk Analysis and Society:
An Interdisciplinary Characterisation of the Field. Cambridge: Cambridge University Press.
Seedhouse DF (2000) Health Promotion: Philosophy, Prejudice and Practice. New York: Wiley.
SECTION 7
Initiatives – local
to global
Issues of local policy
19
Overview
The final section of this book is about past and current environmental health
policy initiatives. While you will read about some of the many schemes which
have been devised, the intent is not to give a detailed résumé, but to identify
issues which are deserving of thought from an environmental health policy per-
spective. Many initiatives are driven by genuine concerns and this is what places
them on the political agenda, but to carry them through to fruition requires
more than a vision. It requires a will to make things work and staying power. There
are also many questions which should be asked of policies. What is their exact
purpose? What motivates them? What tangible evidence is there that they are
working? Can their hoped for effectiveness be measured? How much evidence
should you expect? In the first of these closing chapters, the focus is upon local
initiatives.
Learning objectives
Key terms
Agenda 21 A global plan led by the United Nations which seeks to integrate environmental,
social and health issues into decision making.
Capacity 21 and 2015 Capacity-building programmes which support Agenda 21 and the quest
for sustainability.
Social capital The institutions, relationships, and norms that shape the quality and quantity of
a society’s social interactions.
246 Initiatives – local to global
Policy routes
Very broadly speaking, there are two principal routes for instigating environmental
health policies. One is via so-called top-down initiatives which emanate from
central or regional governments or international organizations and which influence
central planning. The other is through stimulating bottom-up or ‘grassroots’ acti-
vities. Grassroots activities may of course arise spontaneously, but there has been an
acceptance that the gains which are potentially achievable by this route can be
realized more quickly with appropriate help from the centre.
You will also have sensed, in reading through this book, the gradual shift from
the way of thinking which saw problems as being solved by a centralized expert-
driven technocracy to a process in which the wider community could usefully
participate. Likewise, the view has taken hold that local communities, by making
use of their own ‘social capital’ could do much to help themselves, and might
even provide a solution to larger-scale problems if effective measures were repli-
cated across other communities. The term ‘social capital’ is interesting. Unlike
the economist’s concept of ‘human capital’, social capital could be described as
the sociologist’s counterfoil, namely, it is about the importance of social net-
works and trustworthiness in a properly functioning society. You might recall the
argument over RAP, which sees society as a collection of ‘atomistic’ individuals
pursuing their own individual interests, a view rejected by sociology. Thus, a RAP
society may contain many virtuous but isolated individuals, but could still be
low in social capital. To paraphrase the World Bank, social capital refers to the
institutions, relationships, and norms that shape the quality and quantity of a
society’s social interactions. Social capital is not just the sum of the institutions
which underpin a society – it is the glue that holds them together. In this way social
capital is said to be important because it allows citizens to resolve collective prob-
lems more easily, it oils the wheels that allow communities to advance smoothly,
and it widens awareness of the interlinking of people’s interests (Putnam 1995,
2000).
Sustainability implies the need to conserve both energy and material resources, to
maintain biodiversity and to reduce pollution to stabilize planetary conditions.
Note, however, that sustainability is notoriously difficult to quantify and there is
no consensus on what constitutes a sustainable state for the world. Currently the
richest 20 per cent of the world’s population consume 80 per cent of its resources.
This puts the burden of responsibility for change on the rich countries, though it is
clear that rising living standards in the rest of the world will have a large impact on
resource consumption. This raises both economic and ethical issues but
unfortunately these two disciplines are uncomfortable companions.
Issues of local policy 247
interdependence among the factors of health, environment and development and has
revealed that most countries are lacking such integration as would lead to an effective
pollution control mechanism. Without prejudice to such criteria as may be agreed upon by
the international community, or to standards which will have to be determined nationally,
it will be essential in all cases to consider the systems of values prevailing in each country
and the extent of the applicability of standards that are valid for the most advanced
countries but may be inappropriate and of unwarranted social cost for the developing
countries.
ActivityThe
19.1
above passages describe the basis for action on environmental pollution and
hazards. What are seen as the causes of health and environmental problems, what are
seen as the solutions, and by what philosophy or culture might these views be inspired?
Feedback
Problems are attributed to insufficient and inappropriate development, over-
consumption, and an expanding population. The solution is seen as lying in intersectoral
efforts and cooperative planning which enable people to bring about sustainable devel-
opment. The reference to population growth is reminiscent of the hierarchist view of
global problems as described by cultural theorists in Chapter 13. According to that
analysis, the ‘population story’ emerges from hierarchically structured institutions, and
the heroes would be those organizations with the capacity (technical knowledge) and
‘right’ sense of moral responsibility to sort it out. Nonetheless, this apparent hierarchist
tendency is softened by the evident desire to involve communities, though how this
links to the end game of sustainable development is not explained.
ActivityThe
19.2
following extract describes the objectives and activities of this part of the pro-
gramme. What is the nature of the approach to environmental health improvement?
Reducing
Objectives
health risks from environmental pollution and hazards (Agenda 21)
The overall objective is to minimize hazards and maintain the environment to a degree that
human health and safety is not impaired or endangered and yet encourage development to
proceed. Specific programme objectives are:
(a) By the year 2000, to incorporate appropriate environmental and health safeguards as
part of national development programmes in all countries;
(b) By the year 2000, to establish, as appropriate, adequate national infrastructure and
programmes for providing environmental injury, hazard surveillance and the basis for
abatement in all countries;
Issues of local policy 249
(c) By the year 2000, to establish, as appropriate, integrated programmes for tackling
pollution at the source and at the disposal site, with a focus on abatement actions in all
countries;
(d) To identify and compile, as appropriate, the necessary statistical information on health
effects to support cost/benefit analysis, including environmental health impact assess-
ment for pollution control, prevention and abatement measures.
Activities
Nationally determined action programmes, with international assistance, support and
coordination, where necessary, in this area should include:
(a) Urban air pollution:
i Develop appropriate pollution control technology on the basis of risk assessment
and epidemiological research for the introduction of environmentally sound
production processes and suitable safe mass transport;
ii Develop air pollution control capacities in large cities, emphasizing enforcement
programmes and using monitoring networks, as appropriate;
(b) Indoor air pollution:
i Support research and develop programmes for applying prevention and control
methods to reducing indoor air pollution, including the provision of economic
incentives for the installation of appropriate technology;
ii Develop and implement health education campaigns, particularly in developing
countries, to reduce the health impact of domestic use of biomass and coal;
(c) Water pollution:
i Develop appropriate water pollution control technologies on the basis of health
risk assessment;
ii Develop water pollution control capacities in large cities;
(d) Pesticides:
Develop mechanisms to control the distribution and use of pesticides in order to
minimize the risks to human health by transportation, storage, application and residual
effects of pesticides used in agriculture and preservation of wood;
(e) Solid waste:
i Develop appropriate solid waste disposal technologies on the basis of health risk
assessment;
ii Develop appropriate solid waste disposal capacities in large cities;
(f) Human settlements:
Develop programmes for improving health conditions in human settlements, in par-
ticular within slums and non-tenured settlements, on the basis of health risk
assessment;
(g) Noise:
Develop criteria for maximum permitted safe noise exposure levels and promote
noise assessment and control as part of environmental health programmes;
(h) Ionizing and non-ionizing radiation:
Develop and implement appropriate national legislation, standards and enforcement
procedures on the basis of existing international guidelines;
(i) Effects of ultraviolet radiation:
i Effects of ultraviolet radiation: Undertake, as a matter of urgency, research on the
effects on human health of the increasing ultraviolet radiation reaching the earth’s
surface as a consequence of depletion of the stratospheric ozone layer;
ii On the basis of the outcome of this research, consider taking appropriate
remedial measures to mitigate the above-mentioned effects on human beings;
250 Initiatives – local to global
Feedback
The approach is technocratic and inspired by RAP-like thinking.
ActivityIf19.3
you can visit the UNDP’s Capacity 21 website (UNDP 2006) and, taking the widely
acclaimed Costa Rican programme as an example, make notes of how that programme
strove to meet the three principles of Capacity 21, that is, to ensure participation of all
stakeholders in programme development, implementation, monitoring and learning; to
integrate economic, social and environmental priorities within national and local pol-
icies, plans and programmes; and to provide information about sustainable development
to help people make better decisions.
Feedback
These objectives were addressed by generating a new National Environmental Action
Plan and setting up a National Council of Sustainable Development (CONADES) to
foster collaboration between government and society. Major efforts were made to
create public awareness of the work and encourage local participation, also seen as a
means of safeguarding the programme into the future. A network of national organiza-
tions was also established for the purpose of ensuring that sustainability remained a
priority. Technical expertise was garnered from a team of national experts providing
input on green accounting systems, biodiversity and climate change etc.
It should be borne in mind that while the international community has given its
support to Agenda 21 because of its strong commitment to the idea of local
involvement, not all programmes have been seen as successful as that of Costa Rica,
and some have been regarded as failures. While this could be put down to teething
problems, there remain real questions about how Agenda 21 works and whether its
benefits could ever be measured. Nor is it fully understood how or even if sustain-
able development can truly be achieved (notwithstanding this, there appears to be
a consensus that it can only occur at global level if first it takes hold at the local
level).
Consistent with Agenda 21, a plethora of programmes, some informal and many
associated with the WHO, have been instigated. These come under such names as
Healthy Cities, Healthy Islands, Healthy Villages, and Safe Communities.
252 Initiatives – local to global
ActivityDo19.4you live in a city, a village, or on an island? Think about what your vision of a ‘healthy
city/village/island’ would be and answer the following questions:
1 What are the most important things you would want done to make it better?
2 How could this be achieved?
3 Is your choice of what to do based more on evidence or more on values?
4 Would you anticipate any opposition, and if so from where?
5 Would you use scientific evidence to support your case?
Feedback
Because of the range and unpredictability of answers, only some very general pointers
can be given. What you should notice though, in trying to do this exercise, is that your
values are crucial determinants of what you want to do and how you propose to go
about it. Otherwise, how could you say that your healthier vision was the best that
could be achieved? Is your vision of a place which maximizes equality and in this way
reduces health differentials, or is it one which tries to create greater wealth through
work opportunities, thereby improving health and well-being? Or do you aspire for a
simple(r) more environmentally-friendly way of life? Would you achieve your goal by a
bottom-up, community-led approach involving social change, or would a top-down
imposition of ‘rational’ health advancement measures be your preference? As you
know, all these approaches indicate political leanings of one kind or another. There-
fore, in normal circumstances, you should expect opposition from those who have
other leanings and beliefs about how things work and should be done. As for the
matter of evidence, many things that we think, or are firmly convinced, are worth
doing are very hard to prove when it comes down to it. Some of the remaining case
histories in this chapter are evidence of this. Much, it seems, rests on what is believed
to be true.
Following on from the launching by the WHO of the Healthy Cities initiative
in 1988, the Safe Communities movement was also adopted the following year
with the Karolinska Institutet in Sweden acting as Collaborating Centre. The Safe
Community concept focuses upon injury prevention and relies upon community-
based initiatives. By 2004, 83 communities from Scandinavia to China, South
Africa and New Zealand had been designated as Safe Communities (Rahim 2004).
ActivityAs19.5shown in the ROAMEF cycle of Figure 8.1, an important task for policy makers,
though one which is often neglected because it is difficult and time-consuming, is the
evaluation of interventions. In the first of two passages (adapted), Carolyn Coggan and
Issues of local policy 253
the rest of Auckland increased in 1996, decreased slightly in 1997, and then increased again
in 1998. There was no significant difference between the communities in the change in
slopes during the intervention/post-intervention period. Compared with pre-intervention
and comparison data, a highly significant increase in awareness of injury prevention was
found. Increases in awareness of the WCIPP and injury prevention among Waitakere
organisations were also evident.
In summary
In this first outcome evaluation of the WHO Safe Communities model in New Zealand,
significant reductions in the rate of injury hospitalisations for children were achieved in
Waitakere, whereas the comparison community showed an increase. This is an important
finding as child safety was one of the major campaigns conducted by the WCIPP. Reduc-
tions in serious injuries have not yet been able to be demonstrated in similar projects
conducted in Australia.
The WCIPP was not able to demonstrate significant reductions in injury hospitalisations
rates for all age groups. However, it is very encouraging that Waitakere was the only
community to show a decrease in injury hospitalisation rates for two subsequent years
(1997/98).
The WCIPP was unable to replicate findings of significant reductions in self reported
injuries for either adults or children. It is unclear why this has occurred. One explanation
could be that the activities of the WCIPP have resulted in more Waitakere residents being
aware of injury as a public health issue and this knowledge resulted in a more accurate
recall of self reported injury. The WCIPP was successful in raising awareness of injury
prevention among individuals and community organisations.
Issues of local policy 255
Feedback
1 The approach had three strands. One was to look at essentially administrative
aspects of the project, the second was to look at injury trend data, and the third was to
search for behavioural changes in organizations. Importantly, and as a means of trying to
increase confidence that any observed trends in Waitakere were attributable to the
intervention, the study also looked at similar communities and institutions outside of
the WCIPP so that comparisons could be made.
2 The strongest conclusion statistically was that there was an increase in awareness of
injury prevention. The actual description of the injury trend data in the communities is
not very convincing since only for children was a favourable trend discovered and the
time frame was short (Figure 19.1). The authors were nonetheless quite confident that
the programme had had a positive effect upon child injury hospitalizations.
The second passage is from a review, by Per Nilsen (2004), in which he evaluates 16
community injury prevention programmes from around the world. Nilsen con-
cludes as follows:
Ultimately, this study demonstrates the current difficulties of finding scientific evidence to
support the community-based approach to injury prevention programming. Because of the
inherent complexity of multistrategy and multitarget programs, and the interdependency
between contextual, structural, and process aspects, it is still difficult to provide solid
evidence describing the factors that are most important to achieve program effectiveness.
Some of the problems of identifying key success factors are due to meagre descriptions of
community characteristics and conditions, insufficient assessment of structural program
components, and failure to establish process–outcome relationships. To advance the
understanding of community-wide injury prevention programs, the evaluations that
accompany them must develop in complexity.
These findings are clearly more sobering. Nilsen points to the difficulties, and
sometimes even the dangers (since some of the interventions had apparently had
adverse effects), of community intervention programmes. He found in particular
that the most successful programmes had occurred in more homogeneous commu-
nities (in terms of ethnicity and socio-economic status). Programmes, it seemed, had
to be adapted from community to community and country to country, and there
was not a standard WHO Safe Community model which would work everywhere.
ActivityIn19.6
reading the paragraphs by Andrew Kiyu and colleagues, comment on the nature of
the evidence presented and its limitations. What should be done?
Feedback
The evidence is clearly observational and largely qualitative. No pre-intervention
baseline had been established, and the work which was done was relatively ad hoc.
Evidence from similar communities that were non-participating, and from those new
to the programme, apparently supported the proposition that beneficial changes had
occurred as a result of the programme. What else could be done? Realistically it may be
that not much more could be done in the circumstances. Sarawak is probably not as
well established in terms of health surveillance as some other localities and the cost and
time to set up such systems might be better spent on other activities. And quantitative
data are not always what they are said to be. Human observation and insight are in
some circumstances, especially relatively novel situations, better at identifying and
recording what is important.
Issues of local policy 257
Summary
The political view is now widely held that community-level participation in policy
making, and the empowerment which this seemingly implies, have the potential
to be a major factor in improving health and achieving sustainability. As a con-
sequence, many countries have risen to the challenge posed by Agenda 21 and
similar initiatives. While this belief may well one day be proved to have been
correct, at present it appears to be more hypothesis than fact. Environmental
health and social issues are intertwined in complex ways which are seldom fully
understood. On the other hand, pressure always exists to show that programmes
satisfy the three goals of effectiveness, efficiency and equity. In the absence of a
deeper understanding of how complex social systems work and a lack, in most
settings, of appropriate baseline data and surveillance systems, environmental
health policy makers need to tread carefully between underselling their programmes
and making claims which might not stand up in the cold light of day.
References
Coggan C, Patterson P, Brewin M, Hooper R and Robinson E (2000) Evaluation of the Waitakere
community injury prevention project. Injury Prevention 6: 130–4.
Kiyu A, Steinkuehler AA, Hashim J, Hall J, Lee PFS, Taylor R (2006) Evaluation of the healthy
village program in Kapit District, Sarawak, Malaysia. Health Promotion International
21(1): 13–18.
Nilsen P (2004) What makes community based injury prevention work? In search of evidence
of effectiveness. Injury Prevention 10: 268–74.
Putnam RD (1995) Bowling alone: America’s declining social capital, Journal of Democracy 6(1):
65–78.
Putnam RD (2000) Bowling Alone: The Collapse and Revival of American Community. New York:
Simon and Schuster.
Rahim Y (2004) Safe community in different settings. International Journal of Injury Control and
Safety Promotion 12(2): 105–12.
Seedhouse DF (1997) Health Promotion: Philosophy, Prejudice and Practice. New York: Wiley.
United Nations Development Program (2006) Capacity 21 in Costa Rica. Available at: http://
www.capacity.undp.org/index.cfm?module=Projects&page=Project&ProjID=725
Issues of global policy
20
Overview
Protecting the global environment is now a major challenge of public policy and
international diplomacy. Notwithstanding the fact that there are serious matters
which need attention, the purpose of this chapter is to highlight the complex issues
involved in designing and promoting appropriate environmental health interven-
tions. The examples taken are stratospheric ozone deletion and greenhouse gas
emissions. You have already encountered the global climate change story, analysed
from the perspective of cultural theory, in Chapter 13. The emphasis here is upon
barriers to the creation of international agreements and treaties.
Learning objectives
Key terms
Countervailing risk A newly created risk that results from action to reduce a target risk.
Risk trade-off The change in the portfolio of risks that occurs when a new risk is generated
(knowingly or inadvertently) by an intervention to reduce a target risk.
Risk trade-off analysis An organized approach to identifying countervailing risks and assessing
the balance of the full risk portfolio associated with a policy intervention.
Target risks The risk that is the primary focus of an environmental health intervention.
The amount of UV reaching the Earth’s surface has been shown to correlate with
the extent of ozone depletion. In 1997, UV-B levels at the Earth’s surface continued
to rise at a rate of 2 per cent per annum. Increased UV levels at ground level pose
risks to human health, including increases in the incidence of certain types of skin
cancers, cataracts and immune deficiency disorders. Depletion of stratospheric
ozone also alters the temperature distribution in the atmosphere, resulting in
indeterminate environmental and climatic impacts.
Future perspective
Perhaps the single most successful international agreement to date has been the Montreal
Protocol.
(Kofi Annan, Secretary General of the United Nations)
In 1985, the Vienna Convention established mechanisms for international
260 Initiatives – local to global
co-operation in research into the ozone layer and the effects of ODS. 1985 also
marked the first discovery of the Antarctic ozone hole. On the basis of the Vienna
Convention, the Montreal Protocol on Substances that Deplete the Ozone Layer was
negotiated and signed by 24 countries and by the European Economic Community
in September 1987. The Protocol called for the Parties to phase down the use of
CFCs, halons and other man-made ODS.
The Montreal Protocol on Substances that Deplete the Ozone Layer is one of the
first international environmental agreements that includes trade sanctions to
achieve the stated goals of a treaty. It also offers major incentives for nations to sign
the agreement. The treaty negotiators justified the sanctions because depletion of
the ozone layer is an environmental problem most effectively addressed on the
global level. Furthermore, without the trade sanctions, there would be economic
incentives for non-signatories to increase production, damaging the competitive-
ness of the industries in the signatory nations as well as decreasing the search for
less damaging CFC alternatives.
CFCs are now identified as rogue chemicals with the capability of creating immense
harm. But they were not always perceived that way. In fact, in the 1930s, CFCs were
seen as wonder chemicals because they were non-toxic, non-flammable, could be
used as refrigerants in place of the highly toxic ammonia compounds then in use,
and, so far as anyone knew, were inert. Perhaps this error can be excused, for the
chemistry of the upper atmosphere was little understood at that time, but there is a
lesson which should be acknowledged, and that is that well-meant interventions
may have disturbing side-effects. The implication is that interventions should be
screened for their potential to cause unexpected harm, just as health care patients
should be warned about the side-effects of prescription drugs.
ActivityRead
20.1
the following abridged passages from the Government of Alberta and from
Jonathan Wiener, and note the concern about possible new (countervailing) risks cre-
ated by CFC replacements. Weiner’s analysis was published in 1995 and some things
have since changed, but the central point here is not the detailed technical content but
the existence of risk trade-offs:
Issues of global policy 261
1 In the light of this information, what questions should a policy maker ask?
2 The example here is specifically about ozone-depleting compounds but potentially
the problem of creating new risks while removing others is much more widespread.
Write down a list of six such situations from as wide a range of environmental
health areas as your imagination allows.
Feedback
The new risks identified are health and safety through, mainly, occupational exposure;
implications for food spoilage in developing countries; and possible impacts on global
warming.
1 Policy makers are always in a situation which involves trade-offs. Sometimes they
are simple and obvious as in the case of investing finite resources for health gains, but
very often there are additional unwanted side-effects and these are not necessarily
262 Initiatives – local to global
small. The advice is that all significant interventions should be scrutinized for their
likely non-target effects, and these should be assessed so that a full balance sheet of
associated gains and losses can be set up before a decision on whether to go ahead
is made.
2 The problem of risk trade-offs, the creation of new risks while dealing with old
risks, is ubiquitous. A few examples, by way of illustration, include: prohibiting the use
of DDT on environmental grounds increases the number of cases of malaria because
DDT is cheap and effective and changes the risk to farmers who must use more toxic
organophosphate products; removing lead from gasoline results in higher fuel con-
sumption and hence higher emissions of toxic gaseous pollutants; restrictions on the
movements of nuclear waste (mainly proposed on ethical grounds) means that nuclear
waste is not always dealt with by those best able to do so; the imposition of strict safety
standards on children’s playgrounds results in many playgrounds being closed; the
chlorination of drinking water to combat waterborne microbial disease poses a (small)
cancer risk; evacuating people from their homelands because of some environmental
hazard greatly increases stress and associated health effects, and deprives them of their
heritage; refusal of permission on health grounds to build a waste incinerator may mean
it has to be disposed of by landfill. In each case, it should be incumbent on the policy
maker to assess the benefits and disbenefits, including costs, prior to deciding what
to do.
John Graham and Jonathan Wiener (1995) state that their research finds that risk
trade-offs are not an imagined and inevitable perversity of life but rather a real
consequence of incomplete decision making, and that with attention and effort,
policy makers can wage the campaign to manage environmental health with better
tools that help to recognize and progressively reduce overall risk. This, they say, is
important because in their view risk trade-offs are seriously hindering campaigns to
improve health. Thus, Graham and Wiener propose a framework for systematic
‘risk trade-off’ analysis which policy makers could apply at any level to environ-
mental health problems.
ActivityIn20.2
reading the following passage adapted from Graham and Wiener (1995) think
about:
1 How their approach aligns itself with the concerns expressed by Mary O’Brien in
Chapter 16.
2 Identify interventions which give rise to each of the four varieties of risk trade-off
shown in Figure 20.1. You could start with those you identified in Activity 20.1, or
those listed in the feedback.
3 How does the proposition that risk trade-off analysis amounts to front-loading the
policy process tie in with other changes in the approach to environmental policy
making which you have read about?
Issues of global policy 263
describing the phenomenon. Here, the term ‘target risk’ refers to the risk that is the
primary focus of the policy whereas the ‘countervailing’ risk is the chance of an adverse
outcome that results from the attempt to reduce the target risk.
Feedback
1 Though Graham and Wiener have a different approach and possibly a different
philosophy, you will observe that their message shares common ground with that of
Mary O’Brien whose strong concern was that the scope of risk assessment needed to
be broader and more flexible, in order to allow consideration of non-target risks and
alternatives.
2 Regarding the activities listed in the feedback: the DDT example is one of risk
transformation because different people are affected in new ways; lead removal from
petrol is a risk offset because one pollutant is replaced by others; restrictions on
nuclear waste movements mean that different people bear the burden of the same risk
and constitutes a risk transfer; the children’s playground example is one of risk substitu-
tion because children are protected from injury but have less opportunity for exercise
leading to reduced health; the chlorination example is one of risk offset; and the case of
evacuation affects the same population but in very different ways and so is a risk
substitution.
3 The proposition fits in well with the trend in thinking towards a more holistic
approach to the framing of policy issues. This is illustrated, for example, by Figure 17.1
as compared with Figure 3.2, and by Figure 17.2, as well as ideas brought to bear in
Chapters 14 to 16.
ActivityGiven
20.3
that policy will frequently run into the problem of risk trade-offs, decision
makers are going to have to choose between different risk types. On what basis could
this be done? What factors should be considered?
Feedback
Policy makers will in most cases be unable to eliminate all countervailing risks. To
decide if an intervention is overall beneficial or not, the characteristics of the reduced
target risk and the newly created risks must be compared. Obvious parameters for
consideration are the magnitude (probability of occurrence) of each risk, the size of
population exposed, the type of consequence expected, the reliability of the forecasts,
and equity including distributional considerations (who bears each risk). Less obvious
is the degree to which qualitative factors of the kind described by Paul Slovic and
colleagues in Chapter 12 should influence decisions.
Issues of global policy 265
Graham and Wiener’s model of risk trade-offs, with its all-too-obvious grounding
in reality, could be seen as disheartening by policy makers. Undoubtedly it imposes
a serious challenge, but, as is sometimes correctly recognized, ‘Good decisions are
hard to make’, and no-one ever promised that the environmental health policy
makers lot was going to be an easy one, why else would Sarah Lichtenstein and
colleagues (Chapter 18) express their compassion for them? On a more optimistic
note, Graham and Wiener have this to say:
we believe that contemporary social systems leave room for improvement and that, with
intelligent efforts, superior choices can be found through which overall risk can decline.
‘Risk reduction’ is thus a meaningful goal: choices can be made that on balance, make the
world safer for humans and other life forms.
Just as ‘market failures’ occur when markets fail to maximize social well-being
because the full set of consequences is not considered by market decision makers,
so may risk trade-offs occur if policy makers fail to take proper account of the
externalities which derive from their interventions. But why does this happen?
Why are decisions made without due reflection on the full range of impacts?
Graham and Wiener (1995) offer the following suggestions:
Omitted voices
This refers to the absence of some affected parties from the decision process who
may have particular interests at stake or who may have specific useful knowledge.
Or, as cultural theory reminds us, it could be to do with people who hold a particu-
lar world-view and which should at least be recognized in the decision process. As
you have read, front-end framing of decision processes to include stakeholders and
public interest groups is now intended to address this deficiency.
Use of heuristics
You will recall from Chapter 12 that heuristics are simplified mental models that aid
decision processes. When policy makers are presented with large bodies of informa-
tion there is a tendency to focus on the problem of immediate concern, leaving the
wider picture for another day. A variant of this is the tendency to focus on events
which are salient, especially in the aftermath of some disaster, like a train crash, or an
unusual event, such that issues of greater long-term significance are lost from view.
Bounded specialization
specific remits, or may have structured themselves in such a way as to have created
an infrastructure each part of which is circumscribed in what it can consider and
do. This ‘bounded specialization’ phenomenon is deeply rooted in many agencies
dealing with human and environmental health. There are obvious benefits asso-
ciated with such an approach, which is why it is so widespread, for example, the
capability to understand specific problems in depth. However, the loss is that
decision makers may be ‘blind’ to risks just outside of their jurisdiction.
Uninformed people
A further explanation which should not be discounted is that people may simply
be uninformed or not want to know. The story of methyl bromide, a powerful
ozone-depleting chemical used as a pesticide, as recounted by Mary O’Brien, dem-
onstrates how these factors could in that case have led to a preservation of the
status quo rather than a full examination of the possibilities.
Methyl bromide
From 1992 to 1997 I served as a member of the Methyl Bromide Technical Options
Committee. This Montreal Protocol Committee was charged with assembling a report on
existing and potential alternatives to using the potent, ozone-depleting pesticide, methyl
bromide, as a soil fumigant (i.e., to kill all soil organisms with poisonous fumes); structural
fumigant (e.g., in ships’ holds, granaries, and homes); and fumigant of durable and perishable
commodities of international trade such as wood, grains, fruits, vegetables and flowers. In
other words, the Committee was charged with conducting an alternatives assessment. Its
report would be used to help determine the schedule of a worldwide phaseout of methyl
bromide use.
The process of exploring alternatives to methyl bromide within the Committee was
eye-opening. Only a small minority was publicly supportive of alternatives, and even fewer
knew of non-chemical alternatives to the pesticide. However, the Committee’s mandate
to discuss all feasible technical options lent clout to the minority of members who did
have knowledge of non-chemical alternatives. If a committee member could provide
evidence that a particular alternative was in use or close to availability, it had to be
entered into the report. The definition of an alternative to methyl bromide adopted by
the Committee was also critical: ‘A technology demonstrated or in use in one region of the
world that would be applicable in another unless there were obvious technical constraints
to the contrary.’
Notably, economic considerations were dealt with elsewhere, so it was possible to discuss
all options. The resulting picture of alternatives, including non-chemical alternatives, is
impressive. The Committee revealed that technically, alternatives already existed for 90%
by volume of worldwide methyl bromide use.
The US Department of Defense, for instance, together with shipping companies and the
University of California at Davis, has been successful in developing an alternative to one use
of methyl bromide. Prior to 1994, the DoD had been shipping fresh produce by air to US
military personnel stationed overseas. The produce was fumigated with methyl bromide
to quickly kill insects or mites prohibited in the country to which the produce was being
shipped.
Issues of global policy 267
While concern about the ozone layer and greenhouse gas accumulation in the
atmosphere, fed by scientific research, is growing, there remains a further highly
problematic stage prior to the necessary global-scale action being taken. This is the
negotiation of international agreements which are binding and effective.
ActivityThe
20.4
following passage adapted from an article by Joseph Bial and colleagues (2000)
describes factors that influence the translation of environmental concerns into policies
and international treaties. The example used here, which exhibits the full range of
factors, is the accumulation of greenhouse gases in the atmosphere. In reading the
extract make notes on the following:
1 What do you understand by ‘an open-access resource problem’?
2 Why is it so difficult to negotiate international agreements on global environmental
problems?
3 Under what circumstances is it easier to reach agreement over collective action?
released as by-products of human activities and other natural sources across countries.
CFCs are the most potent per molecule, but CO2 is the most abundant. Regardless of their
origin the gases are spread around the globe with potential external effects. The gases
retard the re-radiation of the sun’s energy from the earth’s surface back into space. Under
debate are whether and how much the further accumulation of these gases from human
actions will generate a damaging rise in global temperatures. The causal relationship
between the build-up of greenhouse gases and an increase in global temperatures has
not been defined conclusively and hence is subject to debate. Nevertheless, the expected
linkages have stimulated international efforts to control emissions and thereby mitigate any
possible rise in temperatures.
Most of the focus has been on CO2, which is released from burning fossil fuels, such as coal,
and the USA is currently the largest emitter. At least some restructuring of the American
and other industrial economies likely will be necessary with a possible reduction in gross
domestic product (GDP). The macroeconomic effects, however, are uncertain because
they depend on each country’s energy intensity of production, energy sources, and the
magnitude and pace of emission reductions implemented. On a microeconomic level, there
will be important distributional effects within and across countries, both from global warm-
ing and from regulation. Some countries appear to be more vulnerable to any negative
implications of global warming and within countries, energy-intensive industries are apt to
bear the brunt of emission controls. Taxpayers may be called on to fund the implementa-
tion and monitoring of regulations, and to pay for compensating transfers to sectors
harmed. Further, they may be required to pay for side payments to other countries as
inducements to participate in collective action. These heterogeneous constituent effects
and the uncertainty confronted by each party in calculating the net effects of the treaty
create political problems for politicians with implications for the success of international
collective action.
Effective collective action to address potential climate change will be a formidable chal-
lenge. The very nature of global environmental externalities presents incentive problems.
Abatement by any country benefits others as a public good, but if abatement is costly to
a country’s citizens, its politicians have incentive to invest less in reduction efforts than
would be globally optimal and free ride on cutbacks taken elsewhere. Research on collect-
ive action to address more tractable, local common-property resources indicates that
these incentive problems can occur even when there is agreement about the magnitude of
the problem and the aggregate benefits of resolving it. These bargaining issues are more
complex in international environmental agreements where the benefits and costs are very
uncertain and differentially spread across and within countries, and where compliance is
voluntary.
Nor do international negotiations take place detached from the underlying political real-
ities within each of the bargaining countries. Rather, a country is represented by an agent
who is accountable to domestic constituencies. This agent must adopt an international
negotiating position that simultaneously leads to a resolution of the international common-
property problem and generates the greatest political support among his/her electorate.
Thus, the benefits and costs of an international treaty are borne not only by the underlying
constituencies, but also by the elected agent. Hence, the agent will be very cautious before
committing his/her country to an agreement because imposing even minor costs on a
constituency without commensurate benefits may lead to large defections in political
support. The political problem faced by politicians is exacerbated if there is considerable
uncertainty in estimating constituent benefits and costs from international action.
Issues of global policy 269
Feedback
1 This refers to a resource which is freely available for everyone to use, and sometimes
referred to as ‘the tragedy of the commons’.
2 International treaties to control environmental pollution have macroeconomic
effects on countries which will affect their GDP, and microeconomic effects which raise
distribution of wealth issues both within and between countries. Some of these issues
can be dealt with by transfer payments, but these are hard to negotiate especially where
the impacts of agreements are uncertain. Furthermore, politicians who negotiate
such agreements are likely to be very cautious about agreeing anything which will be
seen to have an undesirable effect on their constituencies. In the case of global
environmental issues like greenhouse gas warming, and to a lesser extent the ozone
layer, there is no short-term gain but a high up-front cost. Such ‘deals’ are usually
regarded as unfavourable.
3 Collective agreements could be expected to be simpler to conclude in situations
where there is a consensus about the aggregate benefit of some intervention; the
parties all perceive positive net gains for their constituency from the agreement; the
parties have common bargaining objectives; there are no large distributional issues.
Agreements under these conditions tend to be easier because it is in the interest of
all parties. In situations where distributional issues arise, these may be resolvable by
transfer payments, but this is only likely if the aggregate benefit of the international
agreement is positive.
Summary
Environmental health issues at the global level present policy makers with their
most formidable challenge. On the one hand, interventions on this scale will
inevitably prompt effects which are unintended and undesired, as well as bringing
about the benefits sought. Policy makers will need to take a very broad perspective
when contemplating interventions on this scale, and must expect to be in a pos-
ition of weighing up the advantages and disadvantages of alternative policies.
Nonetheless, there will be many situations in which policy makers will find
themselves constrained, or in situations in which other parties will want to take a
narrower view. In most circumstances, this tendency should be resisted. The nego-
tiation of international agreements to implement policy should be expected to be
particularly demanding given that there will be imperatives on both the domestic
and the international fronts which will in many cases be difficult to reconcile.
270 Initiatives – local to global
References
Bial JJ, Houser D and Libecap GD (2000) Public choice issues in international collective action:
global warming regulation. http://citeseer.ist.psu.edu/bial00public.html
Europa (2005) The Ozone Layer. Available at: http://www.europa.eu.int/comm/environment/
ozone/ozone_layer.htm
Government of Alberta (2004) Health and Safety Issues Associated with the Refrigerant HCFC-123.
Edmonton: Government of Alberta.
Graham JD and Wiener JB (1995) Risk Versus Risk: Tradeoffs in Protecting Health and the
Environment. Cambridge, MA: Harvard University Press.
O’Brien M (2000) Making Better Environmental Decisions: An Alternative to Risk Assessment.
Cambridge, MA: MIT Press.
Slovic P, Fischhoff B and Lichtenstein S (1980) Facts and fears: understanding perceived risk,
in Schwing RC and Albers WA, Societal Risk Assessment: How Safe Is Safe Enough? New York:
Plenum.
Wiener JB (2005) Protecting the global environment, in Graham JD and Wiener JB Risk Versus
Risk: Tradeoffs in Protecting Health and the Environment. Cambridge, MA: Harvard University
Press.
World Bank (1992) World Development Report: Environment and Development. Washington, DC:
World Bank.
Glossary
Collective dose The sum of the radiation doses to the individuals within a speci-
fied group, measured in units called man-sieverts.
Communitarianism A philosophy that values communities and resists the idea
that individuals and free market exchanges are the key social institutions.
Confounding variable (confounder) A variable that is associated with the
exposure under study and is also a risk factor for the outcome in its own right.
Conservatism Assuming a worst case scenario in order to err on the side of safety.
Contingent valuation Survey approach to asking individuals to imagine markets
exist and to give their willingness to pay for (accept) benefits (losses).
Countervailing risk A newly created risk that results from action to reduce a target
risk.
Cross-sectional study A study design where exposure and outcome are measured
at the same time.
Cultural theory A social theory that proposes that social responses to risks are
determined by cultural belief patterns and not objective facts about risk.
Deliberation A reciprocal exchange of ideas between the public, interest groups
and policy makers.
de manifestis When applied to risk, refers to levels which are manifestly
intolerable.
de minimis When applied to risk, refers to levels which are considered trivial or
negligible.
Deoxyribonucleic acid (DNA) The molecule in which the genetic blueprint for
living cells is encoded.
Deterministic effects Effects which can be directly associated with their causes
and which generally increase as dose increases. Normally a threshold exists.
Dose A stated quantity or concentration of a substance to which an organism is
exposed over either a continuous or an intermittent period. Dose is most com-
monly measured in units of milligrams per kilogram of body weight of the receiver
(mg/kg bw).
Dose equivalent A measure of the amount of radiation absorbed by tissue com-
bined with a factor accounting for the ability of the type of radiation involved to
cause damage, measured in units called sieverts.
Dose limit The officially recognized maximum tolerable dose of ionizing radiation
during a specified time interval.
Dose–response assessment The determination of the relationship between
administered doses and the incidence or severity of the associated adverse effects.
Ecological fallacy The effects measured in groups may not be applicable at the
level of individuals.
Effective dose equivalent The dose equivalent incorporating a further weighting
to account for the vulnerability to harm of the body part exposed, measured in
units called sieverts.
Egalitarianism A moral perspective emphasizing equality of people.
Environmental agent Any chemical, biological or physical substance, or social
factor, which is under investigation.
Glossary 273
Environmental health Those aspects of human health and disease that are
determined by factors in the environment.
Environmental impact assessment A technique and process by which informa-
tion about the environmental effects of a project is collected.
Environmental medium A specific part of the environment such as air, water or
soil, sometimes referred to as an environmental compartment.
Epidemiology The study of the distribution and determinants of health states or
events in specified populations, and the application of this study to the control of
health problems.
Ex-ante decisions Decisions that are made in advance of some potential outcome.
Experiential learning Learning from experience.
Exposure The degree to which a person is subject to a given risk factor.
Exposure assessment A qualitative or quantitative estimation of the magnitude,
frequency and duration of exposure of the general population, sub-groups or
individuals, to hazardous substances or situations.
Exposure pathway The means by which risk agents come into contact with
target organisms, e.g. via drinking water, inhalation, dermal contact, or physical
proximity, etc.
Exposure route How an agent enters the body (such as by inhalation, ingestion,
contact).
Expressed preference studies Studies that elicit consumers’ preferences by asking
them questions about real or hypothetical scenarios.
Externality Cost or benefit arising from an individual’s production or consump-
tion decision which indirectly affects the well-being of others.
Genotoxic A chemical that can cause damage in genetic material leading to herit-
able changes.
Governance The process of governing, or of controlling, managing, or regulating
the affairs of some entity.
Hazard A factor or exposure that may adversely affect health.
Hazard identification The identification of adverse health or environmental
effects which could be associated with an agent or situation should exposure arise.
Hazard index (HI) The quotient of the maximum daily dose and the acceptable
daily intake of an hazardous material.
Health impact assessment A method which aims to identify the likely changes in
health risk of a policy, programme, plan, or development action on a defined
population.
Human exposure – rodent potency index (HERP) An index devised for ranking
the risk of potential human carcinogens based on average human exposure and
rodent toxicity data.
Incidence The frequency of new cases in a defined population during a specified
period of time.
in vitro A laboratory test using living cells taken from an organism.
in vivo A laboratory test carried out on living organisms such as whole animals or
human volunteers.
274 Glossary
Ionizing radiation Radiation that is sufficiently energetic to break the bonds that
hold molecules together to form ions.
LD50 The dose that when administered to animals in a test is lethal to 50 per cent.
Linear no threshold hypothesis The presumption that risk is linearly related to
dose at low doses for non-threshold agents.
Lowest observed adverse effect level The lowest experimental or observed con-
centration or amount of a substance that causes adverse alterations in target
organisms.
Multi-hit model Dose–response models that assume that more than one interac-
tion with a toxic material at the molecular level is necessary to cause an effect.
Mutagen An agent which can cause genetic damage to individual cells.
No observed adverse effect level (NOAEL) The highest dose at which no signifi-
cant adverse effects are noticed in a population.
Objectivism The idea that risks can be measured and that we can distinguish
between their real magnitude and what people variously and perhaps erroneously
believe them to be.
One-hit model Dose–response models that assume that a response occurs after a
target has been impacted once at the molecular level by a biologically effective
dose.
Opportunity (economic) cost The value of the next best alternative foregone as a
result of the decision made.
Pareto rule An intervention satisfying this rule would improve the well-being of
some people without harming anyone else.
Policy Broad statement of goals, objectives and means that create the framework
for activity. Often take the form of explicit written documents but may also be
implicit or unwritten.
Potential Pareto improvement (Kaldor–Hicks principle) The basis for cost–
benefit analysis. It stipulates that a reallocation of resources which makes someone
better off and someone worse off represents an improvement only as long as those
who gain could potentially compensate those who lose.
Precautionary principle A principle that advocates the use of prudent social
policy in the absence of empirical evidence in an attempt to solve a problem.
Prevalence The frequency of existing cases in a defined population at a particular
point in time (point prevalence), or over a given period of time (period prevalence),
as a proportion of the total population.
Probability A quantitative or qualitative expression of the likelihood or chance
that a particular outcome will occur as a result of a specified cause or action.
Psychometric paradigm An approach to understanding public attitudes to differ-
ent risks based on the use of quantitative techniques.
Psychometrics The science of psychological measurement.
Quality-adjusted life year (QALY) A year of life adjusted for its quality of its value.
A year in perfect health is considered equal to 1.0 QALY.
Rational actor paradigm (RAP) A sociological theory of human behaviour based
on people acting as self-seeking individuals and maximizing their personal utility.
Glossary 275
Reference dose An estimate of the highest daily dosage of a risk agent that is
unlikely to produce an appreciable harmful effect in humans.
Relativism An extreme doctrine in which anyone’s opinion is as good as anyone
else’s.
Revealed preference studies Studies of actual consumer behaviour which
indirectly reveal their preferences.
Risk The probability that an event will occur within a specified time.
Risk assessment A method for estimating the probability or likelihood of a speci-
fied type of harm occurring to an individual or a population.
Risk aversion The unwillingness of an individual to take on an identified risk.
Risk-based thinking An approach to decision making that relies in part upon risk
assessment.
Risk characterization A synthesis and summary of information about a poten-
tially hazardous situation that addresses the needs and interests of decision makers
and of interested and affected parties. Risk characterization is a prelude to decision
making and depends on an iterative, analytic-deliberative process.
Risk communication Any meaningful communication between parties over mat-
ters of risk; not restricted simply to one-way communication.
Risk governance Encompasses the totality of participants, rules, conventions,
processes and mechanisms concerned with the collection, analysis and communi-
cation of relevant information, and the making of policy decisions.
Risk management The use of risk assessment in combination with socio-economic
and political inputs to evaluate and select measures to manage risk.
Risk trade-off The change in the portfolio of risks that occurs when a new risk is
generated (knowingly or inadvertently) by an intervention to reduce a target risk.
Risk trade-off analysis An organized approach to identifying countervailing
risks and assessing the balance of the full risk portfolio associated with a policy
intervention.
Safety factor A single factor or several factors used to derive an acceptable intake
of a chemical.
Sievert (Sv) A unit of equivalent dose of radiation which relates the absorbed dose
in human tissue to the effective biological damage of the radiation. A millisievert
(mSv) is one-thousandth of a sievert.
Social capital The institutions, relationships, and norms that shape the quality
and quantity of a society’s social interactions.
Social constructivism This approach to knowledge denies the existence of reality
prior to human engagement and the validity of ‘truth’ in the sense of a correspond-
ing representation of reality. Instead, it poses that reality is whatever is known, and
that all knowledge is socially produced. Constructivism can thus lead to relativism,
as it allows no distinction between true and untrue statements.
Social impact assessment The process of assessing or estimating, in advance, the
social consequences that are likely to follow from specific policy actions or project
developments.
Stakeholder An individual or group with a substantive interest in an issue (i.e.
interest group), including those with some role in making a decision or its execution.
276 Glossary
‘acceptable daily intake’ (ADI) comparison with alternatives moral choices within, 198–200
measurement scale, 45 assessment, 205–7 procedure, 196–8, 197
accidents, prevention environmental health reasons for interest in, 18–21,
interest in, 118–19 applications, 84–8, 87, 140–42 19–20
policies, 119–20 analysis, risk see assessment, risk relationship with risk
typologies of, 120–23, 121, 122 analytic-deliberative process of management, 29
achievability environmental health policy see also assessment,
as radiation protection formulation, 217–9, 218, 219 environmental health;
principle, 111–14, 112, 114 Anderson, F., 198–9 exposure, hazards and toxins;
Adams, J., 159, 162, 164, 168 Annan, K., 259–60 Formal Safety Assessment;
ADI (‘acceptable daily intake’) appraisal, economic perceptions, risk;
measurement scale, 45 as applied to environmental ‘precautionary principle’
‘affect heuristic’ health policy, 82, 84, 84 assessment, social impact
role in perception of ‘risk’, 151, Arrow, K., 140–41 case study and characteristics,
153 Ashford, N., 205 182–5
Agenda, 21, 247–50 ‘as low as reasonably achievable’ atoms
agents, toxic (ALARA) principle of radiation role in ionizing radiation,
definition, 36–7 protection, 108–11, 111, 237 102–4, 103
implications for policy makers, ‘as low as reasonably practicable’ structure of, 103
48–9 (ALARP) principle of risk Australia
safety measurement, 44–5, 46, tolerability, 123, 237 case study of gold mining social
47, 50–51 assessment, alternatives see impact assessment, 184
see also chemicals, hazardous alternatives assessment
and toxic; exposure, hazards assessment, dose response Ball, D., 29, 83, 93, 96, 111,
and toxins as step in risk assessment, 32, 33 121
agreements, environment assessment, environmental see type Bara Operational Forest
protection eg environmental impact Management Plan (OFMP),
negotiation difficulties, 267–9 assessment; strategic 179–80
see also Montreal Protocol environmental assessment Bate, D., 88
ALARA (‘as low as reasonably assessment, environmental health BATNEEC (‘best practicable
achievable’) principle of case studies of impact, 187–8, technique not entailing
radiation protection, 108–11, 187, 188 excessive cost’) principle of
111, 237 definition and characteristics, risk tolerability), 125
ALARP (‘as low as reasonably 28–30, 29, 185–6, 186) Baxter, M., 181
practicable’) principle of risk process of, 32–4, 33 Berger pipeline inquiry (Canada),
tolerability, 123, 237 risk assessment within, 11–13, 184
alternatives assessment (O’Brien) 12, 23–4 ‘best available environmental
case study of process and use, assessment, risk option’ (BPEO) principle of
203–5, 204 critiques of, 138–9 risk tolerability, 125
comparison with cost-benefit definition, 28 ‘best practicable technique not
analysis, 205–7 evolution and history, 15–17 entailing excessive cost’
Ames, B., 191–2, 192 guiding principles within (BATNEEC) principle of risk
analysis, cost benefit/effectiveness environmental health, tolerability, 125
advantages, 205 74–5, 74 Bial, J., 267–9
278 Index
insurance and reinsurance London Dumping Convention environmental health risks and
need for, as risk assessment (1972), 207 policy formation, 9, 232–3
driver, 17 Lower Seyhan Irrigation Project environmental impact
International Atomic Energy (Turkey), 178–9 assessment, 175
Agency (IAEA), 20 ‘lowest observed adverse effect risk assessment and trade off, 29,
International Commission on levels’ (LOAELs) (dosage), 32–4, 33, 263–4, 264
Radiation Protection (ICRP), 130–31 risk management, 125–7, 127,
108–16, 111, 112, 114 ‘lowest observed effect levels’ 222–3
International Maritime (LOELs) (dosage), 44–5, 49 risk decision process and
Organization (IMO), 124, Lundgren, R., 225, 226 governance, 218, 219
124 risk perception, 155–6
International Risk Governance Malaysia see also theories
Council (IRGC), 218–9, 219, case study of ‘healthy’ villages see also perspectives eg
223, 224 programme, 255–6 psychometric paradigm;
ISOE (Information System on management, risk rational actor paradigm
Occupational Exposure), 114 definition and characteristics, monitoring (hazard exposure
28, 120–23, 121, 122, 222–3 assessment), 56–61, 57, 59,
Jaeger, C., 136–7 historical development, 216–19, 60
Joyce, S., 183–4 216, 217, 219 Montreal Protocol on Substances
levels of policy engagement, that Deplete the Ozone Layer
Kahneman, D., 145 220–25, 222–3 (UNEP, 1987), 259–60
Kaldor, N., 87, 88, 115 relationship with risk Morgan, M.G., 196, 197
Karolinska Institutet (Sweden), assessment, 29 Mumpower, J., 15, 16, 21–3
252 see also trade-offs, risk ‘myths of nature’ (cultural theory),
Khadka, R., 179–81 Management of Health and Safety at 159–62, 159, 162
Kiyu, A., 255–6 Work Regulations (1999), 20
Kocher, D., 125–7, 126, 130–31 Mäoris National Comparative Risk Project
case study of social impact (USA), 192–3, 193
LADD (‘lifetime average daily assessment, 182–3 National Council of Sustainable
dose’), 64 matrices, risk consequence/profile, Development (CONADES),
Langley, A., 59 120–21, 121 251
LaPlace, P.S., 16 ‘maximum daily dose’ (MDD), National Environmental Action
laws 64 Plan, 251
compliance with, as driver for measurement National Impregnated Bednet
risk assessment, 17 dose-response safety, 44–5, 46, Programme (Gambia), 85
learning, experiential 47, 50–51 National Radiological Protection
role in ‘risk’ perception, 153–6, radiation, 104–5, 104 Board (NRPB), 111–13
154, 156 tools see name eg Waitakere National Research Council (NRC)
Leiss, W., 217 Community Injury (USA), 28, 33, 217–18, 218,
liberalism (concept) Prevention Project 220, 223
as influence on policy making, Merkhofer, M., 61 Nepal
237–8 Mexico City Bara Operational Forest
Lichtenstein, S., 146–7, 147, 241 case study of air pollution Management Plan, 179–80
life, quality of control, 80–81, 97–9 New Zealand
assessment criteria, 197 mining, gold case study of injury prevention
‘lifetime average daily dose’ case study of social impact programme, 253–5, 254
(LADD), 64 assessment, 184 case study of Mäori social
linear no threshold (LNT) model modelling impact assessment, 182–3
of dose-response use in exposure assessment, Nilsen, P., 253
measurement, 46–7 58–61, 57, 59, 60 ‘no observed adverse effect levels’
LOAELs (lowest observed adverse models and typologies (NOAELs) (dosage), 44–5, 49,
effect levels) (dosage), 130–31 accident and injury prevention, 50, 64, 130–31
LOELs (lowest observed effect 120–23, 121, 122 NRC (National Research Council)
levels) (dosage), 44–5, 49 dose-response measurement, (USA), 28, 33, 217–18, 218,
Löfstedt, R., 227, 228–9, 233 46–7 220, 223
Index 281
rbGH (bovine growth hormone) safety, hazards and toxic agents environmental health
case study of use, 203–5, 204 guidelines, 49–50 policy, 11
Red Book (National Research safety, public and occupational see also policies, environmental
Council), 28 interest and policies, 118–23, health; protection,
reference doses (RfDs), 45 121, 122 environment
Relman, A., 21 see also risk Sweden
resources, allocation of Safety of Life at Sea (SOLAS) Karolinska Institutet, 252
as reason for interest in risk Convention, 20, 124
assessment, 18–20 Sandin, P., 208, 209 TDI/TWI (‘tolerable daily/weekly
RfDs (reference doses), 45 Sarawak (Malaysia) intake’ measures of hazardous
risk (concept) case study of ‘healthy’ villages agents, 45
changes in perception and programme, 255–6 ‘technocratic approach’ to social
coping, Schwartz, M., 136–7 impact assessment, 183–4
21–3 SEA (Strategic Environmental Tengs, T., 19–20, 20
characteristics, 30–31, 31, 222–3 Assessment) theories
theories of, 139–40, 139 case study and characteristics, risk, 139–40, 139
see also assessment, risk; 179–81 see also models and typologies
communication, risk; Seedhouse, D., 234, 236, 240 see also name eg cultural theory;
exposure, hazards and toxins; Sexton, K., 138 psychometric paradigm;
management, risk; Slovac, P., 148, 149, 150–51, 152, rational actor paradigm
perceptions, risk; trade-offs, 153 Thérivel, R., 179
risk Snyder, W., 62 Thompson, M., 136–7, 164, 168
Risk Governance (International Risk Soby, B., 93 ‘tolerable daily/weekly intake’
Governance Council), 218–9, social impact assessment (TDI/TWI) measures of
219, 224 case studies and characteristics, hazardous agents, 45
Risk Management in Post-Trust 182–5 ‘Tolerability of Risk’ (ToR)
Societies (Löfstedt), 228–9 SOLAS (Safety of Life at Sea) framework, 122–3, 122
Risk vs. Risk (Graham & Wiener), Convention, 20, 124 toxins and toxicity see agents,
263–5 Starr, C., 145 toxic
risks, environmental Stevens, G., 80, 97, 99 trade-offs, risk
aims of characterization of, Stokell, P., 109, 111 consequences of ignoring,
67–70, 68, 69, 70 Strategic Advisory Board (SAB) 265–7
prioritization of, 192, 193, 194, (USA), 193–6, 194, 196 models of, 263–4, 264
196 Strategic Environmental Travis, C., 127–30, 128, 129
taxonomies of health risks, 9 Assessment (SEA) trust (concept)
see also cancers; ranking, risk case study and characteristics, importance in risk
risks, radiation exposure 179–81 communication, 227–8, 228
assessing, 111–14,112, 114 studies, epidemiological and Turkey
limits of, 123–4 toxicological Lower Seyhan Irrigation Project,
ROAMEF (Rationale, Objectives, as method of hazard 178–9
Appraisal, Monitoring, identification, 38–40 Tversky, A., 145
Evaluation and Feedback) substances, hazardous typologies and models see models
cycle, 84 methods of identification, and typologies
Rodricks, J., 63–4, 65, 74 36–40
Rothstein, H., 176, 228–9 see also agents, toxic; chemicals, uncertainty
hazardous and toxic characteristics and role in risk
SAB (Strategic Advisory Board) substances, non-carcinogenic characterization, 71–5, 74
(USA), 193–6, 194, 196 environmental risk limits, Understanding Risk: Informing
Sadler, B., 181 130–31 Decisions (National Research
Safe Communities Initiative Sunstein, C., 209–10 Council), 220, 223
(WHO), 252–5, 254 sustainability United Nations
safety, dose-response as element in environmental Montreal Protocol on
threshold/non-threshold policy implementation, Substances that Deplete the
approach to identification, 246 Ozone Layer (UNEP, 1987),
44–5, 46, 47, 50–51 as guiding principle of 259–60
Index 283
World Charter for Nature see also ‘willingness to Wiener, J., 261, 262–6, 263
(1982), 209 accept/pay’ ‘willingness to accept/pay’
World Commission on Vermont’s Quality of Life Criteria, (concept)
Environment and 197 as driver of environmental
Development (1987), 246 health policy, 90–96, 93, 94,
United States of America Waitakere Community Injury 96
Environmental Protection Prevention Project (WCIPP), Workplace Safety Directorate (EU,
Agency, 192–3, 193 253–5, 254 1993), 20–21
National Comparative Risk water, supply of World Charter for Nature (UN,
Project, 192–3, 193 case study of health impact 1982), 209
National Research Council, assessment, 187–8, 187, 188 World Commission on
28, 33, 217–18, 218, 220, WCIPP (Waitakere Community Environment and
223 Injury Prevention Project, Development (UN, 1987),
Strategic Advisory Board, 193–6, 253–5, 254 246
194, 196 welfare, human World Health Organisation
utilitarianism (concept) as guiding principle of (WHO), 7–8, 20, 49, 68–9,
as influence on policy making, environmental health policy, 119, 252–6, 254
237–8 11 World Report on Road Traffic Injury
WHO (World Health Prevention (WHO), 119
‘value of a statistical life’ (VSL) Organisation), 7–8, 20, 49, World Report on Violence and Health
measurement tool, 98 68–9, 119, 252–6, 254 (WHO), 119
UNDERSTANDING
David Ball
PUBLIC HEALTH
UNDERSTANDING PUBLIC HEALTH
SERIES EDITORS: NICK BLACK & ROSALIND RAINE
Health Policy
will deal effectively with a seemingly Understanding Public Health
never-ending supply of hazards which is an innovative series of
impinge on health and wellbeing. This twenty books, published by
book provides a multidisciplinary window Open University Press in
onto environmental policy and its collaboration with the
formulation. From this you will observe London School of Hygiene
both order which exists at the centre and & Tropical Medicine.
controversies around the borders. It provides self-directed
This book considers: learning covering the major
◗ Key threats to human health from issues in public health
the physical environment affecting low, middle and
◗ Policies that might be pursued to high income countries.
minimise those risks The series is aimed at those
◗ How risks can be identified and studying public health,
quantified either by distance learning
◗ How such information can be or more traditional
communicated to the public methods, as well as public
◗ How health impact assessments can health practitioners and David Ball
be carried out policy makers.
◗ How risks can be managed and
regulated
PUBLIC HEALTH
UNDERSTANDING PUBLIC HEALTH
SERIES EDITORS: NICK BLACK & ROSALIND RAINE
Health Policy
will deal effectively with a seemingly Understanding Public Health
never-ending supply of hazards which is an innovative series of
impinge on health and wellbeing. This twenty books, published by
book provides a multidisciplinary window Open University Press in
onto environmental policy and its collaboration with the
formulation. From this you will observe London School of Hygiene
both order which exists at the centre and & Tropical Medicine.
controversies around the borders. It provides self-directed
This book considers: learning covering the major
◗ Key threats to human health from issues in public health
the physical environment affecting low, middle and
◗ Policies that might be pursued to high income countries.
minimise those risks The series is aimed at those
◗ How risks can be identified and studying public health,
quantified either by distance learning
◗ How such information can be or more traditional
communicated to the public methods, as well as public
◗ How health impact assessments can health practitioners and David Ball
be carried out policy makers.
◗ How risks can be managed and
regulated